12.07.2015 Views

GAZETA MATEMATIC˘A - SSMR

GAZETA MATEMATIC˘A - SSMR

GAZETA MATEMATIC˘A - SSMR

SHOW MORE
SHOW LESS

You also want an ePaper? Increase the reach of your titles

YUMPU automatically turns print PDFs into web optimized ePapers that Google loves.

<strong>GAZETA</strong> MATEMATICĂSERIA AANUL XXVII(CVI) Nr. 3 / 2009Număr închinat împlinirii a 100 de ani de la înfiinţareaSocietăţii Gazeta MatematicăARTICOLE ŞTIINŢIFICE ŞI DE INFORMARE ŞTIINŢIFICĂMetoda etichetării binare în probleme de combinatoricăVasile Pop 1)Abstract. In this paper we present some applications of binary methodin combinatorics. Many of this problems are highly difficult and some ofthem were given at olympids and international contestsKeywords: set chrcteristic function, binary methodMSC : O5A18, O5B20.IntroducereAvând o mulţime finită de obiecte identice ca formă, pentru individualizareaşi apoi pentru identificarea unui anumit obiect trebuie să facem oetichetare (codificare) a respectivelor obiecte. Cel mai simplu mod de etichetareeste de a atribui fiecăruia câte un număr natural, de exemplu de a leenumera: alegem un element din mulţime pe care punem eticheta 1, alegemal doilea şi punem pe el eticheta 2,..., alegem ultimul element şi punem peel eticheta n. Pentru etichetarea unui element arbitrar dintr-o mulţime decardinal 10 n − 1 avem nevoie, în scrierea zecimală dencifre din mulţimea{0, 1,...,9}, deci în total numărul simbolurilor folosite pentru codificareatuturor elementelor este 10n. Dacă facem codificarea folosind scrierea binară,atunci pentru un element arbitrar dintr-o mulţime cu 16 n − 1=2 4n −1elementeavem nevoie de 4n simboluri din mulţimea {0, 1}, deci în total folosim8n < 10n simboluri deşi am codificat o mulţime mai numeroasă. Acestargument justifică eficienţa codificării binare în raport cu codificarea într-oaltă bazădenumeraţie.În probleme de combinatorica mulţimilor etichetarea binară esteextremde naturală datorită funcţiei caracteristice a unei submulţimi, funcţie1) Universitatea Tehnică, Cluj-Napoca.


174 Articolecu valori de 0 şi 1, care caracterizează orice submulţime. Astfel dacă A == {a 1 ,a 2 ,...,a n } este o mulţime cu n elemente, atunci orice submulţimeB ⊂ A, din cele 2 n submulţimi poate fi etichetată prin vectorul său caracteristicv B ∈{0,1} n definit astfel: v B =(b 1 ,b 2 ,...,b n ) unde b i =0dacăa i ∉ B şi b i =1dacăa i ∈B. Pentru mulţimea vidă vectorul caracteristiceste v ∅ =(0,0,...,0) iar pentru mulţimea A este v A =(1,1,...,1).Pentru înţelegerea metodei şi pentru recunoaşterea tipurilor de problemeîn care ea poate fi folosită amalescâteva probleme din domeniul teorieijocurilor şi din domeniul combinatoricii mulţimilor, pe care le vom prezentacu rezolvări amănunţite.Probleme alese1. Un casier are la dispoziţie 10 plicuri în care trebuie să introducă osumă de1000 lei. Cum trebuie să distribuie banii în plicuri astfel ca dacăvine un angajat săîşi ridice salariul, casierul săîi poată dasumaexactăfărăsă mai scoată banii din plicuri?2. La curtea regelui Merlin urmează unmareospăţ. El a primit cadou1000 de sticle de vin, dar a aflat că una din sticle conţine o otravă foarteputernică. Având 10 condamnaţi la moarte, regele se hotărăşte să testezevinul pe aceşti condamnaţi, putând da fiecăruia câte o picătură devindinfiecare sticlă. Cum poate el identifica sticla otrăvită, dacă până laospăţ maisunt 10 ore şi otrava îşi face efectul în 10 ore?3. Fie A ∈M 2n (Z)o matrice cu proprietatea:(P) Pentru orice două linii L i , L j cu i ≠ j, sumaL i +L j conţine nelemente numere pare şi n elemente numere impare.Să searatecă pentru orice două coloaneC i ,C j cu i ≠ j, sumaC i +C jconţine n elemente numere pare şi n elemente numere impare.4. Jocul Nim (un joc chinezesc extrem de vechi) se joacă de douăpersoane care ridică depemasăunnumăr oarecare de pietre dintr-o singurăgrămadă dintretreigrămezi de pietre. Câştigătorul este jucătorul care ia depe masă ultima piatră. Dacă grămezile conţin 3, 5 şi 7 pietre, să se precizezecare jucător va câştiga (care ia primul sau al doilea) şi să se descrie strategiade câştig.5. Fie m, n numere naturale nenule, A omulţime cu n elemente şiA 1 ,A 2 ,...,A m submulţimi nevide şi distincte din A. Săsearatecă dacăm ≥ n +1 atunci există două submulţimi disjuncte I,J ⊂{1,2,...,n} astfelca ⋃ A i = ⋃ A j .i∈Ij∈J6. Fie m, n numere naturale nenule, A omulţime cu n elemente şiA 1 ,A 2 ,...,A m submulţimi nevide şi distincte din A. Săsearatecă dacăm ≥ n +2 atunci există două submulţimi disjuncte I,J ⊂{1,2,...,n} astfelca ⋂ A i = ⋂ A j .i∈Ij∈J


V. Pop, Metoda etichetării binare în probleme de combinatorică 1757. Fie m, n numere naturale nenule, A omulţime cu n elemente şiB 1 ,B 2 ,...,B m submulţimi proprii, nevide şi distincte, din A. Se ştie căpentru orice două elemente distincte din A există o singură mulţime B i carele conţine pe ambele. Să searatecăm≥n.8. Fie m, n numere naturale nenule, A o mulţime cu n elementeşi B 1 ,B 2 ,...,B m submulţimi nevide şi distincte din A. Se ştie că existăk ∈{1,...,n−1} astfel ca |B i ∩ B j | = k, oricarearfii, j = 1,n, i ≠ j.Săse arate că m ≤ n.9. Fie A ∈ M m,n (Z) omatricecum > n. Săsearatecă existăk ∈{1,2,...,m} şi k linii distincte L i1 ,L i2 ,...,L ik astfel ca suma L i1 ++L i2 + ...+L ik săaibătoatecelencomponente numere pare.10. Fie m, n numere naturale cu m > n > 1, A o mulţime cu nelemente şi A 1 ,A 2 ,...,A m submulţimi nevide, distincte ale lui A.Să searatecă există indici distincţi i 1 ,i 2 ,...,i k ∈{1,2,...,m} astfelca A i1 ∆A i2 ∆ ...∆A ik = ∅, unde am notat cu X∆Y =(X∪Y)\(X∩Y),diferenţa simetrică amulţimilor X şi Y .11. Fie m, n numere naturale nenule, A omulţime cu n elemente şiB 1 ,B 2 ,...,B n submulţimi nevide şi distincte din A astfel ca fiecare mulţimeB i , i = 1,n, să conţină unnumăr impar de elemente şi pentru orice i ≠ jmulţimea B i ∩ B j conţine un număr par de elemente. Să searatecăm≤n.Soluţii la problemele alese1. Mai întâi observăm că oricesumă cuprinsăîntre 1 şi 1000 lei (chiarpână la 1023 lei) poate fi reprezentatăîn scrierea binară folosind cel mult 10cifre:S = ε 1 · 1+ε 2·2+ε 3·2 2 +...+ε 10 · 2 9unde ε 1 ,ε 2 ,...,ε 10 ∈{0,1}. Dacă am introduce în cele 10 plicuri câte 1 leu, 2lei, 2 2 = 4 lei, 2 3 = 8 lei,..., 2 9 = 512 lei, atunci, pentru a da suma S, alegemplicurile pentru care ε i =1,i=1,10. Observăm că în acest mod am aveanevoie de 1023 lei. Corecţia o facem astfel: în primele 9 plicuri introducempe rând 1 leu, 2 lei, 2 2 lei,..., 2 8 = 256 lei, iar în ultimul plic restul baniloradică 489 lei.Dacă salariatul cere o sumă maimică decât 512 lei aceasta poate fidată folosind doar plicuri din primele 9 (orice număr mai mic ca 512 are înreprezentarea în baza doi cel mult 9 cifre). Dacă salariatul cere mai mult de511 lei atunci îi dăm mai întâi ultimul plic (cu 489 lei) şi apoi suma rămasă(mai mică ca 512 lei) poate fi acoperită folosind primele 9 plicuri.2. Etichetăm sticlele cu numerele 1, 2,...,1000 scrise în baza 2, decifiecare sticlă va avea un cod de 10 cifre din mulţimea {0, 1}, (c 1 ,c 2 ,...,c 10 )în loc de k = c 1 · 1+c 2·2+c 3·2 2 +...+c 10 · 2 9 .Primul condamnat bea din sticlele în care c 1 = 1 (toate sticlele cunumăr impar), al doilea din cele cu c 2 = 1, . . . , al zecelea din cele încare c 10 = 1 (fiecare bea cam din jumătate din sticle). Dacă după cele 10


176 Articoleore au murit condamnaţii P i1 ,P i2 ,...,P ik atunci sticla otrăvită arenumărul2 i1−1 +2 i2−1 +...+2 ik−1 .3. Asociem matricii A =[a ij ], matricea B =[b ij ]în care b ij =1dacăa ij este număr par şi b ij = −1 dacăa ij este număr impar (b ij =(−1) a ij).Observăm că matricea A are proprietatea (P) dacă şi numai dacă produsuloricăror două linii L ′ i şi L′ j din matricea B conţine n de 1 şi n de −1,adică2n∑k=1b ik b jk = 0. Deoarece2n∑k=1(b ik ) 2 =2n, oricare ar fi i = 1, 2n, rezultăcă A are proprietatea (P) dacă şi numai dacă B · B t =2n·I 2n .Evidentavemşi B t · B =2n·I 2n ,relaţie care reinterpretată dă aceleaşi condiţii asupracoloanelor matricei B, respectiv asupra coloanelor matricei A.Observaţie. Se poate pune problema: care sunt numerele naturale npentru care există A de dimensiune 2n cu proprietatea (P). Nu ştiu răspunsuldar cred că sunt numai numerele de forma n =2 k ,k∈N.4. Vom prezenta jocul Nim în cazul general. Se dau n grămezi de pietre.Doi jucători ridică alternativ orice număr de pietre dintr-o singură grămadă.Câştigă cel care ia ultima piatră.Vom preciza situaţiile în care câştigătorul este cel care începe jocul saual doilea jucător şi în fiecare caz vom prezenta strategia de câştig.– Dacă avem o singură grămadă, evident că primuljucător câştigă,luând dintr-o dată toate pietrele.– Dacă avemdouăgrămezi avem două cazuri esenţial diferite:a) dacăîn cele două grămezi sunt un număr egal de pietre va câştiga aldoilea jucător: după ridicarea unui număr de pietre de către primul jucător,rămân număr inegal de pietre în cele două grămezi. Al doilea joacă binedacăridică din cealaltă grămadă acelaşi număr de pietre câte a ridicat primul.Astfel primul jucător este în aceeaşi situaţie (număr egal de pietre în fiecaregrămadă). Continuând astfel jocul, la ultima mutare a primului jucător,acesta trebuie să termine una din grămezi după care al doilea ia toate pietreledin a doua grămadă.b) dacă în cele două grămezi sunt numere diferite de pietre, primuljucător câştigă folosind aceeaşi strategie (ridică dingrămada mai numeroasăatâtea pietre astfel ca să rămână număr egal de pietre şi îl pune pe al doileajucător în situaţia de pierdere).– În cazul în care numărul n este cel puţin 3, strategia câştigătoare pentruprimul sau al doilea jucător necesită la fiecare moment scrierea numerelorde pietre din fiecare grămadăîn baza 2.Să presupunem că numerele de pietre N 1 ,N 2 ,...,N n din cele n grămezise scriu în baza doi cu cel mult k cifre. Dacă N i = ε 1·2 k−1 +ε 2·2 k−2 +...+ε k·1,cu ε 1 ,ε 2 ,...,ε k ∈{0,1}, atunci etichetăm grămada respectivă cuk-uplul(ε 1 ,ε 2 ,...,ε k ) ∈{0,1} k . În {0, 1} k sau Z k 2 definim suma modulo 2 care se


V. Pop, Metoda etichetării binare în probleme de combinatorică 177face pe componente după regula0⊕0=1⊕1=0şi 0 ⊕ 1=1⊕0=1(numită uneori şi sumă nim).Arătăm că orice poziţie câştigătoare (pentru jucătorul care urmează lamutare) este orice poziţie în care suma nim N 1 ⊕N 2 ⊕...⊕N n ≠(0,0,...,0)în {0, 1} k şi vom descrie strategia de joc prin care se câştigă. Strategiacâştigătorului este de a lua atâtea pietre dintr-o anumită grămadă astfel casuma nim pe care o lasă pemasăsăfie(0,0,...,0). Într-o astfel de stare,celălalt jucător nu poate evita să-i lase primului tot o situaţie câştigătoare.Avem de demonstrat două lucruri:1) Dintr-o poziţie în care suma nim este (0, 0,...,0), prin orice mutarese ajunge la o stare în care suma nim este diferită de(0,0,...,0).2) Dintr-o poziţie în care suma nim este diferită de(0,0,...,0) putemgăsi o grămadă din care luăm un număr (bine gândit) de pietre ca să ajungemlaopoziţie cu suma nim (0, 0,...,0).Pentru 1) să observăm că dacă grupăm în fiecare grămadă pietrele conformscrierii în baza 2 (de exemplu dacă N i =13=2 3 +2 2 + 1 avem treigrupe: una cu o piatră, una cu 4 pietre şi una cu 8 pietre), atunci sumanim egală cu(0,0,...,0) semnifică faptulcăavemîn toate cele n grămezinumăr par de grupe de 1, număr par de grupe de 2,..., număr par de grupe de2 k−1 pietre. Luând pietre dintr-o singură grămadă desfiinţăm câte o singurăgrămadă de fiecare tip care intrăîn exprimarea numărului de pietre ridicate(de exemplu dacă ridicăm 6 = 2 + 2 2 pietre, desfiinţăm o grupă de2şi ogrupă de4rămânând aceste grupe în număr impar) şi rămân unele grupeimpare deci suma nim nenulă.Pentru 2) să notăm cu S =(ε 1 ,ε 2 ,...,ε k ) ≠(0,0,...,0) suma nim anumerelor N 1 ,N 2 ,...,N n :S =N 1 ⊕N 2 ⊕...⊕N nşi considerăm sumele nim:M 1 = N 1 ⊕ S, M 2 = N 2 ⊕ S,..., M n =N n ⊕S,din care alegem pe cea care rescrisă canumăr în baza 2 este cea mai mică.Dacă aceastaesteM i =N i ⊕Satunci M i


178 Articoledeci primul jucător câştigă.S ⊕ N 1 =(0,1,0) = 2, S ⊕N 2 =(1,0,0) = 4, S ⊕N 3 =(1,1,0) = 6şi avem 2 < 4 < 6. Vom lăsa în prima grupă doar două pietre şi astfel a douapoziţie este:2:=(0,1,0), 5:=(1,0,1), 7:=(1,1,1), cu S =(0,0,0).Să presupunem că al doilea jucător ia din ultima grupă 6 pietre şirămâne situaţia:2:=(0,1,0), 5:=(1,0,1), 1:=(0,0,1),cu S =(1,1,0) ≠(0,0,0).Avem:S ⊕ 2=(1,0,0) = 4, S ⊕5=(0,1,1) = 3, S ⊕1=(1,1,1) = 7şi 3 < 4 < 7. Mutarea bună estesălăsăm în a doua grămadă trei pietre, decirămâne situaţia:2:=(0,1,0) 3 := (0, 1, 1) 1 := (0, 0, 1),cu S =(0,0,0).Dacă, de exemplu, al doilea jucător ia din grămada a doua două pietrerămâne situaţia:2:=(0,1,0) 1 := (0, 0, 1) 1 := (0, 0, 1),cu S =(0,1,0) ≠(0,0,0).Avem:S ⊕ 2=(0,0,0) = 0, S ⊕1=(0,1,1) = 3, S ⊕1=(0,1,1) = 3şi mutarea bunăestesă eliminăm prima grămadă. Continuarea este evidentă.5. Fie A = {a 1 ,a 2 ,...,a n }şi pentru fiecare submulţime A i ⊂ A definimvectorul caracteristic v i =(x i1 ,x i2 ,...,x in ), i = 1,m, unde x ij =1dacăa j ∈A i şi x ij =0dacăa j ∉ A i .Observăm că:n∑ n∑n∑|A i | = x ik = x 2 ik şi că |A i ∩ A j | = x ik x jk .k=1k=1Deoarece m>n, vectorii v 1 ,v 2 ,...,v m sunt liniar dependenţi în R n ,deci există numerele reale α 1 ,α 2 ,...,α m astfel ca:m∑α i v i =0.i=1Notăm cu I mulţimea indicilor i pentru care α i > 0şi cu J mulţimeaindicilor J pentru care α j < 0şi scriem relaţia sub forma:∑α i v i = ∑ not(−α j )v j =(y 1 ,y 2 ,...,y n ).i∈I j∈Jk=1


V. Pop, Metoda etichetării binare în probleme de combinatorică 179Vom arăta că ⋃ A i = ⋃ A j .i∈Ij∈JFie a k în reuniunea din stânga. Dacă a k ∈ A i atunci x ik =1,α i x ik > 0şi din y k ≥ α i x ik rezultă y k ≠0. Ţinând cont de egalitatea:∑(−α j )v j =(y 1 ,...,y k ,...,y n )j∈Jrezultă că există unv j cu:(v j )x jk ≠0⇔x jk ≠0⇔a k ∈A jşi am arătat o incluziune (cealaltă este simetrică).Observaţie. Numărul m = n + 1 este cel mai mic număr de mulţimicu această proprietate după cum rezultă din exemplul:A 1 = {a 1 }, A 2 = {a 2 },...,A n ={a n } şi ⋃ A i ≠ ⋃ A j .6. Considerăm mulţimile B i = A i = A \ A i , i = 1,m. Cel puţin m − 1din ele sunt nevide şi cum m − 1 >n, conform problemei 5, rezultă că existăI, J disjuncte astfel ca:⋃B i = ⋃ B j ⇔ ⋃ B i = ⋃ B j ⇔ ⋂ A i = ⋂ A j .i∈I j∈J i∈I j∈JNumărul m = n + 2 este cel mai mic cu proprietatea din enunţ, dupăcum rezultă din exemplul:A 1 = {a 1 }, A 2 = {a 2 },...,A n ={a n }, A n+1 = {a 1 ,a 2 ,...,a n }.7. Fie A = {a 1 ,a 2 ,...,a n } şi notăm cu L i ∈{0,1} n vectorul caracteristical mulţimii B i (L i = (α 1 ,α 2 ,...,α n ); α j = 1 dacă a j ∈ B i şiα j =0dacăa j ∉ B i ). Notăm cu M ∈M m,n ({0, 1}) matricea cu liniileL 1 ,L 2 ,...,L m .Condiţia din problemă spune că pentru orice i, j ∈ {1,2,...,n} cui ≠ j, există o linie L k astfel ca pe poziţiile i şi j să avem1. Faptulcăaceastă linie este unicăînseamnă că produsul (scalar) al coloanelor C i şi C jeste 1. Astfel se sugerează ideea de a considera matricea M t · M ∈M n (N)pentru care M t · M = P =[p ij ] i,j=1,nunde p ij = C i C j (produsul coloanelorC i şi C j ). Conform observaţiei făcute p ij = 1, oricare ar fi i ≠ j şi p ii ≥ 1,i = 1,n. Arătăm că p ii ≥ 2, i = 1,n. Dacă am avea un p ii = 1, arînsemna că pe coloana C i avem un singur element nenul, deci un anumitelement a j se găseşte într-o singură mulţime B k ; atunci, din condiţia datăpentru orice alt element a j1 , existăomulţime care conţine pe a j şi pe a j1 , deciaceastă mulţime este B k ;în concluzie B k ar conţine toate elementele din A,încontradicţie cu faptul că B k este submulţime proprie. Astfel M t · M = D + Junde D este o matrice diagonală cu elementele diagonalei strict pozitive şi J


180 Articoleeste matricea cu toate elementele egale cu 1. Se arată uşor că determinantulmatricei M t · M este strict pozitiv (mai mult matricea este pozitiv definită:n∑n∑n∑n∑p ij x i x j = d i x 2 i + x i x j = d i x 2 i +(x 1+...+x n ) 2 ,i,j=1i=1i,j=1unde d i > 0, i = 1,n).În concluzie, rangM t · M = n, deci rangM ≥ n şi deoarece rangM ≤ mrezultă m ≥ n.8. Considerăm matricea M ∈ M m,n ({0, 1}) care are pe liniileL 1 ,L 2 ,...,L n vectorii caracteristici ai mulţimilor B 1 ,B 2 ,...,B m şi notăm cuG matricea pătratică G = M·M t ∈M m (R). Elementele sale sunt g ij = L i·L j(produsul (scalar) al liniilor L i şi L j )şi se observăcăg ij = |B i ∩B j | = k, undei ≠ j şi evident g ii = |B i |≥k,i=1,n. Dacă notăm g ii = k + b i , i = 1,n,arătăm că unsingurb i poate fi 0, iar ceilalţi sunt strict pozitivi: dacă prinabsurd b 1 = b 2 = 0 atunci B 1 şi B 2 au fiecare câte k elemente şi, doareceB 1 ≠ B 2 , rezultă căB 1 ∩B 2 are mai puţin de k elemente (contradicţie).Determinantul matricei G este:∣ k + b 1 k k ... k ∣∣∣∣∣∣∣∣∣k k +b 2 k ... k∆ m =k k k +b 3 ... k... ... ... ... ...∣ k k k ... k+b ncare verifică relaţia de recurenţă ∆ m =k·b 1·b 2·...·b n−1 +b n ∆ n−1 şi, înfinal:⎛⎞m∏∆ m = b k + k ⎝ ∏ b k + ∏ b k + ...+ ∏ b k⎠>0,k=1k≠1 k≠2k≠ndeci matricea G = M ·M t are rangul m şi cum rangM ≥ rangG şi rangM ≤ nobţinem n ≥ m.9. Vom trece de la numere întregi la clase de resturi modulo 2. Numerelepare se înlocuiesc cu ̂0 şi cele impare cu ̂1 ca elemente ale corpului Z 2 .Problema cere să arătăm că există liniile L i1 ,L i2 ,...,L ik cu suma:̂L i1 + ̂L i2 + ...+ ̂L ik =(̂0,̂0,...,̂0).Liniile pot fi privite ca elemente ale spaţiului vectorial Z n 2 care estespaţiu vectorial de dimensiune n peste corpul Z 2 . Deoarece m>n, liniilesunt liniar dependente, deci există scalariiα 1 ,α 2 ,...,α m ∈{̂0,̂1}astfel ca:α 1 ̂L1 + α 2 ̂L2 + ...+α m̂Lm =(̂0,̂0,...,̂0).Dacă în relaţia de mai sus nu scriem coeficienţii ̂0 şi rămân doarα i1 = α i2 = ...=α ik =̂1obţinem:̂L i1 + ̂L i2 + ...+ ̂L ik =(̂0,̂0,...,̂0).i=1


V. Pop, Metoda etichetării binare în probleme de combinatorică 18110. Fie A = {a 1 ,a 2 ,...,a n } şi, pentru fiecare submulţime A i , definimvectorul său caracteristic v i =(x i1 ,x i2 ,...,x in ), unde x ij =1dacăa j ∈A i şix ij =0dacăa j ∉ A i , i = 1,m, j = 1,n.Observăm că vectorul caracteristical mulţimii A i1 ∆A i2 este suma modulo 2 a vectorilor V i1 şi V i2 .Considerăm numerele 0 şi 1 ca elemente ale corpului Z 2 , renotate ̂0 şi ̂1,iar vectorii v i ca elemente ale spaţiului vectorial Z n 2 ,careestespaţiu vectorialde dimensiune n peste corpul Z 2 . Problema astfel reformulată cere să arătămcă există vectorii caracteristici ̂v i1 , ̂v i2 ,...,̂v ik cu suma zero:̂v i1 + ̂v i2 + ...+̂v ik =(̂0,̂0,...,̂0).Având m > n vectori (caracteristici) într-un spaţiu vectorial de dimensiunen (Z n 2 ) rezultă că ei sunt liniar dependenţi. Există deci scalariiα 1 ,α 2 ,...,α m ∈{̂0,̂1}astfel ca:α 1̂v 1 + α 2̂v 2 + ...+α m̂v m =(̂0,̂0,...,̂0).Dacă în relaţia de mai sus nu mai scriem coeficienţii ̂0 şi rămân doarcoeficienţii egali cu ̂1, α i1 = α i2 = ...=α ik =̂1obţinem:̂v i1 + ̂v i2 + ...+̂v ik =(̂0,̂0,...,̂0),adică:A i1 ∆A i2 ∆ ...∆A ik =∅.11. Notăm cu L i ∈ {0,1} n vectorul caracteristic al mulţimii B i ,i = 1,m şi notăm cu M ∈M m,n ({0, 1}) matricea cu liniile L 1 ,L 2 ,...,L m .Matricea pătratică G = M · M t ∈M n (R) are pe diagonală numereimpareşiîn afara diagonalei numere pare (g ij = |B i ∩ B j |). Trecând la clasa modulo 2,în Z 2 matricea Ĝ este matricea unitate Îm cu determinantul nenul. G fiindmatricea Gram a vectorilor ̂L 1 , ̂L 2 ,...,̂L m rezultă că vectorii ̂L 1 , ̂L 2 ,...,̂L msunt liniar independenţi în Z n 2 şi atunci m ≤ n.Bibliografie[1] T. Andreescu, G. Dospinescu, Problems from the Bock, XYZ Press, 2008.[2] J. D. Beasley, The Mathematics of Games, Oxford Univ. Press, 1989.[3] M. Eigen, R. Winkler, Law of the Games, Princeton Univ. Press, 1981.[4] M. Kaitchik, Mathematical Recreations, W. W. Norton, 1942.[5] L. Babai, P. Frankl, Linear Algebra Methods in Combinatorics, Dep.Comput.Sci.Univ. Chicago, 1992.[6] B. Lindstrom, Another theorem of families of sets, Ars Combinatorica, 35(1993),123-124.[7] O. Pikhurko, Algebraic Methods in Combinatorics.


182 ArticoleAsupra teoremei lui Beatty, şirurilor lui Wythoff şicuvântului lui FibonacciAdrian Reisner 1)Abstract. Beatty’s theorem establishes an equivalence relation so thatthe sets {[nα],n ∈ N ∗ } and {[nβ],n ∈ N ∗ } form a disjoint union of N ∗ ,for α, β ∈ R − Q. ThecaseofWythoff ’s pairs is a specific case of Beaty’stheorem with α = ϕ = 1+√ 5(the golden ratio) and allows us to define2the golden semigroup (or Wythoff ’s semigroup) and on the other hand theFibonacci word. The latter, the sturmian characteristic word of slope 1 ρ , 2is also obtained as the fixed point of the morphism σ(0) = 01, σ(1) = 0 inthe free monoid of base A = {0, 1}. Finally, the Fibonacci word allows usto determine a winning strategy for Rufus Issac’s Nim game (or Wythoff ’sgame).Keywords: Beatty theorem, Zeckendorf representation, Ecuaţii Sturm -Liouville, Combinatorics on words.MSC : 11A67, 68R15.I. Partiţiile mulţimii N ∗Fiind dat numărul real α notăm cu E(α) mulţimea:E(α) ={[nα],n ∈N ∗ },unde [α] este partea întreagă a numărului real α. Avem atunci teoremaurmătoare:Teorema 1. Fiind date trei numere reale α 1


A. Reisner, Teorema Beatty, şiruri Wythoff, cuvântul Fibonacci 183şi deci:[tα 1 ]=m−1;[(t +1)α 1 ]=m+1⇒(t+1)α 1 =m+1+ϕ;tα 1 + α 1 = tα 1 +1+ε=m+1+ϕ⇒ε−ϕ=m−tα 1 > 0, ε>ϕ;[(t +2)α 1 ]=m+2⇒(t+2)α 1 =m+2+ϕ+ε, ϕ + ε


184 ArticoleNe propunem aici să demonstrăm teorema lui Beatty utilizând proprietăţilecuvântului caracteristic.Considerând cuvântul caracteristic C(θ) de pantă θ = 1 avem, cuanotaţile precedente:Lema 5. Următoarele două aserţiuni sunt echivalente:i) c k = 1, ii) k ∈ E(a)i.e. funcţia F [E(a)] : N ∗ →{0,1},n↦→ c n , este funcţia caracteristică asubmulţimii E(a) ⊂ N ∗ .Demonstraţie. Să demonstrăm că pentru orice k ∈ N ∗ avem:card{E(a) ∩ [1,k]} =[(k+1)θ].Într-adevăr:card{E(a) ∩ [1,k]} =card{n∈N ∗ |[na]


A. Reisner, Teorema Beatty, şiruri Wythoff, cuvântul Fibonacci 185Deducem atunci echivalenţa celor trei aserţiuni:a) N ∗ ( este reuniune ( ) disjunctă ( aluiE(a)şi E(b);1 1a ′ ) C = C = C 1 −a)1 )i.e.b bF[E(a)] = F[E(b)] = 1 −F[E(b)];b) a/∈Q,b/∈Qşi 1 a + 1 b =1Astfel teorema lui Beatty este demonstrată.Caz particular. Pentru a = ϕ = 1+√ 5[b=ϕ 2 ] teorema lui Beatty2arată că cele două submulţimi :E 0 = E(ϕ) ={[nϕ], n∈N ∗ }şi E 1 = E(ϕ 2 )={[nϕ 2 ],n∈N ∗ }formează opartiţie a mulţimii N ∗ . Aceste două submulţimi se numesc şirurilelui Wythoff :E 0 = {1, 3, 4, 6, 8, 9, 11, 12, 14, 16, 17, 19, 21, 22, 24, 25, 27, 29, 30, 32, 33,...};E 1 ={2,5,7,10, 13, 15, 18, 20, 23, 26, 28, 31, 34, 36, 39, 41, 44, 47, 49, 52,...}.Cuvântul caracteristic de pantă 1 ( ) ( )1 1ϕ 2 fiind C ϕ 2 = c 1 c 2 ...c n ...,Cϕ 2 == 0100101001001 ..., funcţia n ↦→ c n este funcţia caracteristică a submulţimiiE 1 = E ( ϕ 2) ⊂ N ∗ .Ne propunem să demonstrăm două proprietăţi interesante ale şirurilorlui Wythoff .III. Şirurile lui WythoffA. Şirurile lui Wythoff şi sistemul de numerotaţie al luiFibonacciŞirul lui Fibonacci este definit prin: F 0 =1,F 1 =2şi F n+1 = F n +F n+1 ,n>0.Avem teorema:Teorema 7 (Zeckendorf). Orice număr n ∈ N se scrie în mod unicsub forma n = ∑ n i F i , unde n i ∈{0,1}şi n i n i+1 =0,pentru orice întregi≥0i ≥ 0. Notaţia lui Fibonacci este Fib(n)=n k n k−1 ...n 0 .n∑n∑F 2p +1=F 2n+1 ; F 2p+1 +1=F 2n+2 .p=0Sistemul de numerotaţie al lui Fibonacci. -Sistemul de numerotaţie al lui Fibonacci este un caz particular al sistemuluide numerotaţie al lui Ostrowski (vezi [1]) sistem care are pentru bazănumitorii convergenţilor în dezvoltarea unui număr real în fracţii continue.p=1(∗)


A. Reisner, Teorema Beatty, şiruri Wythoff, cuvântul Fibonacci 187admite ca rădăcini ϕ = 1+√ 5şi θ = 1 − √ 522proprietatea şirurilor recurente există k, k ′ reali astfel încât, pentru oricen ∈ N, F n = kϕ n + k ′ θ n . Considerând cazurile n =0şi n =1,obţinemk = 5+3√ 510şi k ′ = 5 − 3√ 510F n = 5+3√ 510şi, în final:= − 1 . Ţinând seama deϕϕ n − 5 − 3√ 5θ n = 1 (√ ϕ n+2 − θ n+2) .10 5a) În continuare, deorece ϕθ = −1, avem:F n ϕ = 1 √5(ϕ n+3 + θ n+1) = 1 √5(ϕ n+3 − θ n+3) + 1 √5(θ n+3 + θ n+1) == F n+1 + θ n+1 ( θ 2 +1√5)=F n+1 − θ n+2 .Fiind dat n = ∑ i≥0n i F i ,notăm cu π(n) =inf{i|n i =1}.Ţinând seamade cele precedente avem nϕ = ∑ n i F i+1 + r(n) unde r(n) =− ∑ i θi≥0i≥0n i+2 .Cu n i n i+1 =0obţinem: în expresia lui r(n) singurele puteri care intervinsunt puterile lui θ având aceeaşi paritate cu π(n).– Dacă π(n) estepar,Fib(n) se termină cuunnumăr par de 0. Pe dealtă parte:0 >r(n)=− ∑ ()n i θ i+2 > − θ 2 ...+θ 2k ... =− 1 ϕi≥0iparşi obţinem [nϕ]+1=...+F π(n)+1 deci Fib([nϕ] + 1) se termină printr-unnumăr impar de 0 de unde Fib([nϕ]) se termină prin 01 – vezi formulele (∗)din teorema 7 – şi, în final, Fib([nϕ] − 1) se termină cu 00.– Dacă π(n) estimpar,Fib(n)seterminăcuunnumăr impar de 0. Înacest caz avem:0


A. Reisner, Teorema Beatty, şiruri Wythoff, cuvântul Fibonacci 189b) ⇒ a): Presupunând că 0≤q−pϕ < 1 avem: 0 ≤ qϕ− p(ϕ+1)


190 ArticoleÎn particular, oricare ar fi w m ∈Wşi oricare ar fi W n ∈B:W n (w m )∈B(a 2-a coloană a produsului W n W m )[ ( )]q − p pW n W m =p qşi, în final, se obţine – vezi observaţia precedentă – teorema 10.Mulţimea W înzestrată deoperaţia internă astfel definită:w n · w m = w n·m , unde n · m = nm + a n a m ,este un semigrup comutativ izomorf semigrupului (B, ×).IV. Cuvântul lui FibonacciConsiderăm aici alfabetul A = {0, 1}. Pentru noţiunile introduse aicivezi [4] capitolul 8 şi [5] capitolele I, II.Teorema 12. (A N ,d), unde d este definit prin d(u, u ′ )=0dacă u = u ′şi d(u, u ′ )=exp(−inf {n ∈ N,u n ≠u ′ n}) dacă u ≠ u ′ , este un spaţiu metriccompact, deci complet.Demonstraţie. Să demonstrăm că d este o distanţă ultrametrică i.e.o metrică verificând d(x, z) ≤ sup(d(x, y),d(x, z)) pentru orice x, y, z ∈A ∗ .d(x, y) =0dacăşi numai dacă x = y; d este simetrică. Să demonstrăminegalitatea ultrametrică . Presupunem cuvântul y diferit de cuvântul xpentru n ≥ i 0 şi de cuvântul z pentru n ≥ j 0 . Distingem două cazuri:i 0 ≠j 0 ;fiei 0


A. Reisner, Teorema Beatty, şiruri Wythoff, cuvântul Fibonacci 191Teorema 15. Acest morfism σ admite un unic punct fix peste A N .Acest punct fix este cuvântul λ = λ 0 λ 1 ...λ n ... definit în propoziţia 13.Acest cuvânt se numeşte cuvântul lui Fibonacci notat Fib.Demonstraţie. Fie u şi u ′ două cuvinte distincte având pentru prefixcuvântul u 0 ...u n−1 (n∈N).Dacă n = 0 atunci a este prima literă a imaginilor σ(u) şi σ(u ′ ).Dacă n>0 atunci imaginile σ(u) şi σ(u ′ )auîn comun cel puţin n +1litere căci aceste două imagini admit pentru prefix de lungime n +1 cuvântulσ(u 0 )σ(u 1 ) ...σ(u n−1 )0. Avem deci d(σ(u),σ(u ′ )) ≤ e −1 d(u, u ′ )


192 ArticoleProprietatea este verificată pentru n =1,2. Fie n ≥ 3. Dacă n aparţineşirului lui Fibonacci atunci n = F k unde k ≥ 2şi Φ k 0esteprefixalcuvântuluiΦ k Φ k−1 , deci prefix al cuvântului Fib; deducem u n =0. (|Φ k |=F k şi 0 esteprefixul lui Φ k−1 .)n= ∑n i F i fiind forma lui n în baza lui Fibonacci avem n = F k + mi=k...0unde 0


A. Reisner, Teorema Beatty, şiruri Wythoff, cuvântul Fibonacci 193forma y ′′ +Ψ(x)y = 0 unde Ψ(x) esteofuncţie continuă de perioadă l (înparticular o funcţie constantă)). Această funcţie y =sin(πα ′−1 t)estenulăpentru πα ′−1 t = kπ, unde k ∈ N, deci t k = kα ′ .Fie θ(n) numărul rădăcinilor funcţiei y = sin(πα ′−1 t)în intervaluldeschis (n, n + l) pentru orice n ∈ N. Cu notaţiile precedente avemθ(n) =µ(n, α ′ )şi corolarul 18 conduce atunci la teorema următoare:Teorema 19. Fiind dat cuvântul caracteristic C(α) de pantă α considerămecuaţia diferenţială omogenă y ′′ + π 2 α ′−2 y =0unde α ′ =α1 − α . Osoluţie fiind funcţia y =sin(πα ′−1 t), cuvântul C(α) este de forma C(α) =0 θ(0) 10 θ(1) 10 θ(2) 1 ...10 θ(n) 1 ... unde θ(n) este numărul rădăcinilor acesteifuncţii în intervalul deschis (n, n + l).Exemplu. Pentru α = −2, deci α ′ = −1, se obţine cuvântul luiFibonacci C(α) =Fib – vezi observaţia precedentă. În acest caz ecuaţiadiferenţială y ′′ +ω 2 y = 0 unde ω = πϕ admite ca soluţie funcţia y =sin(πϕt).Rădăcinile acestei funcţii sunt t k = kϕ −1 pentru k ∈ N. Tabloul următor indicărădăcinile t k = kϕ −1 ale funcţiei y =sin(πϕt) pentru 0 ≤ k ≤ 21:t 0 =0,ρ −1 =0,618033989 ...; 2ρ −1 =1,236067977 ...;3ρ −1 =1,854101966 ...; 4ρ −1 =2,472135955 ...;5ρ −1 =3,090169944 ...; 6ρ −1 =3,708203932 ...;7ρ −1 =4,326237921 ...; 8ρ −1 =4,944271910 ...;9ρ −1 =5,562305899 ...; 10ρ −1 =6,180339887 ...;11ρ −1 =6,798373876 ...; 12ρ −1 =7,416407865 ...;13ρ −1 =8,034441854 ...; 14ρ −1 =8,652475842 ...;15ρ −1 =9,270509831 ...; 16ρ −1 =9,888543820 ...;17ρ −1 =10,50657781 ...; 18ρ −1 =11,12461180 ...;19ρ −1 =11,74264579 ...; 20ρ −1 =12,36067977 ...;21ρ −1 =12,97871376 ....Valorile lui θ(n) pentru 0 ≤ n ≤ 12 sunt deci :θ(0) = 1, θ(1) = 2, θ(2) = 1, θ(3) = 2, θ(4) = 2, θ(5) = 1, θ(6) = 2,θ(7) = 1,θ(8) = 2,θ(9) = 2,θ(10) = 1,θ(11) = 2,θ(12) = 2 etc.şi cuvântul lui Fibonacci este de forma următoare:c(α) =Fib =0 θ(0) 10 θ(1) 10 θ(2) 10 θ(3) ...10 θ(n) 10 θ(n+1) ...== 0100101001001010010100100101001001 ....Cuvintele caracteristice C(θ) alcăror studiu a început în anii 1875de către Christoffel şi Markoff (1882) se numesc cuvintele caracteristicesturmiene. Ele sunt un caz particular al cuvintelor sturmiene s(θ, β) ==s 1 s 2 ...s n ... unde θ ∈ (0, 1) − Q, β ∈ R şi s n =[θ(n+1)+β]−[θn + β].Cuvintele sturmiene au fost introduse de către Morse şi Hedlund în anii1940 ([6]) şi sunt studiate astăzi din punct de vedere combinatoric, algebric,geometric.


194 ArticoleCuvintele sturmiene sau generalizarea lor – cuvintele episturmiene ([3])– intervin într-o multitudine de domenii: teoria numerelor (aproximaţii diofantiene),geometria discretă (drepte discrete), sisteme dinamice, fizica teoretică(cristalografie), teoria imaginilor pe ordinator (Digital straightness),biologia moleculară (studiu acidului deoxiribonucleic i.e. grupului ADN: A,C, G şi T), teoria muzicală ([7]) etc.Teorema 19 se poate generaliza pentru un cuvânt sturmian oarecare ([6]pagina 40) înlocuind ecuaţia din teorema 19 cu ecuaţia diferenţială omogenăde gradul 2:y ′′ +Ψ(x)y=0,(Sturm - Liouville)unde Ψ(x) esteofuncţie continuă de perioadă l (de aici termenul ,,cuvintesturmiene“).Complemente1) Şirurile lui Wythoff deci cuvântul lui Fibonacci, se pot regăsi utilizândteoria grafurilor; mai precis determinând nucleul grafului lui Wythoff– vezi jocul lui Rufus Isaacs – i.e.:Graful G = {V,A} următor (deplasarea reginei la jocul de şah):Mulţimea vârfurilor V = {(i, j) ∈ N 2 };Mulţimea arcurilor orientate A :Γ(i, j), mulţimea succesorilor lui (i, j),este definită prin Γ(i, j) = { (p, q) ∈ N 2 | fie p = i şi q


M. Olteanu, Asupra rafinării unor inegalităţi în tetraedru 195[4] Z. Kasa, Combinatorică cu aplicai¸i, Capitolul 8, paginile 111 - 168,http://www.cs.ubbcluj.ro/∼kasa/combinatorica.html[5] ∗∗∗Algebraic Combinatorics on words ch. 1 şi 2 Lothaire 2001 Cambridge UniversityPress 2002 Lothaire’s page ( Jean Berstel )[6] K. A. Hedlund, Symbolic dynamics II : Sturmian sequences, Marsten Morse, AmericanJournal Math. 62 ( 1940 ) pages 1 - 42.[7] T. Noll, Sturmian sequences and morphisms : A Music - Theoretical application,Mathématiques et Musique, Société Mathématique de France 2008 - Journée annuellehttp://user.cs.tu-berlin.de/∼noll/SMF Noll.pdfAsupra rafinării unor inegalităţi geometrice în tetraedruMarius Olteanu 1)Abstract. In this paper we improve some geometric inequalities recordingimportant lines in the tetrahedron.Keywords: Durrande inequality, orthocentric tetrahedron.MSC : 26D15Prezentul articol are drept scop prezentarea într-un cadru unitar, a unornoi rafinări ale câtorva inegalităţi de bază din geometria tetraedrului. Pentruînceput se vor stabili noi rezultate valabile într-un tetraedru oarecare, dupăcare vor fi evidenţiate, pentru clasa tetraedrelor ortocentrice şi echifaciale noiîntăriri ale rafinărilor inegalităţii Euler-Durrande (R ≥ 3r) stabilite pentrucazul general al tetraedrelor oarecare şi menţionate în [7] pag. 471 – 478, [8]pag. 625 – 630, [9] pag. 200 – 208, [10] pag. 98 – 108.Vom utiliza următoarele notaţii referitoare la elementele unui tetraedruoarecare [ABCD]: V −volumul său, S A −aria feţei (BCD) (analog S B , S C ,S D ), S = S A + S B + S C + S D , a = BC, b = AC, c = AB, l = AD, m = BD,n = CD, r a −raza sferei exînscrise de speţa întâi care este tangentă feţei(BCD) (analog r b , r c , r d ), h a , m a −lungimea înălţimii, respectiv a medianeitetraedrului ce conţine vârful A (analog h b , h c , h d şi m b , m c , m d ), r A , R A −raza cercului înscris respectiv circumscris triunghiului (feţei) BCD (analogr B , r C , r D şi R B , R C , R D ), d 1 , d 2 , d 3 − lungimile perpendicularelor comunecorespunzătare celor trei perechi de muchii opuse, b 1 , b 2 , b 3 − lungimile celortrei bimediane, r, R− razele sferei înscrise, respectiv circumscrise tetraedrului,I−centrul sferei înscrise, O− centrul sferei circumscrise, G−centrulde greutate al tetraedrului, H−ortocentrul tetraedrului ortocentric [ABCD],Ω−centrul sferei lui Euler asociată tetraedrului, iar Γ− simetricul punctuluiΩfaţă de centrul de greutate G.Lemă. Fie x, y, z, t ∈ R+. ∗ Atunci au loc inegalităţile:3a) x + y + z +1x + 1 y + 1 ≥ 4 3√ xyz;z1) S. C. Hidroconstrucţia S.A. Bucureşti, Sucursala ,,Olt-Superior“ din Râmnicu-Vâlcea


196 Articole4b) x + y + z + t +1x + 1 y + 1 z + 1 ≥ 5 4√ xyzt.tDemonstraţie. a) Fie m a ≥ m g ≥ m h mediile aritmetică, geometricărespectiv armonică a celor trei numere x, y, z > 0. Vom arăta că m 2 a · m h ≥≥ m 3 g. Într-adevăr:( ) x + y + z 23xyz·3 xy + yz + zx ≥ xyz ⇔ (x + y + z)2 ≥ 3(xy + yz + zx) ⇔⇔ (x − y) 2 +(y−z) 2 +(z−x) 2 ≥0,evident. Atunci:3x + y + z +1x + 1 y + 1 =3m a +m h ≥4 4√ m 3 a·m h=4 4√ m a (m 2 a·m h)≥z√≥4 4 m a·m 3 g ≥4√m 4 4 g =4 3√ xyz.b) Procedăm ca la punctul a), unde notaţiile (valabile pentru patrunumere strict pozitive) au aceleaşi semnificaţii.Vom arăta că m 3 a · m h ≥ m 4 g. Într-adevăr:( ) x + y + z + t 3·44xyztxyz + xyt + xzt + yzt ≥ xyzt ⇔⇔ (x + y + z + t) 3 ≥ 16(xyz + xyt + xzt + yzt),inegalitate adevărată conform rezultatului (212), de la pag. 84 din [11].Atunci:4x + y + z + t +1x + 1 y + 1 z + 1 =4m a +m h ≥5 5√ m 4 a·m h=t=5 5√ √m a (m 3 a·m h)≥5 5 m a·m 4 g√m ≥5 5 5 g =5 4√ xyzt.Propoziţia 1. În orice tetraedru [ABCD] au loc inegalităţile:a) h a + h b + h c + h d +4r≥5 4√ h a h b h c h d ≥5 5√ 4h a h b h c h d r≥20r;b) r a + r b + r c + r d +2r≥5 4√ r a r b r c r d ≥5 5√ 2r a r b r c r d r≥10r;c) 5 4r ≥ 1 + 1 + 1 + 1 45+≥h a h b h c h d h a + h b + h c + h 4√ ;d ha h b h c h dd) 5 2r ≥ 1 + 1 + 1 + 1 45+≥ √ .r a r b r c r d r a + r b + r c + r4 d ra r b r c r dDemonstraţie. a) Înlocuim la punctul b) al lemei pe x = h a, y = h b ,z = h c , t = h d şi ţinem seama de relaţia1 + 1 + 1 + 1 = 1 h a h b h c h d r . A douainegalitate este echivalentă cuh a h b h c h d ≥256r 4 , inegalitate clasică, valabilă


M. Olteanu, Asupra rafinării unor inegalităţi în tetraedru 197în orice tetraedru. Analog, cea de a treia inegalitate este echivalentă totcuh a h b h c h d ≥256r 4 .b) Se procedează ca la punctul a), unde se mai ţine seama de relaţiile1+ 1 + 1 + 1 = 2 r a r b r c r d r şi r ar b r c r d ≥ 16r 4 , valabile în orice tetraedru [ABCD].c) Avem h a + h b + h c + h d ≥ 16r; rezultă:1+ 1 + 1 + 1 4+≤ 1 h a h b h c h d h a + h b + h c + h d r + 1 4r = 5 4r ;a doua inegalitate rezultă imediat prin aplicarea lemei b), unde se considerăx = 1 , y = 1 , z = 1 , t = 1 .h a h b h c h dd) Deoarece r a + r b + r c + r d ≥ 8r, rezultă:1+ 1 + 1 + 1 4+≤ 2 r a r b r c r d r a + r b + r c + r d r + 1 2r = 5 2r ;tot prin aplicarea lemei b), în care se ia x = 1 r a, y = 1 r b, z = 1 r c, t = 1 r d,rezultă imediat cea de a doua inegalitate.Observaţii. 1. Toate cele patru puncte ale propoziţiei 1 rafineazăcunoscutele inegalităţi din geometria tetraedrului:⎧⎪⎨⎪⎩h a + h b + h c + h d ≥ 16r,r a + r b + r c + r d ≥ 8rh a h b h c h d ≥ 256r 4 ,r a r b r c r d ≥ 16r 4 .Menţionăm că rafinări ale inegalităţilor (∗) au fost recent abordate şiîn [2] (pag. 29-37).2. Deoarece m a ≥ h a (şi analoagele), m a + m b + m c + m d ≤ 16R şi√4 ha h b h c h d ≤ 4√ m a m b m c m d ≤ 1 4 (m a + m b + m c + m d ) ≤ 4R 3 ,3(∗)ţinând seama de inegalităţile a) şi c) ale propoziţiei 1, precum şi de relaţiile(23), (24), (25), (26) de la pag. 28 din [2], rezultă căaulocurmătoarelerafinări ale inegalităţii Euler-Durrande (R ≥ 3r) într-un tetraedru oarecare[ABCD]: ( ) 4Ra) 43 + r ≥ m a + m b + m c + m d +4r≥h a +h b +h c +h d +4r≥≥10 5 √2 · r ·≥5 4√ h a h b h c h d ≥5 5√ 4h a h b h c h d r≥(h a h b h c h d ) 2(h a + h b )(h b +h c )(h c +h d )(h d +h a ) ≥20r.


198 Articoleb) 5 4r ≥ 1 h a+ 1 h b+ 1 h c+ 1 h d+4h a + h b + h c + h d≥54√ha h b h c h d≥5≥ √ ≥ 154 ma m b m c m d 4R .( ) 4Rc) 43 + r ≥ m a + m b + m c + m d +4r≥h a +h b +h c +h d +4r≥≥5 4√ h a h b h c h d ≥5 5√ 4h a h b h c h d r≥≥10· √(h 5a h b h c h d ) 22 · r ·[(h a + h b )(h a +h c )(h a +h d )(h b +h c )(h b +h d )(h c +h d )] 2 3d) 16R3≥ m a + m b + m c + m d ≥ h a + h b + h c + h d ≥≥ 20r.√ha h b h c h d≥ 8 √(ha + h b )(h b +h c )(h c +h d )(h d +h a ) ≥16r.4e) 16R3≥ m a + m b + m c + m d ≥ h a + h b + h c + h d ≥√ha h b h c h d≥ 8 √(ha + h b )(h a +h c )(h a +h d )(h b +h c )(h b +h d )(h c +h d ) ≥16r.63. Având în vedere inegalităţile c) şi d) ale propoziţiei 2, pag. 31 din[2] mai putem încă rafina inegalitatea b) a propoziţiei 1 din acest articol,obţinând următoarele şiruri de inegalităţi:i) r a + r b + r c + r d +2r≥5 4√ r a r b r c r d ≥5 5√ 2r a r b r c r d·r≥(h≥5√2r·5 a h b h c h d ) 2(h a +h b )(h b +h c )(h c +h d )(h d +h a ) ≥10r.ii) r a + r b + r c + r d +2r≥5 4√ r a r b r c r d ≥5 5√ 2r a r b r c r d·r≥≥5 5 √ √√√2r·(h a h b h c h d ) 2[(h a + h b )(h a +h c )(h a +h d )(h b +h c )(h b +h d )(h c +h d )] 2 3≥ 10r.şi:4. Deoarece:649 R2 ≥ m 2 a + m 2 b + m2 c + m 2 d ≥ h2 a + h 2 b + h2 c + h 2 d ,h 2 a + h 2 b + h2 c + h 2 d +16r2 ≥ (h a+h b +h c +h d +4r) 2r 2 a + r2 b + r c + r 2 d +4r2 ≥ 1 5 (r a+r b +r c +r d +2r) 2 ,5


M. Olteanu, Asupra rafinării unor inegalităţi în tetraedru 199ca şi consecinţe a celor prezentate până acum,seobţin noi rafinări ale inegalităţiiEuler-Durrande,( )valabile în orice tetraedru [ABCD]:4R2a) 169 + r2 ≥ m 2 a + m2 b + m2 c + m2 d +16r2 ≥≥h 2 a +h2 b +h2 c +h2 d +16r2 ≥ 1 5 (h a+h b +h c +h d +4r) 2 ≥≥5 √ h a h b h c h d ≥5√16r 5 2 h 2 ah 2 b h2 ch 2 d ≥√≥ 20 5 4r 2 (h a h b h c h d ) 4(h a + h b ) 2 (h b + h c ) 2 (h c + h d ) 2 (h d + h a ) 2 ≥ 80r2 .( ) 4R2b) 169 + r2 ≥ m 2 a + m2 b + m2 c + m2 d +16r2 ≥≥h 2 a +h2 b +h2 c +h2 d +16r2 ≥ 1 5 (h a+h b +h c +h d +4r) 2 ≥≥5 √ h a h b h c h d ≥5 5 √16r 2 · h 2 ah 2 b h2 ch 2 d ≥√ √√√4r(h≥ 20 5 2 a h b h c h d ) 4[(h a + h b )(h a +h c )(h a +h d )(h b +h c )(h b +h d )(h c +h d )] 4 3≥ 80r 2 .c) 64 9 R2 ≥ m 2 a + m2 b + m2 c + m2 d ≥ h2 a + h2 b + h2 c + h2 d ≥≥ 1 4 (h a + h b + h c + h d ) 2 ≥h a h b h c h d≥ 16√ (ha + h b )(h b +h c )(h c +h d )(h d +h a ) ≥64r2 .≥d) 64 9 R2 ≥ m 2 a + m 2 b + m2 c + m 2 d ≥ h2 a + h 2 b + h2 c + h 2 d ≥≥ 1 4 (h a + h b + h c + h d ) 2 ≥≥ 16 ·h a h b h c h√ d(ha + h b )(h a +h c )(h a +h d )(h b +h c )(h b +h d )(h c +h d ) ≥64r2 .3( ) 4R 416R5. Menţionăm că prin majorările ≥ h a h b h c h d şi ≥33≥ h a + h b + h c + h d aplicate relaţiilor (21*), (22*), (23), (24), (25) şi (26),de la pag. 28 din [2] se obţin direct încadrări/rafinări ale inegalităţii Euler-Durrande, valabile în orice tetraedru [ABCD].6. Pentru propoziţia1egalităţile se ating numai dacă [ABCD] estetetraedru echifacial. Această condiţie rămâne valabilă şi pentru inegalităţile


200 Articolei) şi ii) ale punctului 3 din prezentele observaţii. Pentru restul inegalităţilorstabilite până în prezent, egalităţile se ating simultan numai dacă [ABCD]este tetraedru regulat.Propoziţia 2. În orice tetraedru [ABCD] are loc următoarea rafinareainegalităţii Euler-Durrande:64R 23≥ a 2 + b 2 + c 2 + l 2 + m 2 + n 2 3+≥1a 2 + l 2 + 1b 2 + m 2 + 1c 2 + n 2≥ 4 3√ (a 2 + l 2 )(b 2 +m 2 )(c 2 +n 2 )≥8 3√ abclmn ≥ 16 3√ 9V 2 ≥ 192r 2 .Demonstraţie. În lema - a) considerăm x = a 2 + l 2 , y = b 2 + m 2 ,z = c 2 + n 2 etc. Egalitatea se atinge dacă şi numai dacă a 2 + l 2 = b 2 + m 2 == c 2 + n 2 , echivalent cu faptul că [ABCD] este ortocentric.Mai departe avem:1a 2 + l 2 + 1b 2 + m 2 + 1c 2 + n 2 ≥ 9a 2 + b 2 + c 2 + l 2 + m 2 + n 2 ,rezultă:3≤ a2 + b 2 + c 2 + l 2 + m 2 + n 2. (1)1a 2 + l 2 + 1b 2 + m 2 + 13c 2 + n 2Deoarece:16R 2 ≥ a 2 + b 2 + c 2 + l 2 + m 2 + n 2 , (2)([4], pag. 23, problema 79), atunci, din (1) şi (2), rezultă inegalitatea dinmarginea stângă.În continuare, a 2 + l 2 ≥ 2al, b 2 + m 2 ≥ 2bm, c 2 + n 2 ≥ 2cn implică:4 3√ (a 2 + l 2 )(b 2 +m 2 )(c 2 +n 2 )≥8 3√ abc l mn. (3)Se ştie că:72V 2 ≤ abclmn, (4)([4], pag. 38, problema 166) şi:V ≥ 8 3√ 3r 3 (5)([7], pag. 473, relaţia (12)).Din relaţiile (3), (4) şi(5)seobţine imediat rezultatul căutat.Toate egalităţile se ating simultan numai dacă [ABCD] este tetraedruregulat.Propoziţia 3. În orice tetraedru ortocentric [ABCD] are loc următoarearafinare a inegalităţii Euler-Durrande:4R 23 ≥ R2 +3r 2 ≥ 1 (a 2 + b 2 + c 2 + l 2 + m 2 + n 2) ≥ 12r 2 . (5 ∗ )12Demonstraţie. Vom demonstra. pentru început, inegalitatea din centru.


M. Olteanu, Asupra rafinării unor inegalităţi în tetraedru 201Conform teoremei 5, pag. 165, din [3], avem relaţia:HI 2 = R 2 +3r 2 − 1 (a 2 + b 2 + c 2 + l 2 + m 2 + n 2) ≥ 0. (6)12În continuare, din R ≥ 3r, rezultă 4R2 ≥ R 2 +3r 2 , iar din [8], pag.3630, se ştie că a 2 + b 2 + c 2 + l 2 + m 2 + n 2 ≥ 144r 2 .Egalităţile se ating numai dacă [ABCD] esteregulat.Având în vedere inegalitatea (5*), prin utilizarea ei – sub diferite forme– se pot rafina, prin trecere la clasa tetraedrelor ortocentrice, rezultatelestabilite în [7], pag. 471-478, [10] pag. 99-108, [9] pag. 203-207, [8] pag. 625-630, [2] pag. 35. În acest sens, prezentăm în continuare, sub formă sintetică,câteva dintre aceste noi rafinări ale inegalităţii Euler-Durrande, cu valabilitate– aşa cum am precizat – doar pentru clasa tetraedrelor ortocentrice.Enunţăm deci:Propoziţia 4. În orice tetraedru ortocentric [ABCD] au loc următoarelerafinări ale inegalităţii Euler-Durrande:a) 64r 2 ≤ h 2 a + h 2 b + h2 c + h 2 d ≤ m2 a + m 2 b + m2 c + m 2 d ≤≤ 16 (R 2 +3r 2) ≤ 6439 R2 .b) 16r ≤ h a + h b + h c + h d ≤ m a + m b + m c + m d ≤≤ √ 8 √R 2 +3r 2 ≤ 163 3 R.c)1r= 1 h a+ 1 h b+ 1 h c+ 1 h d≥ 1 m a+ 1 m b+ 1 m c+ 1 m d≥≥2 √ 3√R 2 +3r 2 ≥ 3 R .d) 48 · r2R ≤ 32√ 3r 2√R 2 +3r ≤ h2 a+ h2 b+ h2 c+ h2 d≤2 m a m b m c m d8√3·√R 2 +3r 2 ≤ 16R3 .e) 36r2R 2 ≤ 48r2R 2 +3r 2 ≤ h2 am 2 + h2 ba m 2 b+ h2 cm 2 c+ h2 dm 2 df) 8 √ 3r 3 ≤ V ≤ 2 √3r ( R 2 +3r 2) ≤ 8rR23 √ 3 .≤4.g) 4√ 23 R ≥ 2√ 2√ √ R 2 +3r 2 ≥r A +r B +r C +r D ≥4 √ 2r.3h) 8 9 R2 ≥ 2 (R 2 +3r 2) ≥rA 2 3+r2 B +r2 C +r2 D ≥8r2 .


202 Articolei) 8 √ ( ) 223 R − rj)329≥ 8√ 2( √3R32 +9r −3r)2 ≥≥R A +R B +R C +R D ≥8 √ 2r.(5R 2 − 36r 2) ≥ 8( 5R23 − 11r2 )≥≥ RA 2 + R2 B + R2 C + R2 D ≥ 32r2 .k) 6√ 2R ≤ 4 √ 6√R 2 +3r ≤ 1 + 1 + 1 + 1 ≤ 2√ 22 r A r B r C r D r .l) 18R 2 ≤ 24R 2 +3r 2 ≤ 1rA2 + 1rB2 + 1rC2 + 1rD2 ≤ 2 r 2.√ √26m) ≤ √23 R − r R 2 +3r 2 − √ 3r ≤ 1 + 1 + 1 + 1 ≤R A R B R C R D√2r .n) 9 2 · 15R 2 − 36r 2 ≤ 65R 2 − 33r 2 ≤ 1RA2 + 1RB2 + 1RC2 + 1RD2 ≤ 12r 2 .o) 64r 4 ≤ R A R B R C R D ≤ 64 ( √R92 +3r 2 − √ ( ) 2 43r)≤643 R − r .(p) 4r 4 ≤ r A r B r C r D ≤ S2 R 216 · 27 ≤ +3r 2 ) 2≤ 4 6 81 R4 .9q)4R 2 ≤ 3R 2 +3r 2 ≤ 1 a 2 + 1 b 2 + 1 c 2 + 1 l 2 + 1 m 2 + 1 n 2 ≤ 14r 2.r) 6 ≤ a b + b c + c l + l m + m n + n a ≤ 1 r · √3(3r 2 +R 2 )≤2 R r .9 3s)4R 2 ≤R 2 +3r 2 ≤ 1 m 2 + 1a m 2 + 1bm 2 + 1c m 2 ≤d≤ 1 + 1 + 1 + 1 ≤ ∑ 1m a h a m b h b m c h c m d h d h 2 ≤a( R 2 +3r 2 ) 2≤24r 3 ≤ 1324 · R4r 6 .t) 1 r 2 ≤ 1 ra2 + 1 rb2 + 1 rc2 + 1 ( R 2 +3r 2 ) 2rd2 ≤12r 3 ≤ 181 · R4r 6 .u) 21027 ·R3 ≥ 273 √ 39v)4R 2 ≤(R 2 +3r 2)3 2≥(∑m2a)(∑ha)≥ 128 √3·V ≥ 2 10·r 3 .3R 2 +3r 2 ≤ 3 b 2 ≤ 11 d 2 + 11 d 2 + 12 d 2 =3= 1 ( 14 ra2 + 1 rb2 + 1 rc2 + 1 )rd2 ≤ 1 ( R 2 +3r 2 )4 12r 3(b 1 = b 2 = b 3 ,[ABCD] fiind ortocentric).≤ 1324 · R4r 6 ;


M. Olteanu, Asupra rafinării unor inegalităţi în tetraedru 203w) AX4km n a+ BX4km n b+ CX4km n c≥34k+n·41−n·r4k+ DX4km n d≥ 2 2−n ·3 4k+ n 2 ·r 4k ( R 2 +3r 2) − n 2R n, unde k ∈ N∗ , n ∈{0,1,2},X∈{O, G, H, Ω, Γ}.(x) AXnm 2 + BXna m 2 + CXnbm 2 + DXn Rc m 2 ≤ R n 2 +3r 2 ) 2·d24r 3 ≤ 1324 · R4+nr 6 ,unde n ∈{0,1,2},iarX∈{O, G, H, Ω, Γ}.y) AX + BX + CX + DX ≤ R ( R 2 +3r 2)r a r b r c r d 6r 3 ≤ 2 9· R3r 3 ,unde X ∈{O, G, H, Ω, Γ}.( )nz) hm am n + hm ba m n + hm cbm n + hm dc m n ≥ 2 2+2m−n · r m 3 2·≥( ) dR 2 +3r 23n≥4 1+m−n · r m · , ∀ m ∈{0}∪[1, ∞), n ∈{0,1,2}.Rα)8≤ m ar a+ m br b+ m cr cβ) 4≤ m ah a+ m bh b+ m ch c+ m d+ m d≤ 1r d 3 √ 3· 1 (R 2r 3 +3r 2)3 2(R 2 +3r 2)3 2h d≤6 √ 3r 3 ≤ 427 · R3r 3 .≤ 8 27 · R3r 3 .γ) 64r 2 ≤ ∑ m a h a ≤ 16 (R 2 +3r 2) ≤ 64 39 R2 .Dacă, în plus, tetraedrul ortocentric [ABCD] are ortocentrul H ∈∈ int(ABCD) semaipotrafinaşi rezultatele stabilite în [6] pag 22-23, [8]pag. 628-629 şi [2] pag. 57 problema 305.Avem aşadar:Propoziţia 5. În orice tetraedru ortocentric [ABCD] având ortocentrulH ∈ int(ABCD) au loc următoarele rafinări ale inegalităţii Euler-Durrande:( )na) rm am n + rm ba m n + rm cbm n + rm dc m n ≥ 2 2+m−n · r m 3 2·≥) dR 2 +3r 2n≥2 2+m−2n · r m · , ∀ m, n ∈{0}∪[1, ∞).( 3Rb) Sm Am n + Sm Ba m n + Sm Cbm n + Sm Dc m n d≥4 1−m−n·c)r m ah m a · m a( 3R( )n≥ 2 2−2m−n 3 2··S mR 2 +3r 2 ≥) n·S m ,∀m, n ∈{0}∪[1, ∞).+ rm bh m b · m b≥2 −m· 3 ,∀m∈{0}∪[1, ∞).R+ rm ch m c · m + rm dc h n d · m d( )1≥ 2 1−m 3 ·R 2 +3r 22≥≥


204 Articoled) 4 ≤ m a+ m b+ m c+ m d≤ 2h a h b h c h d r √ 3 · √R 2 +3r 2 ≤ 4R3r .e) 12 · rR ≤ 8r √ 3√R 2 +3r ≤ h a+ h b+ h c+ h d≤4.2 m a m b m c m df) 16r ≤ m2 a+ m2 b+ m2 c+ m2 d≤ 4( R 2 +3r 2)≤ 16 h a h b h c h d 3r 9 · R2r .( ) 2 ( ) 2 ( ) 2 ( ) 2(ma mb mc md R 2 +3r 2) 3g) 4 ≤ + + + ≤h a h b h c h d 2 4·33·r6 ≤ 4729 · R6r 6 .h)1324 · R4r 6 ≥ ( R 2 +3r 224r 3 ) 2≥ ∑ 1i) m a S A +m b S B +m c S C +m d S D ≤√3≥m a h a 2r · √R 2 +3r ≥ 32 4Rr .√3 (a 2 + b 2 + c 2 + l 2 + m 2 + n 2) 3 2≤18≤ 4 ( R 2 +3r 2)3 2≤ 32R33 √ 3 .Propoziţia 6. În orice tetraedru ortocentric [ABCD] au loc inegalităţile:a) R 2 ≥ r 2 + rA 2 + r2 B + r2 C + r2 D ≥ 9r2 ;b) OH 2 ≥ OI 2 +3HI 2 .Demonstraţie. a) [ABCD] fiind ortocentric, avem identităţile:a 2 + l 2 = b 2 + m 2 = c 2 + n 2 , (7)([4], pag. 6, teorema T. 10-a));h 2 a +4R2 A =h2 b +4R2 B =h2 c +4R2 C =h2 d +4R2 D =a2 +l 2 (8)([4], pag. 1, problema 47).Ţinând seama de relaţia(5*)apropoziţiei 3, împreună cuidentităţile(7) şi (8) se obţine:R 2 +3r 2 ≥ 112 · 3 · (a 2 + l 2) = a2 + l 2= 4 ( a 2 + l 2)=∑ 4 16h2a +4·∑RA2== 11616 · ∑ h 2 a + 1 ∑R24 A .(9)Dar:116 · ∑ h 2 a ≥ 4r 2 (10)şi: ∑R2A ≥ 4 ∑ rA, 2 (11)(deoarece R A ≥ 2r A şi anloagele, conform inegalităţii lui Euler).Din (9), (10) şi (11) rezultă că:R 2 ≥ r 2 + rA 2 + r2 B + r2 C + r2 D . (12)


M. Olteanu, Asupra rafinării unor inegalităţi în tetraedru 205Conform relaţiei (8), pag. 99 din [10] avem:rA 2 + r2 B + r2 C + r2 D ≥ 8r2 implică r 2 + rA 2 + r2 B + r2 C + r2 D ≥ 9r2 . (13)Din (12) şi (13) rezultă că:R 2 ≥ r 2 + rA 2 + r2 B + r2 C + r2 D ≥ 9r2 .b) Deoarece:HI 2 = R 2 +3r 2 − 1 (a 2 + b 2 + c 2 + l 2 + m 2 + n 2) ,12OG 2 = R 2 − 1 (a 2 + b 2 + c 2 + l 2 + m 2 + n 2) ,162 · OG = OH,rezultă 16·OG 2 − 12 · HI 2 =4 ( R 2 −9r 2) .Dar, conform problemei 333, pag. 63 din [4] avem R 2 −9r 2 ≥ OI 2 . Deci16 · OG 2 − 12HI 2 ≥ 4OI 2 este echivalentă cu4OG 2 ≥ 3HI 2 + OI 2 sauOH 2 ≥ OI 2 +3HI 2 .Observaţie. Din OH 2 ≥ OI 2 +3HI 2rezultă OH 2 ≥ OI 2 + HI 2 , adică punctulI se află situat în interiorul sferei dediametru [OH]. În plus, dacă tetraedrul[ABCD] este ortocentric, având ortocentrulH ∈ int(ABCD), se ştie, conform problemei336, pag. 64 din [4], că punctul I este situatîn exteriorul sferei de diametru [OG] Fig. 1În concluzie, în cazul tetraedrelor ortocentrice având ortocentrul în interiorullor, centrul sferei înscrise tetraedrului este situat în zona cuprinsăîntre exteriorul sferei de diametru [OG] şi interiorul sferei de diametru [OH](vezi fig. 1).Propoziţia 7. Fie [ABCD] un tetraedru ortocentric. Dacă I estecentrul sferei înscrise tetraedrului, atunci are loc următoarea rafinare a inegalităţiiEuler-Durrande:12r ≤ AI + BI + CI + DI ≤ 2 √ AI 2 + BI 2 + CI 2 + DI 2 ≤ 4R.Demonstraţie. Din teorema medianei aplicată în triunghiul OHI,avem:4GI 2 =2(OI 2 + HI 2 ) − OH 2 =2(OI 2 + HI 2 ) − 4OG 2 ;rezultă:2GI 2 = OI 2 + HI 2 − 2OG 2 ≤ OH 2 − 2OG 2 =2OG 2şi deci:GI 2 ≤ OG 2 .(∗∗)Din relaţia lui Leibniz avem:AI 2 + BI 2 + CI 2 + DI 2 =4GI 2 + GA 2 + GB 2 + GC 2 + GD 2 =


206 Articole=4(GI 2 + R 2 − OG 2 ) ≤ 4R 2 ,conform relaţiei (∗∗).Aplicând inegalitatea dintre media aritmetică şi media pătratică avem:AI + BI + CI + DI ≤ 2 √ AI 2 + BI 2 + CI 2 + DI 2 ≤ 4R.În continuare, se ştie că dacăP este un punct în interiorul tetraedruluioarecare [ABCD], iar [A ′ B ′ C ′ D ′ ] este tetraedrul său pedal, unde {A ′ } ==(AP ∩[BCD], {B ′ } =(BP ∩[ACD] etc., atunci, conform problemei 92-d),pag. 26 din [4] avem PA·PB ·PC ·PD ≥ 81 · PA ′ ·PB ′ ·PC ′ ·PD ′ . DacăP ≡ I, atunci PA ′ ≡ IA ′ ≥ r (şi analoagele) şi rezultă că:AI · BI · CI · DI ≥ 81r 4 .Cum AI + BI + CI + DI ≥ 4 4√ AI · BI · CI · DI, obţinem că:AI + BI + CI + DI ≥ 12r.(∗∗∗)Propoziţia 8. Fie [ABCD] un tetraedru oarecare, iar I centrul sferei,de rază r, înscrisă tetraedrului. Dreptele AI, BI, CI, DI intersectează adoua oară sfera de rază R, circumscrisă tetraedrului,în punctele A 1 , B 1 , C 1şi respectiv D 1 . Atunci are loc următoarea rafinare a inegalităţii Euler:( ) 3r 8≤ V ( )[A 1 B 1 C 1 D 1 ] R 8≤ .R V [ABCD] 3rDemonstraţie. Conform problemei 337, pag. 64 din [4] se ştie că:V [A1 B 1 C 1 D 1 ]S A S B S C S D≥ 256 ·V [ABCD] (S A + S B + S C + S D ) 4 . (14)Din geometria triunghiului se ştie că S A ≥ 3 √ 3rA 2 , S B ≥ 3 √ 3rB 2 ,S C ≥ 3 √ 3rC 2 , S D ≥ 3 √ 3rD 2 ; rezultă:S A S B S C S D ≥ 3 6 · (r A r B r C r D ) 2 . (15)Însă, conform punctului o) al propoziţiei 4, avem r A r B r C r D ≥ 4r 4 ,deunde, ţinând cont de (15), obţinem:Din geometria tetraedrului se ştie că:Din (14), (16) şi (17) obţinem:S A S B S C S D ≥ 3 6 · 4 2 · r 8 . (16)S A + S B + S C + S D = S ≤ 8 √3R 2 . (17)V [A1 B 1 C 1 D 1 ]V [ABCD]≥ 256 · 36 · 4 2 · r 8 · 3 22 12 · R 8 =( ) 3r 8.R


208 Articolerazele sferei înscrise respectiv circumscrise tetraedrului [ABCD], atunciareloc inegalitatea 1 ≤ m x≤ R ,oricarearfix∈{a, b, c, d}.h x 3rAcest rezultat ar extinde la tetraedru cunoscuta inegalitate dintr-unRtriunghi ABC,2r ≥ m a(unde de această dată, notaţiile sunt cele cunoscuteîntr-un triunghi) şi analoagele, rezultat ce aparţine regretatului profesorh aLaurenţiu Panaitopol.Bibliografie[1] M. Dincă, M. Bencze, About inequalities, Octogon Mathematical Magazine, vol. 12,nr. 2A october 2004.[2] M. Miculiţa, M. Olteanu, Rafinări ale unor inegalităţi geometrice în tetraedru,G. M.-A, nr. 1/2008.[3] L. Nicolescu, A. Bumbăcea, A. Catană, P. Horja, G. G. Niculescu, N. Oprea, C. Zara,Metode de rezolvare a prolemelor de geometrie, Editura Universităţii din Bucureşti,1998.[4] M. Olteanu, Inegalităţi în tetraedru – culegere de probleme, Editura Universitară Conspress,Bucureşti, 2003.[5] M. Olteanu, În legătură cu o problemă dată la olimpiada de matematică dinPolonia,1992, R. M. T., nr. 3/2004.[6] M. Olteanu, Asupra unor inegalităţi în tetraedru, G. M. - B, nr. 1/2006.[7] M. Olteanu, Rafinări ale inegalităţii Durrande în tetraedru – partea I, G.M.-B,nr.8/2006.[8] M. Olteanu, Rafinări ale inegalităţii Durrande în tetraedru – partea a II-a, G.M.-B,nr. 12/2006.[9] M. Olteanu, Asupra unor inegalităţi în tetraedru, G. M. - A, nr. 3/2006.[10] M. Olteanu, Noi rafinări ale inegalităţii lui Durrande în tetraedru, G.M.-A,nr.2/2008[11] M. Onucu-Drimbe, Inegalităţi, idei şi metode, Biblioteca Olimpiadelor de Matematică,nr. 6, Editura Gil, Zalău, 2003.


R. Gologan and C. Lupu, An Olympiad problem 209NOTE MATEMATICE ŞI METODICEAn Olympiad problem:Zeroes of functions in the image of a Volterra operatorRadu Gologan 1) and Cezar Lupu 2)Abstract. We consider a generalization of an olympiad problem whichcan be regarded as a result for a Volterra operator.Keywords: mean value theorem, integrals, Volterra operator.MSC : 26A24, 26A33.Introduction & Main resultOne of the problems of the Romanian National Olympiad in 2006 wasthe following:Let f :[0,1] → R be a continuous function with:∫ 10f(x)dx =0.Show that there exists c ∈ (0, 1) such that:∫ c0xf(x)dx =0.In what follows we give two proofs to this problem. In the secondproof we shall use a mean value theorem due to Flett. For more details werecommend [2] and [3].First proof.∀t ∈ (0, 1).and let F (t) =We assume by contradiction thatWithout loss of generality, let∫ t0f(x)dx. Integrating by part, we obtain:∫ t0∫ t0xf(x)dx ≠ 0,xf(x)dx > 0, ∀ t ∈ (0, 1)Polytechnic University of Bucharest, Department of Mathematics II and Instituteof Mathematics ,,Simon Stoilow“ of the Romanian Academy, Bucharest; E–mail:Radu.Gologan@imar.ro2) University of Bucharest, Faculty of Mathematics and University of Craiova, Facultyof Mathematics, E-mail: lupucezar@yahoo.com, lupucezar@gmail.com1)


210 Note Matematice şi Metodice0 0,so µ is increasing on the interval (0, 1), so it is increasing on the interval [0, 1](by continuity argument). Because H(0) = 0, it follows that:∫ 100F (x)dxF (x)dx >0,which is in contradiction with (∗). So, there exists c ∈ (0, 1) such that:∫ c0xf(x)dx =0.Second proof. We consider the following differentiable functionH :[0,1] → R, defined by:with H ′ (t) =∫ t0∫ tH(t) =t0∫ tf(x)dx −0xf(x)dxf(x)dx. It is clear that H ′ (0) = H ′ (1) =∫ 10(∗)f(x)dx =0.Applying Flett’s mean value theorem (see [1]), there exists c ∈ (0, 1) such


R. Gologan and C. Lupu, An Olympiad problem 211that:or:∫ ccwhich is equivalent to:0H ′ (c) = H(c)−H(0)c∫ cf(x)dx = c∫ c00∫ cf(x)dx −xf(x)dx =0.0xf(x)dx,An extension of theorem 1.1 was given in [4], namely:Theorem 1.1. Let f,g :[0,1] → R be two continuous functions. Thereexists c ∈ (0, 1) such that:∫ 10∫ cf(x)dx0xg(x)dx =∫ 10∫ cg(x)dx0xf(x)dx.The proof is almost the same with the second proof of theorem 1.1, onlythis time we shall consider the function ˜H :[0,1] → R defined by:∫ 1 ⎛∫ t ∫ t ⎞˜H(t) = f(x)dx ⎝t g(x)dx − xg(x)dx⎠ −∫ 1−00⎛g(x)dx ⎝t∫ t00∫ tf(x)dx −00⎞xf(x)dx⎠ .The proof of the main result involves some non-elementary facts. Thefollowing lemma will be used.Lemma 1.2. Let h :[0,1]→R be a continuous function and φ :[0,1]→R is nondecreasing, continuous in 0 and φ(0) = 0. Then:limt→0 +∫t0h(x)φ(x)dxφ(t)=0.Proof. We assume by contradiction that limt→0 +∫t0h(x)φ(x)dxφ(t)≠0.


212 Note Matematice şi MetodiceThus, there exists a sequence t n > 0 such thatwhich is equivalent to:∫ t n0h(x)φ(x)dxφ(t n )≥ c>0,∫ t n0h(x)φ(x)dx ≥ cφ(t n ) > 0.On the other hand, using the continuity and the fact that φ is nondecreasingwe obtain:0


R. Gologan and C. Lupu, An Olympiad problem 213Now, by letting ε → 0, if we assume that∫1˜H(1)φ(1) =0∫ 10˜H(x)d 1 φ .h(x)dx = 0, we get:This implies that the function ˜H(x) cannot be of constant sign on (0, 1).Thus there is c ∈ (0, 1) such that ˜H(c) = 0. In the particular case when:we clearly have∫ 1h(t) =f(t)∫ 1c ∈ (0, 1) such that∫ 1000∫ 1g(x)φ(x)dx − g(t)0f(x)φ(x)dx,h(x)dx = 0, so by the considerations above there exists∫ c0∫ cf(x)dxh(x)φ(x)dx = 0 which is equivalent to:0g(x)φ(x)dx =∫ 10∫ cg(x)dx0f(x)φ(x)dx.To formulate a consequence, denote by C([0, 1]) the Banach space ofcontinuous functions on [0, 1] and by C null the subspace of functions havingzero integral.Theorem 1.4. Let φ : [0,1] → R a nondecreasing function continuousat 0 and such that φ(0) = 0, and consider the Volterra operatorV φ : C([0, 1]) → C([0, 1]) given by V φ (f)(x) =φ(t)f(t)dt. Then, all functionsin V φ (C null ) have at least one zero in (0, 1).∫ x0Bibliografie[1] T.M. Flett, A mean value problem, Mathematical Gazette 42(1958), 38–39.[2] T.L. Rădulescu, V.D. Rădulescu, T. Andreescu, Problems in Real Analysis: asvancedcalculus on the real axis, Springer Verlag, 2009.[3] T. Lupu, Probleme de Analiză Matematică: Calcul Integral, GIL Publishing House,1996.[4] C. Lupu, T. Lupu, Problem 11290, American Mathematical Monthly, no. 4/2007.


214 Note Matematice şi MetodiceInegalităţi şi elemente de teoria şirurilorPetru Ivănescu 1) şi Florin Nichita 2)Abstract. In this note we prove an elementary inequality by instrumentsof mathematical analysisKeywords: sequence, Riemann integral.MSC : 40A05.Se dă următoarea inegalitate:12 < 1 3 + 1 4 < 1 5 + 1 6 + 1 7 + 1 8


P. Ivănescu şi F. Nichita, Inegalităţi şi elemente de teoria şirurilor 215adică:12 < 1 3 + 1 4 < 1 5 + 1 6 + 1 7 + 1 8


216 Examene şi concursuriii) Inegalitatea de la începutul lucrării poate fi abordată deelevideclasa a VII-a. Propoziţia dată poate fi expusă laclasaaXI-a. Parteadindemonstraţie care foloseşte sume Riemann este un exerciţiu greu, pentruelevii din clasa a XII-a.iii) Demonstraţiile noastre sunt foarte concise. La clasă sugerăm o abordaremai explicită.EXAMENE ŞI CONCURSURIConcursul Naţional de ocupare a posturilor didactice dinmunicipiul Bucureşti, 15 iulie 2009Sorin Rădulescu 1) şi I. V. Maftei 2)EnunţuriSubiectul I1. Să searatecă ecuaţia x 2 + y 2 + z 2 =2xyz nu are soluţii în numerenaturale nenule.2. Fie două triunghiuri ABC şi A ′ B ′ C ′ care au acelaşi centru de greutate.Să se calculeze suma:−−→AA ′ + −−→AB ′ + −−→AC ′ + BA −−→ ′ + −−→BB ′ + −−→BC ′ + −−→CA ′ + −−→CB ′ + CC −−→ ′ .3. a) Definiţi probabilitatea condiţionată.b) Fie n ∈ N, n ≥ 4. Alegem patru numere distincte din mulţimea{1, 2,...,n}.Notăm cu p n probabilitatea ca aceste patru numere să formezeo progresie aritmetică curaţia strict pozitivă. Să se calculeze p n .Subiectul II1. Să se determine a ∈ R astfel încât G =[a, +∞) să fie parte stabilăaluiRîn raport cu legea de compoziţie x ∗ y = xy − 2x −[2y, cux, y ∈ R.2. Să se determine valoarea maximă afuncţiei f : 0, π ]→ R,2f (x)=sin 3 x·cos 5 x.3. Să se calculeze:∫1xdtlimx→∞ x 4+cost .01) Profesor, Liceul ,,Aurel Vlaicu“, Bucureşti2) Profesor, Colegiul Naţional ,,Sf. Sava“, Bucureşti


S. Rădulescu şi I. V. Maftei, Concursul de ocupare a posturilor, 2009 217Subiectul III1. Proiectaţi unitatea de învăţare: ,,Progresii geometrice“, precizânddefiniţia unităţii de învăţare.2. Pentru tema ,,Şiruri monotone“, alcătuiţi un test formativ din treiitemi, menţionând definiţia testului formativ.3. Elaboraţi o propunere de opţional (Curriculum la decizia şcolii – C.D. Ş) în maximum o pagină, care să abordeze următoarele aspecte:a) titlul opţionalului;b) conţinutul opţionalului;c) argument care să motiveze propunerea opţionalului şi care să se referela unul dintre următoarele aspecte: nevoi ale elevilor, nevoi ale comunităţiilocale, formarea unor competenţe de transfer.Notă:• Toate subiectele sunt obligatorii.• Fiecăreia dintre cele trei probleme ale unui subiect i se va acorda 10puncte.• Se acordă 10 puncte din oficiu.• Timpul efectiv de lucru este de 4 ore.SoluţiiSubiectul I1. Dacă x, y, z este o soluţie a ecuaţiei din enunţ, rezultă că existăcel mai mare număr natural k cu proprietatea că 2 k divide pe x, y, z. Deciexistă a, b, c ∈ N ∗ cu proprietatea că x =2 k a,y=2 k b,z=2 k c. Din alegereanumărului natural k rezultă că cel puţin unul dintre numerele naturale a, bşi c este număr impar. Înlocuind în ecuaţie obţinem:sau, după simplificare:2 2k a 2 +2 2k b 2 +2 2k c 2 =2 3k+1 abc, (1)a 2 + b 2 + c 2 =2 k+1 · abc. (2)Pentru ca (2) să aibă loc, este necesar ca unul dintre numerele a, b,c să fieparşi celelalte impare. În acest caz, membrul stâng al relaţiei (2)este de forma 4m + 2, iar membrul drept este de forma 4n (multiplu de 4).Contradicţie. În concluzie, rezultă că ecuaţia din enunţ nuaresoluţii numerenaturale nenule.Observaţie. Se observă că, utilizând aceeaşi idee, se poate demonstracă oricare ar fi a ∈ Z, ecuaţia x 2 + y 2 + z 2 =2axyz nu admite decât soluţiax = y = z =0.Comentariu. Problema a fost considerată dificilă de foarte mulţi candidaţi.Au existat puţini candidaţi care au rezolvat-o corect. Dificultatea a


218 Examene şi concursuriconstat în faptul că puţini concurenţi au dovedit că ştiu să lucreze corect cuclasele de resturi modulo n.2. Notând cu G centrul de greutate al triunghiului ABC şi cu G ′ centrulde greutate al triunghiului A ′ B ′ C ′ avem:−→ −−→S = AA ′ + −−→AB ′ + −−→AC ′ + BA −−→ ′ + −−→BB ′ + −−→BC ′ + −−→CA ′ + −−→CB ′ + CC −−→ ′ =( −→ −−→= AG + GG ′ + −−→ ) ( −→G ′ A ′ −−→+ AG + GG ′ + −−→ ) ( −→G ′ B ′ −−→+ AG + GG ′ + −−→ ) G ′ C ′ +( −→ −−→+ BG + GG ′ + −−→ ) ( −→G ′ A ′ −−→+ BG + GG ′ + −−→)( −→G ′ B ′ −−→+ BG + GG ′ + −−→ ) G ′ C ′ ++( −→ CG +−−→=3( −→ CG +−−→GG ′ + −−→ ) G ′ A ′ +( −→ −→ −→)AG + BG + CG +3( −→ CG +−−→GG ′ + −−→G ′ C ′ ) =GG ′ + −−→G ′ B ′) +( −−→G ′ A ′ + −−→G ′ B ′ + −−→G ′ C ′) +9GG −−→ ′ .Ţinând seama că −→ AG + −→ −→ −→ −−→BG + CG = 0 şi G ′ A ′ + −−→ G ′ B ′ + −−→ G ′ C ′ = −→ 0,obţinem −→ S =9 −−→ GG ′ .În particular, când G ≡ G ′ ,obţinem −→ S = −→ 0.Comentariu. Problema nu a pus dificultăţi candidaţilor; mare partedin ei au rezolvat-o cu destul de multă uşurinţă.3. a) Fie A şi B evenimente cu P (A) ≠ 0. Atunci, probabilitatea ca săaibă loc evenimentul B în condiţiile în care a avut loc evenimentul A este datăde formula P A (B) = P(A∩B) şi se numeşte probabilitatea condiţionată aP(A)evenimentului B în raport cu evenimentul A.b) Prin definiţie probabilitatea căutată este:nr. cazuri favorabilep n =nr. cazuri posibile . (1)Numărul de cazuri posibile este egal cu Cn. 4 (2)Să calculăm numărul de cazuri favorabile.Fie a primul termen şi r raţia progresiei aritmetice; atunci progresiaeste de forma ÷a, a + r, a +2r, a +3r,cu:a≥1 şi a +3r≤n. (3)Din (2) va rezulta:1+3r≤a+3r≤n⇒1+3r≤n⇒r≤ n−13{ [ n−1şi din faptul că r ∈ N ∗ , rezultă căr∈ 1,2,3,...,3Deducem că numărul de cazuri favorabile este:[ ][ n−1 n−1[∑n−13][ ] n−1 3 3(n − 3r) =n −332r=1]}.]+1. (4)


S. Rădulescu şi I. V. Maftei, Concursul de ocupare a posturilor, 2009 219Înlocuind în (1) numărul de cazuri posibile din (2) şi numărul de cazurifavorabile din (4), obţinem în final:[ ]( [ ] )n − 1n−12n−3 −333p n =. (5)Observaţie. Problema se poate generaliza în modul următor:Fie k ∈ N ∗ şi n ≥ k ≥ 3. Să se determine probabilitatea ca alegândk elemente distincte din mulţimea {1, 2, 3,...,n} acestea să fieîn progresiearitmetică.Făcând un raţionament asemănător, se determină probabilitatea:p n =[ n − 1k − 12C 4 n](2n−(k−1)2C k n[ n − 1k − 1]− k +1Comentariu. Problema a fost apreciată de candidaţi ca foarte dificilăşi de aceea rezolvările nu au fost complete. S-a remarcat faptul că puţinicandidaţi au făcut dovada că deţin noţiuni de combinatorică şi noţiuni deteoria probabilităţilor.Subiectul II1. Dacă x = y = a, atunci avem a ∗ a = a 2 − 4a şi din faptul căa ∗ a ∈ G, rezultă a 2 − 4a ≥ a şi apoi a 2 − 5a ≥ 0 ceea ce este echivalent cua ∈ (−∞, 0] ∪ [5, ∞). Avem de analizat două cazuri:1) a ∈ [5, ∞) şi 2) a ∈ (−∞, 0].Cazul 1). Dacă a ∈ [5, ∞), atunci oricare ar fi x, y ∈ [a, +∞) avem:x∗y=xy − 2x − 2y =(x−2)(y − 2) − 4 ≥ (a − 2)(a − 2) − 4=a 2 −4a≥a.Am demonstrat că pentru a ∈ [5, ∞), G =[a, +∞) esteopartestabilăaluiRîn raport cu legea de compoziţie dată.Cazul 2). Dacă y =1∈[a, +∞) rezultă:x ∗ y = x ∗ 1=x·1−2x−2=−x−2≥a, ∀ x ∈ [a, +∞).Acest lucru este fals. Este suficient să tindem cu x →∞şi obţinem ocontradicţie. Deci (−∞, 0] nu poate fi parte stabilă.În concluzie mulţimea căutată esteG=[5,+∞).Comentarii. Problema a fost abordată demulţi concurenţi, dar mareamajoritate au demonstrat numai că a ∈ (−∞, 0] ∪ [(5, +∞). Cazul cânda ∈ (−∞, 0] a fost abordat de puţini concurenţi,carenuaudatsoluţii complete.S-a observat că nus-aînţeles în profunzime noţiunea de parte stabilăîn raport cu o lege de compoziţie internă, cu toate că astfel de tipuri deprobleme se găsesc din abundenţăîn culegerile de probleme şi în manualelealternative.).


220 Examene şi concursuri2. Problema în sine prezintă un anumit interes, motiv pentru care ovom soluţiona prin mai multe metode.Metoda 1. Vom studia maximul funcţiei[f : 0, π ]→ R, f (x) =sin 3 xcos 5 x2cu ajutorul derivatei de ordinul întâi.Prin derivare se obţine f ′ (x) = sin 2 xcos 4 x ( 3 − 8sin 2 x ) . Alcătuimurmătorul tabel de variaţie al funcţiei:√6πx 0 arcsin42f ′ (x) 0 + + +(0 − − − 0√ )6f(x) 0 ↗↗↗ f arcsin4 ↘↘↘ 0MaxCalculăm:( √ ) [ ( √ )] 3 [ ( √ )] 5666f arcsin = sin arcsin · cos arcsin =444(√ ) 3 (√ ) 56=1 − 3 = 75√ 1548 4096 .În concluzie, valoarea maximă afuncţiei este egală cu 75√ 154096 .Metoda 2. Funcţia se mai poate scrie sub forma:f(x) = ( sin 2 x ) 32 · (cos 2 x ) 5 2.Notând cos 2 x = y şi sin 2 x =1−y,obţinem o funcţie g :[0,1] → R,g(y) =(1−y) 3 2 ·y 5 2.Derivând în raport cu y obţinem:⎡⎤g ′ (y) =(1−y) 1 32 ·y 2 ⎣− 3 52 y −3 22+ 5 2 (1 − y) 3 2 − 1 2 ⎦ =(1−y) 1 32 ·y 2 ·5−8y.2Facem tabelul de variaţie pentru funcţia g şi obţinem:5y 018g ′ (y) + + + 0)− − − 0( 5g(y) 0 ↗↗↗ g 8Max( ( 35 3 2Calculăm g = ·8)8)( 58)52=75 √ 154096 .↘↘↘ 0


S. Rădulescu şi I. V. Maftei, Concursul de ocupare a posturilor, 2009 221Metoda 3. Vom demonstra următoarea:Lemă. Fie a 1 ,a 2 ,...,a n ∈R ∗ +, cu proprietatea că a 1 +a 2 +...+a n =1.Atunci este adevărată următoarea inegalitate:x a 11 · xa 22 · ...·xan n ≤a 1x 1 +a 2 x 2 +...+a n x n , ∀x 1 ,x 2 ,...,x n >0.Egalitatea are loc numai dacă x 1 = x 2 = ...=x n .Demonstraţie. Fie funcţia f :(0,+∞)→R,f(x)=−ln x. Evident:f ′′ (x) = 1 >0, ∀x∈(0, +∞) (1)x2 şi rezultă că funcţiaestestrictconvexă. Aplicând inegalitatea lui Jensenavem: ( n∑)n∑f a i x i ≤ a i f(x i ), ∀ x 1 ,x 2 ,...,x n >0. (2)i=1i=1Înlocuind în (2) funţia f obţinem:( n∑)− ln a i x i ≤i=1n∑−a i ln x i . (3)Înmulţind cu (−1) se obţine:(n∑n∑)a i ln x i ≤ ln a i x i . (4)i=1Dezvoltând după i:i=1a 1 ln x 1 + a 2 ln x 2 + ...+a n ln x n ≤ ln (a 1 x 1 + a 2 x 2 + ...+a n x n ), (5)sau:ln (x a 11 · xa 22 · ...·xan n )≤ln (a 1 x 1 + a 2 x 2 + ...+a n x n ),ceea ce implică:x a 11 · xa 22 · ...·xan n ≤a 1 x 1 +a 2 x 2 +...+a n x n , (6)adică inegalitatea propusă. În inegalitatea lui Jensen avem egalitate cândx 1 = x 2 = ...=x n .Vom demonstra următoarea:Teoremă. Fie a 1 ,a 2 ,...,a n ∈ R ∗ + şi a 1 + a 2 + ... + a n = a (dat).Atunci este adevărată următoarea inegalitate:x a 11 ·xa 22 ·...·xan n ≤a a 11 ·aa 22 ·...·aan n · 1a a (x 1 +...+x n ) a , ∀x 1 ,...,x n ∈(0, ∞).Demonstraţie. Aplicăm Lema precedentă şi obţinem:x a a11 · xa 22 · ...·xan n(x = 1ai=1a 2a) a1 ·x2 ·...·x an a n =


222 Examene şi concursuri=[ (x1a 1) a 1a·(x2a 2) a 2a=a a 11 ·aa 22 ·...·aan n ·])an axn a a 1· ...·(·aa1 ·a2 ·...·a an a n =a n[ (x1 ) a 1 ) ]a x2·( a 2 )an aa xn a·...·(≤a 1 a 2 a n(≤a a 11 ·aa 22 ·...·aan n ·a1a · x1 + a 2a 1 a · x2 +...+ a ) ana 2 a · xn =a n( )=a a 11 ·aa 2x1 +x 2 +...+x an2 ·...·aan n ·=a=a a 111 ·aa 22 ·...·aan n ·a a (x 1 +x 2 +...+x n ) a . (1)Egalitatea are loc când x 1= x 2= ...= x n.a 1 a 2 a nCorolar. Dacă x 1 + x 2 + ...+x n =1,înlocuind în inegalitatea (1) dinteoremă, obţinem:x a 11 · xa 22 · ...·xan n ≤ 1 a a ·aa 11 ·aa 22 ·...·aan n . (2)a 2aAvem egalitate pentru x i = a iaoricare ar fi i ∈{1,2,...,n}.Dacă în (2) considerăm x 1 =sin 2 x,x 2 =cos 2 x,a 1 = 3 2 ,a 2 = 5a = a 2şi1 + a 2 = 4, vom obţine:(sin 2 x ) 32 · (cos 2 x ) 5 2=sin 3 x·cos 5 x ≤ 1 ( ( 34 2)34 ·2 5 2 75 · =2)5 √ 154096 .Observaţie. Problema poate fi abordată la cazul general:Să se determine valoarea maximă afuncţiei :[f : 0, π ]→ R, f (x)=sin p x·cos q x, p, q ∈ N.2Aplicăm corolarul pentru cazul când a 1 = p 2 , a 2 = q 2 , a = a 1 + a 2 == p + q , n =2şi obţinem:21f max = ( m + n2) p+q2( p(2 q 2· · .2)p2)qComentarii. Marea majoritate a concurenţilor (candidaţilor) au abordatproblema folosind prima metodă. O parte dintre ei au fost depunctaţideoarece nu au făcut √tabelul de variaţie al funcţiei f , de unde rezulta cu6uşurinţă x =arcsin ca punct de maxim.4


S. Rădulescu şi I. V. Maftei, Concursul de ocupare a posturilor, 2009 223Mai facem precizarea că doar un număr redus de candidaţi au duscalculele până lacapăt.3. Pentru început vom demonstra următoarea:Teoremă. Fie f : R → R o funcţie continuă şi perodică de perioadăT>0. Atunci este adevărată următoarea egalitate:1limx→∞ x∫ x0f(t)dt = 1 T∫ T0f(t)dt.Demonstraţie. Să notăm cu F : R → R oprimitivă a funcţiei f şi cux+T ∫g funcţia g : R → R, g(x) = f(t)dt. Avem:de unde:1x∫ x0xg ′ (x) =(F(x+T)−F(x)) ′ = f(x + T ) − f(x) =0,∀x∈R.Deci funcţia g este constantă şi atunci oricare ar fi k ∈ Z, rezultă:(k+1)T∫kTf(t)dt =g(kT) =g(0),∫ T0f(t)dt, ∀ k ∈ Z. (1)Dacă x>0 rezultă că există un unic număr natural k x cu proprietatea:Atunci:⎛f(t)dt = 1 ⎜⎝x∫ T0∫f(t)dt +k x T ≤ x


224 Examene şi concursuri∀ x ∈ (0, ∞) şi deci:1limx→∞ x∫ xk xTf(t)dt =0. (5)Trecând la limităîn (3) şi ţinând seama de (5) avem:1limx→∞ x∫ x0∫k T ∫ Txkxf(t)dt = lim f(t)dt = f(t)dt · limx→∞ xx→∞ x .00k xŢinând seama de (2) va rezulta că limx→∞ x = 1 . Vom obţine:T1limx→∞ x∫ x0f(t)dt = 1 T∫ T0f(t)dt. (6)Revenind la problema propusă observăm că funcţia f : R → R, f (t) =1= este continuă şi periodică de perioadă T =2π.4+costAplicând (6) vom avea:I =∫1xlimx→∞ x0dt4+cost = 1 ∫ 2π2πTotul se reduce la calculul integralei I =Avem succesiv:∫ 2π0dt4+cost =∫π−π∫0dt4+cost =∫ π−πdtIntegrala definită J =4+cost0de variabilă x = 2arctgu şi vom obţine:şi:J =∫ ∞00dt4+cost . (7)∫ 2π0∫πdt4+cost +0dt4+cost .∫πdt4+cost =20dt4+cost .se calculează cuajutorulschimbării∫∞1 2·4+ 1−u2 1+u 2 du= 2du5+3u 2 = √ π151+u 2 0I = √ 2π = 2π√ 15.15 15


S. Rădulescu şi I. V. Maftei, Concursul de ocupare a posturilor, 2009 225Atunci:∫1xlimx→∞ x · dt4+cost = 1 ·2π√ √15 15=2π 15 15 .0Observaţii. 1) Problema pusă în discuţie poate fi generalizată suburmătoarea:Teoremă. Fie f : R → R o funcţie continuă şi periodică de perioadăT>0.Dacăa, b > 0, atunciavem:a) limx→∞∫ ba1b) limx→∞ x1f(tx)dt = limx→∞ x∫ bxaxf(t)dt = b − aT∫ bxax∫ T·0f(t)dt;f(t)dt.Demonstraţie. a) Egalitatea de la punctul a) se obţine cu ajutorulschimbării de variabilă u = tx, de unde du = xdt şi pentru t = a rezultău = ax, iarpentrut=brezultă u = bx. Înlocuind obţinem:1x∫ baf (tx)dt = 1 xb) Avem:∫ bxaxf(t)dt = 1 x∫ 0ax∫ bxax∫ b1f(u)du ⇒ lim f(tx)dt = limx→∞x→∞ xaf(t)dt + 1 x= bbx∫ bx0∫ bx0f(t)dt = 1 xf(t)dt − aax∫ ax0∫ bx0f(t)dt.∫ bxaxf(t)dt − 1 xAplicând teorema demonstrată anterior în final avem:1limx→∞ x∫ bxax= b Tbf(t)dt = limx→∞ bx∫ T0f(t)dt − a T∫ T0∫ bx0af(t)dt − limx→∞ axf(t)dt = b − aT∫ T·0∫ ax0f(t)dt.f(u)du.∫ ax0f(t)dt =f(t)dt =Pentru cazul particular a =0,b=1,T =2πşi funcţia f : R → R,1f (t) = se obţine limita cerutăîn problemă.4+cost2. Problema propusă se poate generaliza în modul următor:


226 Examene şi concursuri∫Fie 0


N. Stanciu, Metode active în didactica matematicii 227DIDACTICA MATEMATICIIMetode active în didactica matematiciiNeculai Stanciu 1)Abstract. Mathematics is one of the gate keepers for success in all fieldsof life.The first question which arises in our mind as techers that why shouldwe teach Mathematics to our students? One of the main objectives ofteaching and learning Mathematics is to prepare students for practical life.Students can develop their knowledge, skills; logical and analytical thinkingwhile learning Mathematics and all these can lead them for enhancing theircuriosity and to develop their ability to solve problems in almost all fieldsof life. This problem solving nature of Mathematics can be found in subdisciplinesof Mathematics such as in geometry, calculus, arithmetic andalgebra. That’s why it is common saying the Mathematics is mother of allsubjects.This article illustrate practical use active methods of mathematics lessons.Keywords: Goals of mathematics teaching, curriculum development,teaching methods and classroom techniques, lesson preparation. Methodologyof mathematics, didactics.MSC : 00A35Procesul de predare şi învăţare este, în cea mai mare parte, un procesde comunicare între cel care predă (profesorul) şi cei care învaţă (elevii).Cele două componente ale acestui proces – predarea şi învăţarea – sunt eleînsele, în bună măsură, procese de comunicare sau care implicăîn mod directcomunicarea. A preda înseamnă aelaboraşi a transmite mesaje, iar a învăţa(cu sensul de a învăţa în clasă, în relaţie cu profesorul) înseamnă a recepta şi aasimila mesaje. Fireşte, procesul real al comunicării este mult mai complex.Aînvăţa în procesul de învăţământ nu se reduce la a recepta, ci implicăparticiparea activă a elevului în ambele ipostaze, de receptor şi emitent demesaje, după cum a preda nu se limitează la a transmite, ci implică şi actulreceptării şi al reacţiei de feed-back la mesaje emise de elevi, schimbareadinamică a rolurilor fiind una din condiţiile principale ale comunicării eficienteîn procesul de învăţământ. Important este faptul că procesul de predare –învăţare în matematică poate fi mai bine înţeles şi mai bine condus dacă secunosc şi se aplică câteva dintre metodele active, care se potrivesc acesteidiscipline. În cele ce urmează vă prezentăm utilizarea câtorva dintre acestemetode active folosite de noi la clasă – le vom exemplifica printr-un proiectdidactic.1) Profesor, Şc. gen. ,,George Emil Palade“, Buzău


228 Didactica MatematiciiProiect didacticClasa a VIII-aObiectul: Matematica / AlgebrăSubiectul: FuncţiiTipul lecţiei: LecţiedeconsolidareObiectivele lecţiei (1.4,1.5,2.2,2.3,2.6)Obiective de referinţă:1. Cunoaşterea şi înţelegerea conceptelor, a terminologiei şi a procedurilorde calcule specifice matematicii:1.4. să aplice în rezolvarea problemelor elemente de logică şi elementede teoria mulţimilor;1.5. să identifice funcţii de gradul I (domeniul R sau o mulţime finită)şi să le reprezinte grafic.2. Dezvoltarea capacităţii de explorare/investigare şi de rezolvare aproblemelor:2.2. să identifice reguli de formare a unor şiruri şi formule de definirea unor funcţii ;2.3. să analizeze veridicitatea unor rezultate obţinute prin procedeediverse (măsurare, calcul, raţionament);2.6. să determine, folosind metode adecvate (măsurare şi/sau calcul)lungimi de segmente , măsuri de unghiuri, arii şi volume.3. Dezvoltarea capacităţii de a comunica utilizând limbajul matematic.4. Dezvoltarea interesului şi a motivaţiei pentru studiul şi aplicareamatematicii în contexte variate.Obiective operaţionale:a) cognitive:–să reprezinte grafic o funcţie de gradul I;–să calculeze coordonatele punctului de intersecţie a graficelor pentrudouă funcţii date;–să determine o funcţie în condiţiile date.b) afective:– stimularea curiozităţii şi dezvoltarea simţului critic;– dezvoltarea spiritului de observaţie şi a concentrării în rezolvareaproblemelor;– concentrarea afectivă la lecţie.Metode şi procedee didactice: conversaţia, lucrul în echipă, demonstraţia,mozaicul,turul galeriei.Mijloace de învăţământ: manual, culegeri, instrumente geometrice,markere, coli A3.Desfăşurarea lecţiei1) Etapa organizatorică. Se notează absenţii, se verifică tema pentruacasă, comentându-se ideile de rezolvare enunţate de elevi, se captează atenţiaclasei prin anunţarea temei lecţiei şi a obiectivelor ....................2min


N. Stanciu, Metode active în didactica matematicii 2292) Reactualizarea cunoştinţelor. Metoda Mozaicului. Clasa se împarteîn grupe de câte 4-5 elevi, aleatoriu. Fiecare grupă primeşte o temă teoretică(alta pentru fiecare grupă), care se găseşte pe fişa nr. 1 de lucru, pe careo vor rezolva împreună timp de 10 min, o vor redacta pe un poster care vafiafişat pe tablă sau pe un alt suport. Un reprezentant al grupei ales deelevi, va prezenta răspunsurile argumentând. Membrii celorlalte grupe potpune întrebări, pot cere lămuriri sau completări. În acest timp profesorulcompletează ghidul de observare al elevilor. Dacă este nevoie profesorul sauelevii pot interveni ..................................................15min3) Fixarea cunoştinţelor. În continuare, fiecărei grupe i se va cere sărezolve problema corespunzătoare de pe aceeaşi fişă nr. 1pecareauprimit-o.Această problemăvafirezolvatădeasemeneaîn echipă şi va fi redactatăpe un poster pe care îl au la dispoziţie. Grupele rămân aceleaşi. Încadrul grupului pot apărea discuţii ,,certuri“ toate însă constructive. Eleviipot cere profesorului, pe parcursul activităţii, informaţii, lămuriri suplimentare,asupra enunţului, cerinţei, realizării desenului, demonstraţiei, etc.Toate posterele vor fi de asemenea afişate pe pereţii clasei .........15 min.Urmează turul galeriei. Grupele, într-o ordine bine stabilită, trec prinfaţa posterelor celorlalte grupe, menţionând folosind culoarea caracteristicăgrupei, observaţii, aprecieri (corecte sau nu) asupra modului de redactare,apreciind prin note. Aceştia trebuie să-şi argumenteze observaţiile, criticileşi metodele. Se impune supravegherea permanentă a elevilor pentrudesfăşurarea în condiţii optime a lecţiei ............................15 min.Se vor discuta şi alte metode de rezolvare a problemelor propuse.Concluzii şi aprecieri: ..........................................3min– ale profesorului: orale, criticând (dacă este cazul), dar mai alesîncurajând elevii.– ale elevilor: vor completa fărăsemnătură, biletele ce vor fi introduseîn ,,valiza activităţii“4) Tema pentru acasă. Problemele din Fişa de lucru nr.2 ....... 3minUlterior profesorul va întocmi fişa de evaluare a grupelor şi implicit aclasei stabilind măsurile de eliminare sau îndepărtare a deficienţelor.Pentru evaluarea activităţilor desfăşurate se utilizează:–Fişă de apreciere individuală şi– Chestionar de evaluare a lecţiei / activităţii.Fişa de lucru 1Funcţii – metode active –Reactualizarea cunoştinţelorClasa a VIII-aFie funcţiile f,g : R → R, f(x)=2x−6, g(x) =x−5. Se cere:Grupa 11) determinaţi coordonatele punctelor de intersecţie ale G f cu axeleOx şi Oy;


230 Didactica Matematicii2) reprezentaţi grafic funcţia f ;3) determinaţi coordonatele punctului de intersecţie ale graficelorfuncţiilor f şi g;Grupa 21) determinaţi coordonatele punctelor de intersecţie ale G g cu axeleOx şi Oy;2) reprezentaţi grafic funcţia g;3) determinaţi coordonatele punctului de intersecţie ale graficelorfuncţiilor f şi g;Grupa 31) distanţa dintre punctele de intersecţie ale G f cu axele de coordonate;2) perimetrul şi aria triunghiului format de graficul lui f cu axele;Grupa 41) distanţa dintre punctele de intersecţie ale G g cu axele de coordonate;2) perimetrul şi aria triunghiului format de graficul lui g cu axele;Grupa 51) distanţa de la origine la graficul funcţiei f ;2) raza cercului înscris şi raza cercului circumscris triunghiului formatde graficul lui f cu axele;Grupa 61) distanţa de la origine la graficul funcţiei g;2) raza cercului înscris şi raza cercului circumscris triunghiului formatde graficul lui g cu axele;Fixarea cunoştinţelorGrupa 11) rezolvaţi ecuaţia:f(x)+g(−2)3=4;Grupa 22) rezolvaţi inecuaţia: f(x)+2f(1) ≥ 6;Grupa 33) determinaţi coordonatele punctului de pe graficul funcţiei f ,careare ordonata triplul abscisei;Grupa 44) arătaţi că f(a)+f(b) ( ) a+b=f ;22Grupa 55) Care dintre punctele: A(1, −4), B(0, −6), C(−10, −26) se găsescpe graficul funcţiei f ?Grupa 66) Determinaţi a ∈ R, astfel încât (a − 2)f(−4) + 1 = 0.


N. Stanciu, Metode active în didactica matematicii 231Fişa de lucru 2Temăpentruacasă(în conformitate cu criteriile unice de evaluare la matematică, clasa a VIII-a)Capitolul: Funcţii, clasa a VIII-apentru nota 5-61) Fie f : R → R, f(x) ( =x−2. Calculaţi: (1a) f(1); b) f(0); c) f ;d)f(−2); e) f −3)1 ).62) Reprezentaţi grafic funcţia f : R → R, f (x) =x−3.pentru nota 6-71) Fie funcţia f : R → R, f (x) =x−3. Determinaţi m ∈ R astfelîncât punctele: a) A(m;7); b) B(m;−4); c) C(4; m); d) D(2; m) săaparţinăgraficului funcţiei.2) Determinaţi funcţia f : R → R, f (x) =ax + b, a, b ∈ R ştiind căreprezentarea graficului funcţiei conţine punctele: A(3; 4) şi B(−2; 3).pentru nota 7-81) Determinaţi funcţia f : R → R, f (x) =ax + b, a, b ∈ R care are careprezentare grafică dreaptaAB cu A( √ 3+1, √ 3) şi B(2, −1)2) Determinaţi punctele de intersecţie ale graficului funcţiei f : R → R,f (x) =− x 3 +1 6 , cu axele de coordonate x′ x şi y ′ y.pentru nota 9-101) Fie funcţia f : R → R, f (x) =x−4.a) Determinaţi a ∈ R, astfel încât (a − 2)f(a)+1=0.b) Determinaţi b ∈ R, astfel încât f(−b +1)=f(b+1)−f(b−1).c) Determinaţi un punct al graficului care are coordonatele numereopuse.2) Se consideră funcţia f : R → R, f (x) = ( 2− √ 5 ) x+ √ 5.a) Arătaţi că punctul A(1; 2) aparţine graficului funcţiei.b) Rezolvaţi în R inecuaţia f(x) − 2 > 0.c) Determinaţi numerele raţionale a, b pentru care punctulM ( a; b + b √ 5 ) aparţine graficului funcţiei f .Fişă de apreciere individuală:Unitatea de învăţământ Data. . . . . . . . .Clasa .........Profesor .........Elev .........La sfârşitul lecţiei completaţi spaţiile libere:1. Am învăţat că ............2. Am descoperit că ...............3. Am fost surprins de faptul că ............4. Am folosil metoda ............deoarece .............


232 ProblemeÎn realizarea sarcinilor am întâmpinat următoarele dificultăţi..........Vă mulţumesc pentru sinceritate !Chestionar de evaluare a lecţiei / activităţiiUnitatea şcolară Data .........Profesor .........Elev .........1. Marcaţi pe scala de mai jos utilitatea acestei lecţii/activităţi, dinperspectiva activităţii dvs.0 1 2 3 4 5inutilfoarte util2. Apreciaţi lecţia /activitatea de astăzi0 1 2 3 4 5inadecvatăfoarte bună3. Vă rugăm, enumeraţi 3 secvenţe care v-au captat interesul şi le-aţireţinut pentru agenda dvs.4. Au existat secvenţe complet neinteresante? Justificaţi, vă rugăm,răspunsul.5. Profesorul a reuşit să fie ......6. Cum v-aţi simţit în cadrul grupului?7. Apreciaţi participarea dvs. în cadrul acestei activităţi ..........8. Sugestii ..........Vă mulţumesc pentru sinceritate !Notă. Lecţia a fost filmată şi se găseşte pe www.mateinfo.ro .[1] www.didactic.ro[2] www.edu.ro[3] www.mateinfo.roBibliografiePROBLEME PROPUSE285. Fie (x n ) nun şir de numere strict pozitive. Să se arate că:∞∑x k1+(x 1 +x 2 +...+x k−1 ) 2 > π 2 .k=1În plus, dacă x 1 + x 2 + ...+x n = 1, atunci:∞∑x k1+(x 1 +x 2 +...+x k−1 ) 2 > π 4 .k=1


Probleme propuse 233Mai mult, constantele sunt optime. (În legătură cuoproblemă propusăla Concursul ,,Traian Lalescu“ din anul 2009.)Radu Gologan286. Fie f :(0,∞)→(0, ∞) o funcţie strict descrescătoare şi derivabilăpe(0,∞)şi fie F oprimitivă a ei. Presupunem că suntîndepliniteurmătoarele condiţii: ( ) f(n +1)i) şirulare limita 1;f(n)n≥1ii) şirul (F (n)) n≥1are limita 0;iii) f ′este o funcţiestrictcrescătoare pe intervalul (0, ∞).fAtunci:a) şirul (x n ) n≥1definit prin x n = f(1) + f(2) + ...+f(n)esteconvergent(notăm cu x limita lui);b) şirul (u n ) n≥1cu termenul general u n = x − x neste convergentF (n)(care este limita sa?) şi este strict monoton.Marian Tetiva287. Fie f :[a, b] → R o funcţie continuă astfel încât:∫ ta∫ tf(x)dxbf(x)dx ≠0, ∀t∈[a, b].Să se arate că există c ∈ (a, b) astfel încât:⎛∫ b ∫ c ⎞(c − a)f(c) ⎝ f(x)dx − f(x)dx⎠ = 1 2ca∫ ca∫ bf(x)dxcf(x)dx.Cezar Lupu şi Tudorel Lupu288. Fie p ≥ 2, q ≥ 2numereîntregi cu (p, q) =1. Să se demonstrezecă numărul log p q este transcendent.Adrian Troie289. Să se determine curbele plane care au proprietatea că razavectoarea unui punct curent face un unghi constant α (0


234 ProblemeSOLUŢIILE PROBLEMELOR PROPUSE264. Fie n ∈ N ∗ şi V spaţiul vectorial al polinoamelor de grad cel mult n − 1 cucoeficienţi complecşi. Vom nota cu id aplicaţia identică aluiV şi pentru orice a ∈ C vomdesemna prin u a : V → V aplicaţia definită de egalitatea:u a(p(x)) = p(x + a).a) Să searatecămulţimea G = {u a} a∈C este un subgrup al grupului GL(V )(grupulautomorfismelor lui V ) izomorf cu grupul aditiv al numerelor complexe. Folosind acestrezultat, să se precizeze un subgrup al grupului GL n(C) (grupul matricelor inversabile cucoeficienţi în C) izomorf cu grupul aditiv C. Să se determine inversa unei matrici din acestsubgrup.b) Pentru a ∈ C −{0}, se consideră în V polinoamele:p 0(x) =1, p xx(x−a)·...·(x−(n−2)a)1(x)= ,..., pn−1(x)= .1! · a (n − 1)! · a n−1Să searatecă familia {p 0,p 1....,p n−1} constituie o bazăîn V şi să se scrie matriceaendomorfismului u a − id în raport cu această bază.c) Pentru k ∈ N, să se determine ker(u a − id) k şi im(u a − id) k .d) Fie a, b, α, β ∈ C. Săsearatecăurmătoarele afirmaţii sunt echivalente:(i) (αu a + βu b ) n = o;(ii) ker (αu a + βu b ) ≠ {0};(iii) α + β =0.Dan RaduSoluţia autorului. a) Fie α, β ∈ C şi p, q ∈ V ; atunci:u a (αp(x)+βq(x)) = αp(x + a)+βq(x + a) =αu a(p(x)) + βu a(q(x)),ceea ce arată cău a∈L C (V). Pentru a arăta că G este subgrup în GL(V ), să observăm căpentru orice a, b ∈ C avem:(u a ◦ u b )(p(x)) = u a (u b (p(x))) = u a(p(x + b)) = p(x + a + b) =u a+b (p(x))şi deci u a ◦ u b = a a+b ∈ G. Cum pe de altă parteu 0 = id, rezultă cău −1a =u −a ∈G,de unde, în baza criteriului subgrupului, conchidem că G este subgrup în GL(V ). Evident,izomorfismul dintre grupul aditiv C şi grupul G este realizat de aplicaţia ϕ : C → G, datăde egalitatea ϕ(a) =u a.Să considerăm în V baza canonică {e 0,e 1,e 2,...,e n−1}.Vomavea:u a(e 0(x)) = e 0(x + a) =1=e 0(x)u a(e 1(x)) = e 1(x + a) =a+x=ae 0(x)+e 1(x)..........................................u a(e n−1(x)) = e n−1(x) =(a+x) n−1 =a n−1 +Cn−1a 1 n−2 x+...+C n−1n−1 xn−1 ==a n−1 e 1(x)+Cn−1a 1 n−2 e 1(x)+...+C n−1n−1 +en−1(x).Urmează că matricea M a aluiu a în raport cu baza canonică vafi:⎛1 a ... a n−1 ⎞0 1 Cn−1a 1 n−2M a= ⎜⎝.⎟.. ⎠ .0 0 ... 1Uzând de izomorfismul GL(V ) ≃ GL n(C), rezultă căGva fi izomorf cu subgrupulM⊆GL n(C) constituit din toat matricile de forma M a, unde a parcurge pe C. Conchidemcă grupul aditiv C este izomorf cu grupul multiplicativ M = {M a} a∈C. Din raţionamentele


Soluţiile problemelor propuse 235anterioare, deducem că pentruM a∈M,avem:⎛1 −a ... (−1) n−1 a n−1 ⎞Ma −10 1 Cn−1(−1) 1 n−2 a n−2= ⎜⎝.⎟..0 0 ... 1⎠ =M−a.b) Evident, deoarece familia considerată este constituitădinnvectori, pentru a probacă estebazăîn V va fi suficient să verificăm liniar independenţa.Să presupunem că:λ 0p 0(x)+λ 1p 1(x)+...+λ n−1p n−1(x)=0, ∀x∈C.Făcând în această egalitate x =0,x=a, ..., x=(n−1)a, obţinem sistemul:⎧λ 0 =0⎪⎨λ 0+λ 1 =0........................⎪⎩λ 0 +λ 1 +...+λ n−1 =0,de unde λ 0 = λ 1 = ...=λ n−1 =0şi deci liniar independenţa familiei considerate. Pentrua scrie matricea P a a endomorfismelor u a − id în raport cu această bază, să observăm că:(u a − id) (p 0(x)) = u a (p 0(x)) − p 0(x) =0,iar pentru k ∈{1,...,n−1},avem:(u a−id) (p k (x)) = u a (p k (x)) − p k (x) =p k (x+a)−p k (x)== (x+a)x·...·[x−(k−2)a] x(x − a) · ...·[x−(k−1)a]− =k!a kk!a kx(x − a) · ...·[x−(k−2)a]= = p(k − 1)!a k−1 k−1 (x).Prin urmare, matricea P a va fi:⎛P a =⎜⎝0 1 0 ... 0 00 0 1 ... 0 00 0 0 ... 0 0.....0 0 0 ... 0 10 0 0 ... 0 0deci o matrice banală semibordată superior cu o diagonală de1.c) În raport cu baza considerată la pct. b), matricea aplicaţiei (ua − id)k va fi Pa k .Evident, P a este nilpotentă deordinndeoarece fiecare ridicare la o putere succesivă aluiP a are drept efect deplasarea către dreapta cu o poziţie a bordului diagonal format dinnumărul 1. Urmează atunci că pentru1≤k≤n−1avem:ker(u a − id) k = S p {p 0(x),...,p k−1 (x)},im(u a − id) k = S p {p 0(x),...,p n−k−1 (x)},iar pentru k ≥ n:ker(u a − id) k = V,im(u a − id) k = {o}.d) (i)⇒(ii). Deoarece proprietatea (i) este adevărată, rezultă că:(αu a + βu b ) ◦ (αu a + βu b ) n−1 = oşi deci im (αu a + βu b ) n−1 ⊆ ker (αu a + βu b ).⎞,⎟⎠


236 ProblemeDacă im(αu a + βu b ) n−1 ≠ {0}, atunci rezultă proprietatea (ii). În caz contrar,conchidem că (αu a + βu b ) n−1 =0. În această situaţie este clar că procedând recursivdescendent vom ajunge la un moment dat la indicele de nilpotenţă k al endomorfismuluiαu a + βu b şi deci, pentru acest k, im(αu a + βu b ) k−1 ≠ {o}, iar im(αu a + βu b ) k−1 ⊆⊆ ker (αu a + βu b ), ceea ce demonstrează afirmaţia (ii).(ii)⇒(iii). Plecând de la premiza (ii), există p(x) ∈ V astfel încât:(αu a + βu b )(p(x)) = 0, ∀ x ∈ C,ceea ce este echivalent cu faptul că:αp(x + a)+βp(x + b) =0, ∀x∈C.Să presupunem că gradp = k ≤ n − 1. Atunci p se poate scrie sub forma:p(x) =a 0x k +q(x),cu a 0 ≠0şi gradq ≤ k − 1. Urmează deci că:αp(x+a)+βp(x+b) =αa 0(x+a) k +αq(x+a)+βa 0(x+b) k +βq(x+b) =a 0(α+β)x k +r(x),unde gradr ≤ k − 1. Însă, după cum am presupus mai înainte, polinomul din membrulstâng este polinomul identic nul, aşa încât rezultă cu necesitate că a 0(α + β) =0. Cumînsă a 0 ≠ 0, conchidem că α + β =0.(iii)⇒(i). Dacă α = β = 0, implicaţia este trivială. În caz contrar, egalitatea (i) esteechivalentă cu egalitatea:(u a − u b ) n =0.Evident, pentru a proba egalitatea de mai sus este suficient să arătăm că:(u a − u b ) n (e k (x)) = 0,pentru polinoamele e k din baza canonică. Să observăm că:(u a − u b ) n (e 0(x)) = 0.Mai departe:(u a − u b ) 2 (e 1(x)) = (u a − u b )(e 1(x+a)−e 1(x+b)) ==(u a−u b )((a−b)e 0(x)) = (a − b)(u a −u b )(e 0(x)) = 0.Să presupunem că:(u a − u b ) i (e i−1(x)) = 0, (1)pentru orice i ∈{1,...,k} şi să arătăm că de aici decurge faptul că:(u a − u b ) k+1 (e k (x)) = 0. (2)Pentru aceasta, să observăm că:(u a − u b )(e k (x)) = (x + a) k − (x + b) k = α 1e k−1 (x)+...+α k e 0(x)şi deci:(u a − u b ) k+1 (e k (x)) = (u a − u b ) k ((u a − u b )(e k (x))) ==(u a−u b ) k (α 1e k−1 (x)+...+α k e 0(x)) == α 1 (u a − u b ) k (e k−1 (x)) + ...+α k (u a −u b ) k (e 0(x)) = 0,ultima egalitate fiind justificată de ipotezele (1) făcute. Rezultă, prin recurenţă, afirmaţia(2). Evident atunci că endomorfismul (u a − u b ) n se anulează pe baza canonică aluiVşi, după cum observam mai la început, aceasta este suficient pentru ca afirmaţia (i) să fieadevărată.


Soluţiile problemelor propuse 237265. Fie (x n) n≥1un şir de numere pozitive care converge la 0 astfel încât:∞∑x n = ∞.n=1Atunci există unsubşir (x nk ) k≥1al lui (x n) n≥1astfel ca:∞∑x nk = ∞k=1Soluţia autorului. Pentru m ≥ 1notăm:{A m = n :şi∞∑x 2 n k< ∞.k=11m +1 ≤xn < 1 m},George Stoicaşi fie ( A mj)j≥1 subşirul format din mulţimile nevide ale lui (Am) m≥1 .Dacă:∞∑ 1= ∞,m jj=1atunci construim subşirul (x nk ) k≥1alegând câteunelementdinfiecareA mj .Dacă:∞∑ 1< ∞,m jj=1construim subşirul (x nk ) k≥1alegând min { }m j, cardA mj elemente din fiecare Amj (card∞∑înseamnă cardinalul mulţimii respective). Se observă uşor că x nk = ∞ şi apoi că:k=1j=1m 2 jj=1k=1∞∑∞∑x 2 min { }m j, cardA mj∞∑ 1n k≤≤ ,m jadică proprietatea din enunţ.Soluţie dată de Marian Tetiva, profesor la Colegiul Naţional Gheorghe Roşca-Codreanu din Bârlad. Săîncepem cu următoarea:Lemă. Dacă (x n) n≥1este un şir de numere reale pozitive, convergent la zero şi∞∑pentru care x n = ∞, iarmşi N sunt numere naturale nenule, atunci există numerelen=1naturale q şi p, q>p≥N,astfelîncât:q∑x k ≥ 1 q∑m şi x 2 k < 2 m . 2n=pn=pDemonstraţie. Deoarece (x n) are limita 0, există M ∈ N ∗ astfel încât x n < 1 mpentru orice n ≥ M;în particular avem şi x n < 1 , pentru orice n ≥ p : = max{M,N}. Pem∞∑de altă parte ipoteza implică, evident, şi faptul că x n = ∞, de aceea trebuie ca la unmoment dat o sumăs∑n=pn=px n să fie≥ 1 . Săalegempeqca fiind indicele unde se producem


238 Problemeschimbarea, deci q să fie acela cu proprietatea că:q−1∑x n < 1 q∑m şi x n ≥ 1 m .n=pAstfel vedem că are loc prima inegalitate din enunţ pentru această alegerealuipşi q.Ca să vedem că şi a doua este verificată nu este greu de loc; avem:q−1q∑ ∑x 2 n = x 2 n + x 2 q < 1 q−1∑x n + x 2 q < 1 mm · 1 ( ) 2 1m + = 2 m m . 2n=pn=pn=pAcum este destul de clar cum se rezolvă problema. Începem prin a alege un grupde termeni (consecutivi) ai şirului (x n) n≥1care au suma ≥ 1şi suma pătratelor mai micădecât 1; alegem adică indicii p 1


Soluţiile problemelor propuse 239Soluţia autorului. Avem:f(x) =lnϕ(x)= g(x)x ,unde g(x) = ax +b x +c xpentru orice x>0şi trebuie să arătăm că f este concavă pe3(0, ∞). Se calculează imediat derivata a doua, f ′′ (x) = h(x) , unde avem:x 3h(x) =x 2 g ′′ (x) − 2xg ′ (x)+2g(x), ∀x>0.Evident, h se poate considera definită pe[0,∞) punând h(0) = 0, prin continuitate.Apoi, h este şi ea derivabilă, având derivata:h ′ (x) =x 2 g (3) (x) ≤ 0, ∀ x ≥ 0,dacă reuşim să arătăm că g are derivata a treia negativă pe[0,∞). Atunci s-ar obţineh(x) ≤ h(0) = 0 pentru x ≥ 0, deci şi f ′′ (x) ≤ 0pentrux>0, ceea ce trebuia demonstrat.Ne mai rămâne aşadar să dovedim că g (3) (x) ≤ 0 pentru orice x>0.În acest scop calculăm:g ′ (x) = ax ln a + b x ln b + c x ln ca x + b x + c xşi apoi:g ′′ (x) = (ln a − ln b)2 a x b x +(lna−ln c) 2 a x c x +(lnb−ln c) 2 b x c x(a x + b x + c x ) 2 ,pentru a ajunge în cele din urmă la:g (3) (x) =1[∑=(ln b − ln a) 3(a x +b x +c x ) 3 (a x − b x )+a x b x c x∑ ](ln a − ln b) 2 (ln a +lnb−2lnc) .Sumele se fac după toate permutările circulare ale literelor a, b, c, iar cea de a douasumă sedovedeşte a fi egală cu produsul:(2 ln c − ln a − ln b)(2 ln b − ln a − ln c)(2 ln a − ln b − ln c).Ipotezele asupra numerelor a, b, c arată că ultimii doi factori din acest produs sunt≥ 0, iar primul este ≤ 0. Împreună cufaptulcă:∑(ln b − ln a) 3 (a x − b x ) ≤ 0,pentru x>0 (evident), asta ne arată ceamdorit,adicănearatăcăg (3) (x) ≤ 0pentruorice x>0şi demonstraţia se încheie aici.Extindere şi generalizare dată deIlie Bulacu, Erhardt+Leimer Romania, PTS,Bucureşti.Fie a 1,a 2,...,a n numere reale pozitive astfel încât a 1 ≥ a 2 ≥ ...≥a n şi n ≥ 3.1. Dacă a 2 j ≤ a j−1a n pentru orice j ∈{2,3,...,n−1}, atunci funcţia f :(−∞, 0)→ R ∗ +definită prin:( axf(x)= 1 +a x 2 +...+a x )1xn ,npentru orice x


240 Problemepentru orice x>0, este logaritmic concavă peintervalul(0,∞).Soluţie. Vom rezolva problema în trei paşi.Pasul 1. În primul rând vom arăta că are loc identitatea:(x−y) 2 (x+y−2z)+(y−z) 2 (y+z−2x)+(z−x) 2 (z+x−2y) =(2x−y−z)(2y−z−x)(2z−x−y),∀ x, y, z ∈ R.Pentru stabilirea acestei identităţi notăm:P (x, y, z) =(x−y) 2 (x+y−2z)+(y−z) 2 (y+z−2x)+(z−x) 2 (z+x−2y), ∀x, y, z ∈ R,şi observăm că:şi:( y + z) (P2 ,y,z =PPrin urmare:(P (x, y, z) =aP (x, y, z) =P(y,z, x) =P(z, x, y), ∀x, y, z ∈ Rx, z + x2x− y+z2)() (,z =Py− z+x2x, y, x + y2)(z− x+y2)=0, ∀x, y, z ∈ R.), ∀x, y, z ∈ R,unde a ∈ R şi a =const.. Identificând coeficienţii lui x 3 ,deexemplu,obţinem a 4 =2,adicăa=8.În concluzie:P (x, y, z) =(2x−y−z)(2y − z − x)(2z − x − y), ∀ x, y, z ∈ R.Observaţie. Prin calcul, se oţine forma desfăşurată aluiP(x, y, z):P (x, y, z) =2 ( x 3 +y 3 +z 3) −3 ( x 2 y+xy 2 + y 2 z + yz 2 + z 2 x + zx 2) +12xyz, ∀ x, y, z ∈ R.Pasul 2. În al doilea rând vom arăta că în condiţiile din enunţ au loc următoareleinegalităţi:1. Dacă a 2 j ≤ a j−1a n, pentru orice j ∈{2,3,...,n−1}, atunci:(ln a i − ln a j) 2 (ln a i +lna j −2lna k )+(lna j −ln a k ) 2 (ln a j +lna k −2lna i)++(lna k −ln a i) 2 (ln a k +lna i−2lna j)≥0, ∀i, j, k ∈{1,2,...,n}, i


Soluţiile problemelor propuse 241unde:h(x) =ln ax 1 +a x 2 +...+a x n, x ≠0,neste convexă peintervalul(−∞, 0) şi concavă peintervalul(0,∞).Calculând derivatele g ′ şi g ′′ , avem succesiv:g ′ (x) = xh′ (x) − h(x), x ≠0;x 2g ′′ (x) = x2 [h ′ (x)+xh ′′ (x) − h ′ (x)] − 2x [xh ′ (x) − h(x)]=x 4= x2 h ′′ (x) − 2xh ′ (x)+2h(x), x ≠0.x 3Derivata numărătorului lui g ′′ (x) (care se poate defini şi în punctul x = 0) este egalăcu x 2 h (3) (x), pentru orice x ≠ 0, deoarece:[x 2 h ′′ (x)+2xh ′ (x)+2h(x) ] ′==x 2 h (3) (x)+2xh ′′ (x) − 2xh ′′ (x) − 2h ′ (x)+2h ′ (x)=x 2 h (3) , ∀ x ≠0.În continuare calculăm h (3) (x), oricare ar fi x ≠ 0. Avem succesiv:∑∑a x i ln a ih(x) =ln1≤i≤nna x i, ∀x≠0; h ′ (x)=1≤i≤n∑1≤i≤nunde numărul termenilor de forma a x i ln a i este Cn.1⎛⎞ ⎛ ⎞ ⎛⎝ ∑a x i ln 2 a i⎠ ⎝ ∑ ⎠ − ⎝ ∑h ′′ (x) =1≤i≤n=∑1≤i


242 Problemeunde:După efectuarea calculelor, obţinem:∑a x i a x j (ln a i − ln a j) 3 ( a x j − a x )ih (3) (x) =A =1≤i


Soluţiile problemelor propuse 243Observaţii. 1. Funcţia f poate fi definită şi în x =0prinf(0) = lim f(x) =x→0= n√ a 1a 2·...·a n.h(x)2. Funcţia g poate fi definită şi în x = 0 prin g(0) = lim g(x) = lim =x→0 x→0 x= h ′ (0) = ln n√ a 1a 2 · ...·a n.3. Funcţiile g ′ şi g ′′ nu pot fi definite şi în x = 0 deoarece nu au limităîn x =0.4. Funcţiile h, h ′ , h ′′ şi h (3) pot fi definite şi în x =0prinh(0) = lim h(x) =0,x→0h ′ (0) = lim h ′ (x) =ln n√ ax→01a 2·...·a n,h ′′ (0) = lim h ′′ (x) = 1 ∑(ln ax→0 n 2i − ln a j) 2 ,1≤i


3) 1 α + 1 β + 1 γ = 2 a . Daniel Văcăreţu244 ProblemeAvem:12n∑S n = − π tgkπ ( )tg2n +1 tg (k +1)π π− tg2n+1 2n +1 =k=12n +112n∑(=− π tgkπ)(k +1)π−tgtg2n +1 2n+1 +tg π=2n+1k=12n +1()1ππ=− π tg −tgπ +2ntg =−(2n +1).tg2n +1 2n +12n +1Dezvoltând S n (în formă originală!) se constată că ultimul său termen este 0. Dintrecei 2n−1 termeni nenuli ai sumei, cel mijlociu este tgnπ (n +1)π nπtg2n +1 2n+1 =−tg2 2n +1 ,iarîn ceea ce priveşte ceilalţi termeni nenuli, efectuând reducerile la primul cadran se constatăcă ultimul termen coincide cu primul, penultimul cu cel de al doilea ş.a.m.d. În concluzieavem:n−1∑−(2n +1)=S n =2 tgkπ (k +1)π nπtg2n +1 2n+1 −tg2 2n +1 ≥deci:k=1(n − 1)π≥2tg2n +1 tg nπ nπ2n +1 −tg2 2n +1 ,tgnπ (tgnπ)(n − 1)π−2tg ≥2n+1⇔2n +1 2n +1 2n +1⇔tgnπ((n − 1)ππ−2tg2n +1 2n +1 ≥(2n + 1)tg 2 − nπ )⇔2n +1⇔tgnπ2n +1−2tg(n − 1)π2n +1πtg≥ 2(2n +1)π2(2n +1)· π2 > π 2 .268. Fie cubul [ABCDA ′ B ′ C ′ D ′ ] de muchie a şi punctele X ∈ (AB, Y ∈ (AD şiZ ∈ (AA ′ astfel încât AX = α, AY = β, AZ = γ, unde α, β, γ > a şi:1α + 1 β > 1 a , 1β + 1 γ > 1 a , 1γ + 1 α > 1 a .Planul (XY Z) împarte cubul în două corpuri [C 1] şi [C 2] şi intersectează muchiile(A ′ B ′ ), (BB ′ ), (BC), (CD), (DD ′ ) şi (D ′ A ′ ) în punctele M, N, P , Q, R, S.Să se demonstreze că următoarele afirmaţii sunt echivalente:1) Dreptele MQ, NR şi PS sunt concurente.2) Sferele situate în interiorul corpurilor [C 1] şi [C 2], tangente planului (XY Z) şifeţelor triedrelor tridreptunghice cu vârfurile în A, respectiv C ′ ,aurazeegale.Soluţia autorului. 1)⇔3) Considerăm reperul cu originea în A şi semidreptele(AB, (AD, (AA ′ în calitate de semiaxe Ox, Oy, Oz. Planul(XY Z) are ecuaţia:xα + y β + z γ =1.


Soluţiile problemelor propuse 245Dreapta MQ este intersecţia planelor (A ′ B ′ CD) şi (XY Z), deci ecuaţiile drepteiMQ sunt:{ y + z = aMQ : xα + y β + z γ =1,analog:{ x +y = aNR : xα + y β + z γ =1,{ x+z = aPS : xα + y β + z γ =1.Dreptele MQ, NR şi PS sunt concurente dacă şi numai dacă sistemul format dinecuaţiile lor este compatibil determinat.Aceste sistem este:y + z = a⎧⎪ ⎨ x + y = ax + z = a⎪ x ⎩α + y β + z γ =1.Subsistemul format din primele trei ecuaţii are soluţia x = y = z = a , care introdusă2în a patra ecuaţienedărelaţia:1α + 1 β + 1 γ = 2 a .Deci 1) ⇔ 3).Centrul sferei înscrisăîn triedrul cu vârful în A este punctul Ω 1 (ω 1,ω 1,ω 1)şi distanţade la Ω 1 la planul (XY Z)esteω 1= raza sferei. Avem deci:∣ ∣∣∣∣ ω1α + ω1β + ω1γ − 1 √ 1α + 12 β + 1 = ω 1.2 γ 2Ω 1 şi A fiind de aceeaşi parte a planului (XY Z)avem:( ) (ω1signα + ω1β + ω10γ − 1 = signα + 0 β + 0 )γ − 1 = −1.Rezultă:( 11 − ω 1α + 1 β + 1 ) √1= ω 1γ α + 12 β + 12 γ ⇔ 21⇔ ω 1 =1α + 1 β + 1 √ 1γ + α + 12 β + 1 .2 γ 2Centrul sferei înscrisăîn triedrul cu vârful în C ′ este Ω 2 (ω 2,ω 2,ω 2)şi:d(Ω 2,(XY Z)) = a − ω 2 = raza sferei,adică: ∣ ∣∣∣ ω 2α + ω2β + ω2γ − 1 ∣ ∣∣∣√ 1α 2 + 1 β 2 + 1γ 2 = a − ω 2


246 Problemeşi:Rezultă:( )ω2signα + ω2β + ω2γ − 1 =+1.) √1− 1=aα + 1 2 β + 12( 1ω 2α + 1 β + 1 γ√ 11+aα + 1⇒ω 2 =2 β + 12 γ 21α + 1 β + 1 √ 1γ + α + 1 2 β + 1 ⇒2 γ 2( 1aα + 1 β + 1 −1γ)a−ω 2 =1α + 1 β + 1 √ 1γ + α + 12 β + 1 .2 γ 21R 1 =R 2 ⇔ω 1 =a−ω 2 ⇔1α + 1 β + 1 √ 1γ + α + 12 β + 1 =2 γ 2( 1aα + 1 β + 1 −1γ)=1α + 1 β + 1 √ 1γ + α + 12 β + 1 ⇒ 1 α + 1 β + 1 γ = 2 a .2 γ 2Deci 2) ↔ 3).Observaţie. Condiţia din enunţ:1α + 1 β > 1 a , 1β + 1 γ > 1 a , 1γ + 1 α > 1 a ,a fost pusă pentru a ne asigura că planul(XY Z) intersectează muchiile (A ′ B ′ ), (BB ′ ),(BC), (CD), (DD ′ )şi (D ′ A ′ )în punctele M, N, P , Q, R, S situate în interiorul acestormuchii.γ 2 −ω2 √1α 2 + 1β 2 + 1γ 2 ⇒Soluţie dată de Marian Tetiva, profesor la Colegiul Naţional Gheorghe Roşca-Codreanu din Bârlad. Vom arăta că fiecare din primele două afirmaţii este echivalentăcu a treia. Pentru aceasta vom considera un reper cartezian Oxyz care are originea Oîn punctul A şi pentru care semiaxele pozitive Ox, Oy, Oz sunt respectiv,semidreptele(AB, (AD şi (AA ′ . În acest reper vârfurile cubului au cordonatele A(0, 0, 0), B(a, 0, 0),C(a, a, 0), D(0,a,0), A ′ (0, 0,a), B ′ (a, 0,a), C ′ (a, a, a) şi D ′ (0,a,a); iar punctele X, Y ,Z au coordonatele (α, 0, 0), (0,β,0), respectiv (0, 0,γ), deci ecuaţia planului (XY Z) estexα +y β +z γ =1.1)⇔3). Punctul M reprezintă intersecţia( (planului (XY Z) cudreaptaA ′ B ′ , caracterizatăde ecuaţiile y =0şi z = a, deci M α 1 − a ) ), 0,a . Remarcăm că M aparţineγsegmentului (A ′ B ′ ) datorită condiţiilor din enunţ γ>aşi 1 γ + 1 α > 1 a .Similar obţinem:( (N a, 0,γ 1− a )) ( (,P a, β 1 − a ) ) ( (, 0 ,Q α 1− a ) ),a,0 ,ααβ( (R 0,a,γ 1− a )) ( (şi S 0,β 1− a ) ),a .βγ


Soluţiile problemelor propuse 247Rezultă că un punct oarecare al dreptei MQ are coordonate de forma:( ((1 − u)α 1 − a ) (+ uα 1 − a )),ua,(1 − u)a ,γβpentru un anumit u ∈ R. Analog, punctele dreptelor NR şi PS au coordonate de forma:(((1 − v)a, va, (1 − v)γ 1 − a ) (+ vγ 1 − a )),αβrespectiv:(((1 − w)a, (1 − w)β 1 − a ) (+ wβ 1 − a ) ),wa ,αγpentru anumiţi v, w ∈ R.Să presupunem că dreptele MQ, NR şi PS sunt concurente: aceasta revine laexistenţa unor numere reale u, v şi w pentru care coordonatele celor trei puncte genericede mai sus sunt respectiv egale. Aceasta conduce în primul rând la egalităţile (1 − v)a ==(1−w)a,ua = va şi (1 − u)a = va, care obligă lau=v=w= 1 . Egalând atunci2abscisele primelor două puncte obţinem:1(22 α − a γ − a )= 1 β 2 a ⇒ a α + a β + a γ =2,adică egalitatea de la punctul 3).( Reciproc, dacă această egalitate are loc, se verifică fără probleme că punctula2 , b 2 2), c aparţine tuturor celor trei drepte (coordonatele sale se obţin luând pe fiecaredintre u, v, w egal cu 1 şi folosind egalitatea menţionată). Sevededinceledemaisus2că, dacă dreptele MQ, NR şi PS sunt concurente, atunci ele neapărat se intersectează încentrul cubului.Să ne ocupăm acum de echivalenţa afirmaţiilor 2) şi 3). Sferele despre care este vorbala punctul 2) sunt, de fapt, sferele înscrise în tetraedrele AXY Z şi C ′ X ′ Y ′ Z ′ , unde puncteleX ′ , Y ′ , Z ′ reprezintă intersecţiile planului (XY Z) cu dreptele C ′ D ′ , C ′ B ′ , respectiv CC ′ .Un calcul simplu arată cărazarasfereiînscrise în tetradrul (tridreptunghic) AXY Zeste:r = 3V S = αβγαβ + αγ + βγ + √ α 2 β 2 + α 2 γ 2 + β 2 γ 2(αβ + αγ + βγ + √ )α 2 β 2 + α 2 γ 2 + β 2 γ 2(V = αβγ şi S =reprezintă volumul şi aria62totală pentru acest tetraeddru). Desigur, r ′ ,razasfereiînscrise în C ′ X ′ Y ′ Z ′ ,vafidatădeaceeaşi formulăîn care α, β, γ se înlocuiesc cu α ′ = ( C ′ ( X ′ , β ′ = C ′ Y ′ , respectiv γ ′ = C ′ Z ′ .Punctul X ′ , de exemplu, are coordonatele α 1 − a β − a ) ),a,a (careseobţinγluând în considerare că X ′ aparţine planului (XY Z)şi dreptei C ′ D ′ ). Atunci se calculeazăimediat:(α ′ = a − α 1 − a β − a )= kα,γunde:k = a α + a β + a γ − 1.Analog calculăm β ′ = kβ şi γ ′ = kγ; aplicând formula de mai sus pentru r ′ găsimatunci că r ′ = kγ.Astfel vedem că:


248 Istoria Matematiciişi soluţia problemei se încheie aici.r ′ = r ⇔ k =1⇔ a α +a β +a γ =2⇔ 1 α + 1 β +1 γ = 2 aISTORIA MATEMATICIIDupă osutădeani,laValeaCălugărească(Societatea ,,Gazeta Matematică“ la centenar)Mircea Trifu 1)Acum o sută de ani la Valea Călugărească. S-au petrecut, iată, o sută deani de când a avut loc, la via lui Ion Ionescu de la Valea Călugărească 2) şedinţa redactorilorGazetei Matematice. La capătul acelei memorabile zile de 31 august 1909 s-a luathotărârea definitivă de constituire a Societăţii ,,Gazeta Matematică“, precursoarea de dreptaSocietăţii de Ştiinţe Matematice de azi. Pe pagina a doua a copertei numărului 1, anulXV al Gazetei, număr apărut la 15 septembrie 1909 a fost publicată următoarea notălămuritoare: ,,Cu începere de la 1 septembrie 1909 Redacţia Gazetei Matematice a fosttransformată în Societate, pentru îndeplinirea unor forme legale. Principiile care ne-aucălăuzit 14 ani în publicarea revistei rămân însă aceleaşi şi pe viitor.“Din cei 21 de membri ai societăţii nou înfiinţate, 12 erau ingineri şi 9 profesori.Menţionăm aici inginerii: Tancred Constantinescu, Vasile Cristescu, Andrei Ioachimescu,Ion Ionescu, Mihail Roco (fondatori ai Gazetei Matematice) şi profesorii următori: NicolaeAbramescu, Anton Davidoglu, Traian Lalescu, Dimitrie Pompeiu, Gheorghe Ţiţeica.Argumente pentru înfiinţarea Societăţii. Motivaţiile care au stat la bazaînfiinţării Societăţii au fost mai multe, legate, de exemplu, de soluţionarea unor posibileconflicte cu persoane din afara Redacţiei, de punerea la adăpost a puţinului capital strânspentru tipărirea revistei faţă deoricefeldeîmprejurări nedorite sau de creştere a credibilităţiiîn faţa unor persoane dispuse să facă donaţii şi cărora organizarea ,,patriarhală“nu le inspira suficientăîncredere.A existat însă şi un incident de altă natură, care poate fi ataşat la motivaţiile de maisus. În anul 1904 salariile funcţionarilor publici s-au redus drastic, cotizaţiile la Gazetă seîncasau greu. Unul dintre redactori – profesor de matematică, nu i s-a păstrat numele –a propus – nici mai mult – nici mai puţin – desfiinţarea revistei şi împărţirea capitaluluistrâns, ba chiar şi a cărţilor din bibliotecă.A fost pusă în circulaţie o Declaraţie, care, semnată de o parte dintre redactori,a fost transmisă luiIon Ionescu, pe atunci casier, însărcinat şi cu administrarea revistei.Acesta, consfătuindu-se cu A. Ioachimescu, A. Davidoglu şi Gh. Ţiţeica,carenusemnaserăDeclaraţia, a convocat adunarea redactorilor la Societatea Politehnică, unde declaraţiei dedesfiinţare i-au opus declaraţia celor patru, de continuare. Ion Ionescu, în calitate decasier, avea răspunderea faţă deabonaţi de a le transmite revista până lasârşitul anuluişi nu putea umbla la capitalul existent până la acea dată. Pe de altă parte, o lichidareîn părţi egale nu era echitabilă, redactorii neavând aceeaşi vechime la Gazetă. RedactorulCaton Erbiceanu amărturisit că semnătura i-a fost luată cu rea credinţă, cel care i-a cerutoasigurându-l că toţi membrii redacţiei sunt de acord cu lichidarea. Redactorul Tancred1) Profesor, Secretar general, S.S.M.R2) Via lui Ion Ionescu este, probabil, o moştenire de familie, de la mama sa, Maria-AtinaIonescu, născută Diamandescu, fiica unui podgorean de la Valea Călugărească. Via a fostvândutăîn 1938, când Ion Ionescu a cumpărat casa din strada Răsuri din Bucureşti.(N.A.)


M. Trifu, După osutădeani,laValeaCălugărească 249Constantinescu a confirmat, revoltat, că şi el a păţit acelaşi lucru. Pentru a se reveni lacondiţii normale de lucru, la 10 iulie 1904, a fost redactată oCirculară, în care se arăta,printre altele, că ,,...este natural că nimeni nu este dator să facă un sacrificiu pentru carenu are tragere de inimă, dar nu nici nu trebuie ca acesta să formeze o piedică pentru ceicare vor să meargăînainte. Gazeta trebuie să-şi continue apariţiunea în acelaşi mod ca şipână acum“. Cum nu toţi redactorii au trimis răspuns, la 20 august este dată oadouacirculară, cu ton aproape ultimativ: ,,. . . În cazul în care nu vom primi răspuns până la7septemvrie a.c. vă vom considera retras din Comitetul de Redacţie al Gazetei.“Unele răspunsuri s-au păstrat: ,,. . . Chiar dacă s-ar dizolva actuala funcţiune pentrupublicarea Gazetei Matematice, voi face tot ce mi-ar sta în putinţă ca,fără nicioîntârzieresă se înjghebeze o alta“ (Ion Ionescu); ,,Vă rog să mă consideraţi redactor al GazeteiMatematice atâta vreme cât voi fi profesor de matematică“ (Vasile Teodoreanu).Gazeta Matematică a continuat să apară ...Înfiinţarea unei societăţi nu este o treabă uşoară. Pentru obţinerea recunoaşteriilegale a Societăţii, la şedinţa de la Valea Călugărească a fost desemnată o comisie,frmată dinV. Cristescu, A. Ioachimescu, I. Ionescu, T. Lalescu, I. D. Teodor şi G. Ţiţeica,pentru elaborarea statutelor. Forma definitivă a acestora conţine 31 de articole, grupateîn 12 Titluri, ca de exemplu: Art. 2. Scopul Societăţii este răspândirea gustului pentrustudiul ştiinţelor matematice şi îndrumarea cercetări originale relative la această ramurădeştiinţă; Art. 7. Nu există decât o categorie de membri: membri activi.După definitivarea statutelor, pentru recunoaşterea Societăţii ca persoană morală(juridică) a fost alcătuit următorul proiect de lege:Art. Unic. Se recunoaşte Societăţii Gazeta Matematică din Bucureşti calitateade persoană morală în baza statutelor aci alăturate şi care nu se pot modifica decât cuautorizaţiunea Consiliuui de Miniştri.Acesta a fost depus, din iniţiativa parlamentară, de către deputatul de Vlaşca, NicolaeBălănescu, în şedinţa Camerei din 27 martie 910, când s-a cerut şi s-a admis urgenţa.Raportor al legii a fost G. C. Dragu. Legea a fost votatăîn Camera deputaţilor la 5 aprilie1910, cu 61 bile albe şi 3 bile negre.La Senat legea a trecut în toamna aceluiaş an, la 27 noiembrie, cu 45 bile albe şi obilă neagră, raportor fiind G. Cnstantinescu-Romniceanu.Regele Carol I a promulgat legea prin Decretul 3798 din 16 decembrie 1910, publicatîn Monitorul Oficial o săptămână maitârziu.Luând cuvântul în Senat, în favoarea legii, Spiru Haret aarătat că această Societate,,...este o fondaţie ...ştiinţifică dezinteresată, care, nu numai că nu are fonduri, dar, dincontră ,dădinpungaeipentruasusţine Gazeta Matematică. A contribuit mai mult decâtorice altă instituţiune pentru dezvoltarea şi întărirea învăţământului matematic.“Comentând rezultatul votului din cele două camere legiuitoare, Ion Ionescu observa,cu bonomă ironie, că ,,...rezultatele sunt foarte surprinzătoare, dacă negândim câtă lumeeste certată cu matematica încă depetimpulcând o învăţau ca şcolari !“Societatea Gazeta Matematică era, potrivit statutelor, administrată de o Delegaţie,compusă din doi membri, aleşi pe o perioad a de doi ani şi fără drept de realegere imediatăşi un casier, ales pe perioadă de cinci ani, cu drept de realegere. Ceilalţi membri au primitsarcini pe probleme: delegat pentru aritmetică, delegat pentru geometrie, delegat pentrucercetarea notelor şi articolelor etc. Delegat pentru rapoarte a fost numit G. Ţiţeica, ofuncţie în care a stat până lasfârşitul vieţii.Administraţia şi redacţia au rămas, în continuare, în odăiţa din strada Manea Brutarunr. 14 (azi General C. Budişteanu), acolo unde, la 14 octombrie 1894 cinci ingineri(iniţiatorii): Victor Balaban, Vasile Cristescu, Ion Ionescu, Mihail Roco şi Ioan Zottu, aupropus înfiinţarea unei reviste de matematică ,,...de care să profite elevii liceelor noastre“.


250 Istoria MatematiciiRevista s-a numit, de atunci şi până astăzi, Gazeta Matematică. Laadresaamintităseaflau birourile centrale ale Serviciului pentru Construcţia liniei ferate Feteşti-Cernavodă,de sub conducerea inginerului Anghel Saligny, iar cei cinci ingineri, fiecare în vârstă cupuţin peste 25 de ani, erau angajaţi acolo.După promulgarea Legii persoanelor juridice, în februarie 1924, statutul Societăţiia fost revizuit. S-au schimbat, printre altele, normele de fixare a cotizaţiei şi a taxelorde admitere în Societate, a fost modificat anul financiar(de la an şcolar, cum era pânăatunci, la an calendaristic). Potrivit noii legi, Societatea a fost pusă sub tutela MinisteruluiInstrucţiunii Cultelor şi Artelor, care a şi aprobat, în anul 1932, noul statut, aprobare,,întărită“ ulterior în tribunal.Calea Griviţei nr. 158: Casa Gazetei Matematice. Încă de prin anul 1920se preconiza construcţia unui local al Gazetei Matematice, redactorul N. Nicolescu donândprimii 500 de lei în acest scop. În anul 1923, Traian Lalescu propune lui Tancred Constantinescu,membru fondator al Gazetei, ajuns Director General al Căilor Ferate, să donezeo parcelă dintr-un lot aflat lângă Gara de Nord, pentru construcţia localului. În aprilieanul următor, regele Ferdinand sancţionează următoarea lege: Articol Unic. Se ratificăînaltul decret regal nr. 3714 din 20 iulie 1923, prin care direcţiunea generală C.F.R. CasaMuncii este autorizată a ceda în mod gratuit Societăţii Gazeta Matematică parcela din CaleaGriviţei Nr. 158-60 în suprafaţă de 250 mp pentru a-şi construi local, care să serveascădrept sediu al Societăţii. .... Dat în Bucureşti, la 4 aprilie 1924, Ferdinand.De la solemmnitatea punerii pietrei fundmentale a localului,,Casa Gazeta Matematică”, 27 octombrie 1933.Legea a fost votată de Adunarea deputaţilor la 29 februarie 1924 în unanimitate cu98 de voturi, iar în Senat, la 10 martie 1924, cu 68 de voturi pentru, contra 9.La fondurile strânse până atunci se adaugă 1 milion de lei daţi de Mihail Manoilescu,subsecretar de stat la Finanţe (participant, ca elev, în patru ani consecutivi, la ConcursurileGazetei Matematice) din fondurile culturale ale ministerului. Suma obţinută, la care seadaugă dobânzile aferente, a fost suficientă pentru a construi parterul şi primul etaj alblocului din Calea Griviţei nr. 158 (azi 144), bloc care se va numi ,,Casa Gazeta Matematică“.Inginerul Aurel Ioanovici, membrualSocietăţii,se oferă săfacă lucrarea în cost,adăugând mobilier, lămpi, perdele . ,,. . . Dintre toate problemele propuse în Gazeta Matematicănu a fost niciuna mai grea, mai frumoasă şi mai interesantă, decât problema Casei


M. Trifu, După osutădeani,laValeaCălugărească 251Gazetei Matematice“, va remarca Gh. Ţiţeica la inaugurare, în ziua de 27 ianuarie 1935.Cu construcţia Casei perioada ,,nomadă“ a Gazetei şi, implicit, a Societăţii, iasfârşit. Aicise vor ţine de acum toate şedinţele bilunare ale redacţiei,aicisevorstrânge plicurile cuarticole şi probleme.Marele Jubileu din anul 1935. Semicentenarul Gazetei Matematice. Anul1935 este anul primului mare jubileu al Gazetei Matematice: 40 de ani de la apariţiaprimului număr. Apare volumul omagial Gazeta Matematică – 1895 – 1935. Istoric,Învăţăminte, în care redactorii: Gh. Ţiţeica, Ion Ionescu, Andrei Ioachimescu, CristeaMateescu, Gheorghe Buicliu, Ovidiu Ţino, Grigore Zapan fac o amplă şi exactă radiografiea ,,fenomenului Gazeta Matematică“, relevând contribuţia revistei la succesul pe care l-aavut reforma lui Haret, cu secţia reală a liceului. Gheorghe Ţiţeica face prima judecată devaloare asupra Gazetei, apreciind căşcoala matemticăromânească este produsul mai multorfactori: înfiinţarea celor două unversităţi, la Iaşi (1860) şi la Bucureşti (1864), apariţiaprimilor doctori în matemaatică cu teze susţinute la Sorbona (Spiru Haret, ConstantinGogu, David Emmanuel şi apariţia celor două reviste, Recreaţii ştiinţifice la Iaşi şi GazetaMatematică la Bucureşti. La şedinţa festivă, ţinutăîn marele amfiteatru al Politehnicii, aluat parte şi regele Carol al II-lea, careafăcut urarea ca ,,. . . cei 40 de ani împliniţi ...săse împlinească încă de40deori!“În anul 1945, într-o ţară pustiită derăzboi, în condiţiiaustere,estesărbătorit semicentenarulGazetei Matematice. Este publicat volumul ,,Gazeta Matematică (Decada a5-a). Istoric - Învăţăminte“, se bat 502 medalii de argint şi bronz, se emit 400 000 deserii de mărci poştale dedicate evenimentului. Cu prilejul Jubileului Gazetei, aînceput laBucureşti şi s-a sfârşit la Cluj, cel de al 3-lea Congres al Matematicienilor Români, la careau participat 220 de persoane din 30 de oraşe ale ţării. O delegaţie a participanţilor laCongres este primită laCasa Gazetei Matematice din Calea Griviţei, unde li se oferă chiaro gustare. Ion Ionescu, grav bolnav, în imposibilitate de a participa la festivităţi, primeşteacasă, în strada Răsuri, o delegaţie a Societăţii, careîi înmânează medalia jubiliară. . . . Înincinta Institutului de Statistică a fost deschisă Expoziţia Cărţii Româneşti de Matematică.Pe mesele şi pe rafturile Expoziţiei se puteau vedea cele 50 de volume ale Gazetei, dar şiAritmetica lui Şincai sau Calculul diferenţial al lui Culianu, colecţia completă a tezelor dedoctorat susţinute de matematicienii români, cărţi de autori români tipărite în străinătate,cărţi de şcoală, extrase din reviste străine.Într-un raport asupra mersului Societăţii pe anul 1945, prezentat în Adunarea generalădin 25 februarie 1946, aflăm despre dificultăţile financiare aferente devalorizării monedeinaţionale, despre costurile ridicate de tipărire a revistei, dar şi de sprijinul materialacordat Societăţii de către autorităţi, unităţi şcolare, persoane fizice, membri ai Societăţii.Astfel, sunt menţionaţi că au donat 5000 lei N. Abramescu, Valeriu Alaci, N. N. Mihăileanuşi A. Popescu-Zorica. Un grup de elevi din Turnu-Severin au donat 87000 petru tipărirearevistei. În acelaşi scop, donează Liceul din Râmnicu-Sărat suma de 70000 lei, MinisterulEducaţiei Naţionale (ministru Ştefan Voitec) suma de 1000000 lei, Gimnaziul Oituz dinTg. Ocna, suma de 20000 lei.Se sfârşeşte o lume, se naşte o lume. Anul 1949 avea să fieunandemarifrământări pentru Societatea Gazeta Matematică. La 15 mai, cu punctualitatea deja cunoscută,a apărut numărul 9 al revistei Celelalte numere, 10, 11, 12 deşi au fost plătite, n-aumai apărut niciodată. Nu s-a dat nicio explicaţie în legătură cu acest fapt, nemaiîntâlnitpână atunci. Mai mult, a început evacuarea localului din Calea Griviţei, mai mulţi trecătoriocazionali putând observa cum pachetele cu cărţi din biblioteca Societăţii (peste 3400 detitluri) erau aruncate de la etajul întâi diret într-un camion ce staţiona în faţa localului.Constantin Ionescu-Ţiu, custodele bibliotecii şi administratorul localului, a fost nevoit să-şi


252 Istoria Matematiciicaute o altă locuinţă, iar Grigore Zapan, casierul Societăţii, a fost obligat să retragă delaImprimeria Naţională ,în stadiul în care se aflau, toate lucrările care urmau să apară.Cărţile din bibliotecă s-auîmprăştiat ca fumul, nu se ştie unde, Casa Gazetei Matematicea fost preluată de I.A.L. fără temei legal şi înţesată de chiriaşi (aşa a rămaspână astăzi, cu deosebirea că statul a permis chiriaşilor să cumpere apartamentele în carestăteau!).Se petreceau lucruri teribile în toată societatea românească, evenimentele se derulauîn secret, se instaura încet-încet atmosfera de suspiciune, de teamă. Au loc procese politiceîn care sunt ,,demascaţi“ duşmanii poporului sau cei care s-au făcut vinovaţi de dezastrulţării, de crime de război şi de înaltă trădare, prin antrenarea României în război alăturide Germania hitleristă. Un val nemaivăzut de arestări loveşte grav Academia Română,Biserica, Armata, iar ,,noua politică“, cu vremea, se va întinde peste tot. Membri marcanţiai Socităţii Gazeta Matematică care au fost, într-un fel sau altul, implicaţi în politică, suntarestaţi şi dispar în închisori.Inginerul Tancred Constantinescu, absolvent–şef de promoţie (1895) al Şcolii Naţionalede Poduri şi Şosele, singurul în viaţă dintre cei zece fondatori ai Gazetei Matematice(el a donat în anul 1927 din partea C.F.R., terenul pe care s-a construit Casa Gazeta Matematică)a fost senator liberal şi ministru al Industriei şi Comerţului în guvernul Brătianuîn perioada 1923-1926. Este arestat în noaptea de 5/6 mai 1950 , numită maitârziu –,,noaptea demnitarilor“ – şi moare în închisoarea de la Sighet la 14 mai 1951.Inginerul Mihail Manoilescu, absolvent–şef de promoţie (1915) al Şcolii Naţionalede Poduri şi Şosele, membru al Societăţii din anul 1926 (a participat, ca elev, la patru ediţiiale Concursului Gazetei Matematice), în perioada când a fost subsecretar de stat la Finanţe(1926-1929) a donat, din fondurile culturale, suma de 1000000 lei pentru construcţia CaseiGazeta Matematică. Creator de doctrină economică – cartea sa ,,Teoria protecţionismuluişi a schimbului internaţional“ din 1929 este, după C. Murgescu, ,,...oprimăstrăpungereromânească în gândirea economică universală“. A fost ministru al Lucrărilor publice şiComunicaţiilor în guvernul Maniu (1930), Guvernator al Băncii Naţionale (1931), ministrual Afacerilor Străine în guvernul Gigurtu (1940), când va semna, ca împuternicit al statuluiromân, nedreptul Dictat de la Viena. Este arestat în anul 1944 şi reţinut, fără proces unan de zile. În anul 1948 este arestat din nou şi internat la Jilava, Ocnele Mari şi Sighet,unde moare în anul 1950. Apoi – caz, probabil unic în analele justiţiei din ţările civilizate– este judecat în absenţă (!) la 12 aprilie 1952 pentru articole profasciste şi condamnat la15 ani închisoare şi confiscarea totală a averii. Familiei i se va comunica moartea sa abiaîn 1958 !Profesorul universitar Constantin Buşilă, membrualSocietăţii încă din anul 1910,absolvent – şef de promoţie (1910) al Şcolii Naţionale de Poduri şi Şosele, mare energetician,a ajuns prorector al Politehnicii şi decan al Facultăţii de Electromecanică. În perioada 1941-1943 a fost ministru al Lucrărilor publice şi Comunicaţiilor în guvernul Antonescu. În anul1946 Tribunalul Poporului îl condamnă la10aniînchisoare. Moare în penitenciarul dinAiud la 3 februarie 1950.Generalul de divizie Gheorghe Potopeanu, membrualSocietăţii Gazeta Matematicădin anul 1930, a fost ministru al Economiei Naţionale (1941), guvernator al Transnistriei(din 26 ianuarie 1944), ministru de finanţe în guvernul Sănătescu (23 august 1944-13 octombrie1944). A fost arestat în octombrie 1944 şi reţinut trei luni de zile fără proces. Înanul 1948 este rearestat şi condamnat la 5 ani de închisoare pentru crime împotriva păcii,la Jilava şi Aiud. După ispăşirea pedepsei este arestat din nou în 1957, condamnat la 10ani de închisoare pentru spionaj şi înaltă trădare şi încarcerat la Jilava, Piteşti şi Dej. Esteeliberat prin graţiere în 1966. Moare în acelaşi an.


M. Trifu, După osutădeani,laValeaCălugărească 253Familiile şi-au plâns în tăcere morţii, îngropaţi, uneori în morminte neştiute, fărăcruci.Şi apoi s-a lăsat, cu nedreptate şi vinovăţie, uitarea ....Viaţa mergea înainte.În Gazeta Învăţământului din 15 iulie 1949 s-a putut citi că: ,,... s-a produs,de curând, un eveniment care va avea efecte de mare însemnătate în ridicarea niveluluiştiinţific din ţară. Este vorba de înfiinţarea Societăţii Ştiinţelor Matematice şi Fizice dinR.P.R.“. Fără alteamănunte. În luna august a aceluiaş anaapărut Gazeta Matematicăşi Fizică – revistă pentru studiul şi răspândirea ştiinţelor matematice şi fizice, Anul I (vol.55), Serie nouă a Gazetei Matematice fondată în 1895, nr. 1, iunie 1949. Era clar de cenu mai apăruseră numerele din iunie-iulie-august ale vechii Gazete Matematice !În acest prim număr al noii reviste, în articolul ,,Cuvânt înainte“, după o facilă şilaconică apreciere pozitivă laadresaGazetei Matematice, autorul, ascuns în umbra anonimatului(articolul este semnat de ... secretariatul de redacţie al Gazetei Matematice şiFizice) aratăcă ,,... Gazeta Matematică, ca şi revistele facultăţilor de ştiinţe, au prezentatîncă delaînceput o serie de lipsuri fundamentale, moşteniri ale regimului moşierescdin trecut .... Izolarea îndelungată, forţată aştiinţei româneşti de ştiinţa sovietică, ceamai înaintată din lume, a ridicat obstacole esenţiale în calea creaţiei ştiinţifice ... GazetaMatematică a acordat problemei învăţământului o atenţie cu totul redusă ... În total, GazetaMatematică nu mai corespunde democraţiei populare din R. P. R. în drum spre socialism.După cum nu corespund nici formele organizatorice ale fostelor societăţi de matematicăşi fizică. În consecinţă, în iunie 1949 a fost formată Societatea Ştiinţelor Matematice şiFizice din R. P. R. Totodată, Gazeta Matematică setransformăîn Gazeta Matematică şiFizică, devenind organ al Societăţii de Ştiinţe Matematice şi Fizice din R.P.R.“Articolul-program continuă cu definirea scopului Gazetei, ca organ al noii Societăţişi se încheie cu îndemnul mobilizator: ,,Înainte, dar, la muncă luminată, pentru o ştiinţăîn slujba poporului!“Defăimarea fără noimăavechiiGazete Matematice ar fi meritat o ripostă pemăsură.Dar cine avea curajul să ofacă, în iureşul ,,revoluţionar“ al acelor ani ?Pe ultimele pagini ale primului număr al Gazetei Matematice şi Fizice este publicatăcomponenţa nominală a Consiliului Central Provizoriu şi al Biroului Societăţii ŞtiinţelorMatematice şi Fizice în R.P.R., stabilite la şedinţa de constituire din 30 mai 1949.Din cei 39 de membri ai Consiliului, 16 au fost membri ai Societăţii Gazeta Matematică,ceilalţi provenid de la secţiile de matematică şi de fizică ale Societăţii Române deŞtiinţe, defiinţatăîn primăvara lui 1949 de către Prezidiul Academiei R. P. R.Nu a existat un decret al Marii Adunări Naţionale, noul organ legisltiv al ţării, caresă abroge Decretul Regal 3798 din 1910, de aceea concluzionăm că, în anul 1949 SocietateaGazeta Matematică s-a transformat în Societatea de Ştiinţe Matematice şi Fizice din R.P. R., la aceasta afiliindu-se şi secţiile de matematică şi de fizică ale Societăţii Române deŞtiinţe.Actul constitutiv al noii societăţi, însoţit de lista participanţilor la şedinţa din 30 mai1949, este înaintat Tribunalului Ilfov, Secţia a III-a, pentru legiferare şi pentru dobândireapersonalităţii juridice. Aceasta este acordată, începând cu data de 3 decembrie 1949,sub controlul Ministerului Învăţământului şi al Ministerului de Interne. Cu acelaşi prilej,Tribunalul Ilfov, Secţia Notariat, certifică cu nr 12497/1949, Statutul S.S.M.F din R.P.R.aprobat la adunarea generală din 30 mai 1949.În mod normal, o astfel de transformare ar fi presupus preluarea de către nouasocietate (S.S.M.F) a activului şi pasivului vechii societăţi şi a portofoliului de lucrări(articole, note, probleme propuse etc.). Nimic din toate acestea nu s-a petrecut. GrigoreZapan, casierul Societăţii Gazeta Matematică, a fost nevoit să facă rost de sumele necesare


254 Istoria Matematiciiachitării datoriilor faţă de tipografie. Corespondenţii care au primit confirmarea reţineriimaterialelor trimise, au aşteptat în zadar, publicarea acestor. Rezolvitorii de probleme auaşteptat degeba să afle rezultatele strădaniilor lor. Noua societate nu a mai folosit vechiulsediu din Calea Griviţei – acesta căpătase, deja, altă destinaţie – ci a peregrinat, în calitatede chiriaş, pe la diferite adrese: la 1 iulie 1949, str. Episcopiei nr. 2, etaj 5, de la 1 ianuarie1950, Bd. 6 Martie nr. 63, de la 8 iulie 1952, str. Dionisie Lupu nr. 40, de la 1 iunie 1954,str. Spiru Haret nr. 12, de la 1 februarie 1955 şi pânăîn prezent, str. Academiei nr. 14.A fost neglijat tot ce amintea de trecut, se voia ca totul să înceapă de la zero, pebaze noi.S-a trecut cu vederea, chiar în actele constitutive, că S.S.M.F. reprezintă ocontinuarefirească aSocietăţii Gazeta Matematică, de aceea nu s-a făcut referire la baza materială decare dispune aceasta din urmă.Afostreţinută numai vechimea, de peste jumătate de secol, a Gazetei Matematice.Renunţarea ,,la trecut“ – sau, la o parte din el – s-a prelungit şi mai târziu. Astfel,în lucrarea ,,Societăţi şi Asociaţii în ştiinţa românescă“ editată deAsociaţia Oamenilor deŞtiinţă din R. S. România, Bucureşti, 1981, se arată că ,,...în 1949, secţiile de matematicăşi fizică ale Societăţii de Ştiinţe din România s-au constituit în Societatea de ŞiinţeMatematice şi Fizice, care s-a scindat în 1964 în două secţii (Ord.M.I. nr. 577 din 29 VI1964), lărgindu-şii cadrul de activitate prin atragerea unui număr mai mare de profesoridin învăţământul gimnazial şi liceal “. Nicio vorbă despreSocietatea Gazeta Matematică!Abia la Conferinţa Naţională a S.S.M. din 1995, în preambului Statutului S.S.M.adoptat atunci, se prevedea că: ,,...Societatea de Ştiinţe Matematice sub diferitele denumiripurtate de-a lungul anilor (Societatea Gazeta Matematică, Societatea de ŞiinţeMatematice şi Fizice, Societatea de Şiinţe Matematice) a fiinţat în temeiul statutului SocietăţiiGazeta Matematică din 1910, cu modificările aduse în 1932, al statutului Societăţiide Şiinţe Matematice şi Fizice, adoptat în anul 1949 şi al statutului Societăţii de ŞiinţeMatematice, adoptat în anul 1964, cu modificările aduse în 1975.“Cinci ani mai târziu, în şedinţa publică din 3 octombrie 2000, Judecătoria Sectorului1 Bucureşti ,,. . . în numele legii, hotărăşte: Admite cererea formulată de petenta, Societateade Ştiinţe Matematice din România, cu sediul în Bucureşti, str. Academiei nr. 14, Sector1. . . . Constatăm că petenta ...este succesoarea în drepturi a Societăţii de Matematică şiFizică din R.P.R., care a succedat , la rândul său. Societăţii Gazeta Matematică“.Acum, la început de mileniu, Societatea privea spre viitor ...cu întreg trecutul!La Valea Călugărească, după osutădeani. S-a întâmplat ca 25 septembrie2009 să fie o frumoasă zi de toamnă când, Biroul S.S.M.R. a convocat la Valea Călugăreascăreprezentanţii filialelor Societăţii,într-o şedinţă aniversară, pentru a marca începutul manifestărilordedicate Centenarului Societăţii Gazeta Matematică. Auparticipatşi numeroşiinvitaţi. Au luat cuvântul, pentru scurte alocuţiuni aniversare, acad. Marius Iosifescu,vicepreşedinte al Academiei Române, prof. univ. Ioan Tomescu, membru corespondent alAcademiei Române, preşedinte de onoare al S.S.M.R., conf. univ. dr. Augustin Mitu, secretarde stat in Ministerul Educaţiei, Cercetării şi Inovării, ing. Mircea Cosma, preşedinteleConsiliului judeţean Prahova, prof. Nicolae Angelescu, preşedintele Filialei Prahova aS.S.M.R şi inspector general la I.Ş.J. Prahova, organizatorul de facto al întâlnirii, prof.univ. dr. Nicolae Victor Zamfir, preşedintele Societăţii RomânedeFizică, prof. univ. dr.Doru Ştefănescu, primvicepreşedinte al S.S.M.R., conf. univ. dr. ing. Alexandru Popa dela Universitatea de Petrol şi Gaze din Ploieşti (care a evocat personalitatea lui Ion Ionescu),prof. Mircea Trifu, secretar general al S.S.M.R. Sarcina de moderator a fost susţinută deprof. univ. dr. Radu Gologan, preşedintele S.S.M.R. Au mai prezentat cuvântări personaleşi alocuţiuni prof. Al. Popescu-Zorica, decanul de vârstă al S.S.M.R., fost membru al vechiiSocietăţi Gazeta Matematică, prof. univ. dr. Miron Oprea şi prof. Olimpia Popescu.


M. Trifu, După osutădeani,laValeaCălugărească 255Din intervenţiile participanţilor la dezbatere s-au conturat câteva măsuri şi activităţiconcrete pentru anul 2010: şedinţă aniversară în Aula Academiei Române, orgnizarea,în România, a şedinţei anuale a Comitetului Societăţii Europene de Matematică (EMS),dezvelirea unui bust al lui Ion Ionescu la Valea Călugăreascăşiauneiplăci comemorative pecasa din strada General Budişteanu nr. 14-16 din Bucureşti, o expoziţie de carte româneascăde matematică, manifestări aniversare la nivelul tuturor filialelor Societăţii.La banchetul organizat la Valea Călugărească, cu acest prilej, gazdele au oferit, după,,pohta“ matematicienilor ,,un dram din gustul bucatelor munteneşti, din locul slobozirii


256 Istoria MatematiciiSocietăţii de Matematică acumunveac“. Lista-meniu, alcătuită după modelul din 1909,şi, evident. ,,actualizată“ în conţinut, a fost oferită fiecărui participant, ca aducere aminte.Facem câteva ,,citări“:,,ŢuicădeVăleni – cu limpezimea unei axiome şi tăria oricărui început de teoriematematică, gustălău (gustare) – condiţie necesară potrivit raţionamentului matematic, caşde cap (tobă), lebăr, şorici, răcituri (aspic), castravete cu urdăşi mărar, ceapă de apă, bitoc(chifteluţe speciale), roşii umplute cu vinete“. După cafea, s-a continuat cu ,,. . . sărmăluţe(integrate în cuib de foaie de varză şi derivate din foaie de viţă), mămăligă vârtoasă (caodemonstraţie riguroasă) ciuşcă ofgroscior(smântână) pentru redactarea finală şi cu unprodus direct şi tensorial (după caz)de cârnăciori la jar: ceafă de purcel, piept de puiotăncit, cârn˘ciori călugăreşti, toate acestea stropite cu vin de Valea Călugărească cugustde celebră teoremă“. Desertul a constat dintr-o ,,. . . delicioasă plăcintă cumere(ca bucuriaunui articol acceptat de Gazeta Matematică)“.Motiv ca discuţiile să se prelungească (chiardacă nu prin continuitate), până târziu.Desfăşurată într-o atmosferă deînţelegere şi bună dispoziţie, întâlnirea de la ValeaCălugărească a fost bine apreciată de participanţi.Aspecte din istoria O.I.M.Constantin Rusu 1) şi Neculai Stanciu 2)La al IV-lea Congres al Matematicienilor Români în anul 1956, se propune organizareaunei olimpiade internaţionale de matematică.În anul 1959, România organizează prima Olimpiadă Internaţională de Matematică(OIM) – competiţie anuală de matematică la care participă elevi din diferite ţări (atunciau fost prezente 7 ţări).Matematica este domeniul în care tinerii români s-au remarcat constant pe planinternaţional. De la lansarea OIM, în 1959, singurul participant care a reuşit o lucrareperfectă(realizarea punctajului maxim – 42 puncte) la fiecare dintre cele trei ediţii (1995,1996, 1997) la care a luat parte este un român, Ciprian Manolescu. Ciprian Manolescu esteabsolvent summa cum laudae în matematică la Harvard, iar în 2004 şi-a încheiat doctoratulla aceeaşi universitate. Acum este Associate Professor la renumita universitate americanăUCLA.Câte trei medalii de aur, chiar dacă nu adjudecate cu scor maxim, le-au revenit,de-a lungul timpului, altor trei liceeni români: Mihai Manea (1999, 2000, 2001), ŞtefanLaurenţiu Horneţ (1997, 1998, 1999) şi Teodor Bănică (1989, 1990, 1991).Alţi români care au obţinut punctaje maxime sunt: Daniel Tătaru (1984, 1985),Adrian Vasiu (1987, 1988), Nicuşor Dan (1987, 1988), Andrei Moroianu (1987, 1989),Mugurel Barcău (1987), Liviu Suciu (1987), Sergiu Moroianu (1991), Dragoş Nicolae Oprea(1995) şi Ovidiu Savin (1995).În afară de cei menţionaţi mai sus medalii de aur au mai obţinut: Dan Voiculescu(1966, 1967), Ana Caraiani (2002, 2003), Adrian Ioan Zahariuc (2005, 2006), AdrianBogdan Ungureanu (2005, 2006), Victor Nistor (1978, 1979), Constantin Chişcanu (1995,1996), Geffry Barad (1992, 1993), Andrei Neguţ (2004), Florin Belgun (1990), Radu HoriaMihăescu (1998), Andrei Dan Ionescu (1991), Louis Funar (1984), Marius Beceanu(1999), Adrian Ocneanu (1974), Şerban Nacu (1992), Dragoş IoanMichnea(2005), GabrielKreindler (2005), Livia Alexandra Ilie (2007), Cezar Gheorghe (1960), Adrian Corduneanu(1996), Barbu Rudolf Berceanu (1968), Mugurel Barcău (1987), Radu Negulescu (1985),Bogdan Enescu (1978), László Zsidó(1963), Ştefan Radu Niculescu (1996), Virgil Petrea1) Profesor, Liceul ,,Ştefan cel Mare“, Râmnicu Sărat2) Profesor, Şc. gen. ,,George Emil Palade“ c si Şc. gen. nr. 6, Buzău


C. Rusu & N. Stanciu, Aspecte din istoria O.I.M. 257(2002), Basarab Nicolescu (1959), Mircea Martin (1972), Flaviu Iepure (1998), Nicolae Beli(1986), Răzvan Gelca (1985), Marius Dabija (1986), Eduard Gabriel Băzăvan (2006), ElenaMădălina Perşu (2009)şi Omer Cerrahoglu (2009).Doar doi participanţi la OIM au patru medalii de aur: germanul Christian Reiher(2000, 2001, 2002, 2003) şi americanul Reid Barton (1998, 1999, 2000, 2001).Oaltă performanţă, neegalată până acum la OIM, o realizează echipajul american înanul 1994. Toţi cei şase reprezentanţi obţin punctajul maxim. Antrenorul echipei olimpicede matematică a Statelor Unite, la Olimpiada Internaţională de Matematică din 1994, dela Hong Kong, a fost profesorul român Titu Andreescu.În ceea ce priveşte ţara organizatoare a OIM, România ocupă primul loc (1959, 1960,1969, 1978, 1999).Cea mai importantă distincţie din lumea matematicii – Medalia Fields (din 1936),este cunoscută ca un fel de Premiu Nobel pentru matematică şi recompensează realizărilemajore din matematică.Ea se acordăîn cadrul Congresului Internaţional de Matematică (ICM). Tot la acestcongres se mai acordă Premiul Nevanlinna (din 1982) şi Premiul Gauss , pentru aplicaţiilepractice ale matematicii (din 2006). Începând cu 2010, alături de aceste trei premii se vaacorda şi Medalia Cern, pentru realizări în matematică pe parcursul întregii vieţi.Medalia Fields Premiul PremiulRolf Nevanlinna Carl Friedrich GaussSpre deosebire de Premiul Nobel, care se acordă anual, Medalia Fields este atribuităla fiecare 4 ani. De menţionat că medaliaţii Fields trebuie să aibăovârstă maimicăde40de ani. Printre aceştia sunt şi următorii olimpici internaţionali:Grigori Margulis – medaliat cu argint pentru U.R.S.S. în 1962. A primit MedaliaFields în 1978.Vladimir Drinfel’d – medaliat cu aur pentru U.R.S.S. în 1969. A primit MedaliaFields în 1990.Jean-Christof Yoccoz – medaliat cu aur pentru Franţa în 1974. A primit MedaliaFields în 1994.Richard Borcherds – medaliat cu argint pentru Anglia (UK) în 1977. A primitMedalia Fields în 1998.Timothy Gowers – medaliat cu aur pentru U.K. în 1981. A primit Medalia Fields în1998.Terence Tao – medaliat cu bronz în 1986, cu argint în 1987 şi cu aur în 1988. Aprimit Medalia Fields în 2006.La ultimul Congres Internaţional de Matematică, organizat în Spania, la Madrid,în 2006, un alt olimpic internaţional, Grigori Perelman, a fost invitat să ridice MedaliaFields. Talentat la matematică, urmează cursurile unui prestigios liceu leningrădean renumitpentru specializarea sa în fizică şi matematică. Rezultatele sale nu se lasă aşteptateşi, în 1982, devine membru al lotului sovietic pentru Olimpiada Internaţională de Matematicăşi obţine medalia de aur cu scorul maxim posibil: 42 de puncte. A refuzat MedaliaFields(2006) şi premiul de 1.000.000 de dolari oferit de Clay Mathematics Institute, pentru


258 Istoria Matematiciirezolvarea Conjecturii lui Poincaré, una din cele şapte probleme ale mileniului. La Princeton,unde olimpicul nostru internaţional Daniel Tătaru a efectuat studiile postdoctorale,lucrează englezul Andrew Wiles (a demonstrat marea teoremă aluiFermat – după 357 deani de la enunţare) şi a lucrat începînd din 1993 şi rusul Grigori Perelman.Viitorul Congres Internaţional de Matematică, cel din 2010, va avea loc în India, înoraşul Hyderabad.În încheiere, felicităm echipa României participantă la a 50-a ediţie a OIM de lBremen, Germania (la care au participat un număr record de ţări – 104) pentru rezultatulobţinut.Aceasta a fost formată din următorii concurenţi:• Elena Mădălina Perşu (vârsta – 18 ani) – argint 2007, argint 2008, aur 2009;• Andrei Deneanu (vârsta – 19 ani) – argint 2009;• Francisc Bozdan (vârsta – 19 ani) – argint 2009;• Tudor Pădurariu (vârsta – 17 ani) – bronz 2009;• Marius Tiba (vârsta – 16 ani) – bronz 2009;• Omer Cerrahoglu (vârsta – 14 ani) – aur 2009;şi condusă de:•Prof.univ.dr. Radu Gologan (Leader);• Prof. Mihail Bălună (Deputy leader);• Prof. Dan Schwarz (Observer A);• Prof. Cristian Alexandrescu (Observer B).Bibliografie[1] Şt. George Andonie, Istoria Matematicii în România, vol.I,Ed.Ştiinţifică, 1965.[2] F. Diac, Monografia S.S.M.R., Bucureşti, 1998, Colecţia Biblioteca S.S.M.R.[3] M.Trifu, Fenomenul Gazeta Matematică la 110 ani, G.M.-B nr. 9/2005 (Număr jubiliar).[4] *** Gazeta Matematică, 1895-1935, Istorie-învăţăminte (Volum jubiliar)[5] *** Gazeta Matematică, nr. 9/1995 (Număr jubiliar).[6] *** Colecţia Gazeta Matematică, 1895-2009,http://www.gazetamatematica.net/ http://www.imo-official.org/Centenar pentru matematicienii români – 2009Marioara Costăchescu 1)Vom trece în revistă, câteva aspecte esenţiale din viaţa şi activitatea a doi matematicieniromâni, născuţi la 1909.George TheilerGeorge Theiler, s-a născut la Bârlad, la 3 august 1909. Studiile primare şi liceul şile-a făcut în oraşul natal. În toamna anului 1928 s-a înscris la Universitatea din Bucureşti,Facultatea de ştiinţe, de unde în 1931 a obţinut licenţa în matematici. În perioada 1931-1949 a lucrat ca actuar şi expert actuar, la ,,Societatea generală de asigurări“, unde aavut şi funcţii importante. În anul 1952 a fost numit lector la catedra de matematici aInstitutului de Petrol şi Gaze din Bucureşti, unde a funcţionat pânăîn 1961. În octombrie1960 şi-a trecut doctoratul în ştiinţele matematice, cu teza: ,,Contribuţii la teoria statisticiineparametrice. Probleme de tip Kolmogorov-Smirnov; Manya Kvit; Renyi.“ Din 1962 a fostnumit conferenţiar de analiză matematică la Institutul Pedagogic de 3 ani din Bucureşti. Pe1) Profesor, Liceul cu program sportiv, Roman, jud. Neamţ


M. Costăchescu, Centenar pentru matematicieni români 259bază deconcurs,în 1963, a fost numit şef de sector la statistica matematică la Institutul dematematică al Academiei Române. Când în aprilie 1964 s-a înfiinţat Centrul de statisticăal Academiei, a trecut ca şef de secţie la teoria probabilităţilor de la acest centru.Urmărind etapele din viaţa acestui matematician, observăm că l-a preocupat cuprecădere Teoria probabilităţilor şi Statistica matematică, domenii în care a publicat şidiverse lucrări. Cu totul întâmplător a atacat şi alte domenii matematice. Amintim aici:a) funcţii egale aproape peste tot, integrabile Riemann;b) rezolvarea sistemelor de ecuaţii diferenţiale liniare şi omogene, în cazul cândecuaţia caracteristică are rădăcini multiple;c) a dat o demonstraţie directă pentru teorema lui Kronecker din teoria funcţiiloraproape periodice.În Statistica Matematică are, în special contribuţii la Teoria Statisticii Neparametrice:a dat un model probabilistic pentru unele probleme de statistică neparametrică;a studiat probleme de statistică neparametrică bidimensionale; a examinat criteriile luiKolmogorov şi Smirnov din teoria statisticii neparametrice; a studiat problema distribuţieiasimptotice comune pentru două cantităţi corespunzând unui şir variaţional de experienţereferitoare la o variabilă aleatoare.În Teoria probabilităţilor, folosind o teoremă aluiOnicescu şi Mihoc despre lanţuricu legături complete, simple şi staţionare, cu un număr finit de stări, George Theiler aarătat că aceasta este valabilă şi în cazul când se renunţă laatreiacondiţie pusă. A trecutapoi la determinarea efectivă a claselor de lanţuri cu legături complete cărora li se aplicăaceastă teoremă ergodică.A avut colaborări cu Gh. Mihoc şi Dumitru Firescu, iarlucrările lui au apărut înAnalele Universităţii Bucureşti, în Comunicările Academiei Române şi în reviste ca: Studiişi cercetări matematice sau Revista de statistică.Dumitru FirescuDumitru Firescu s-a născut la 10 decembrie 1909, în localitatea Bârza, din fostuljudeţ Dolj. Liceul l-a urmat la Craiova, luându-şi bacalaureatul în 1928. A intrat imediatla Facultatea de ştiinţe a Universităţii din Bucureşti, secţia matematici, de unde a obţinutlicenţa în matematici în 1931.În perioada 1932-1933 a urmat Şcoala de statistică ce era condusă de OctavOnicescu, de unde a obţinut diploma de actuar, pe care a folosit-o între anii 1933-1948,îndeplinind funcţia de actuar la Societatea de asigurări ,,Naţionala“, din Bucureşti. Simultana funcţionat tot în Bucureşti ca profesor la cursurile serale ale unei şcoli comerciale.Apoi, în perioada 1948-1958, a funcţionat în învăţământul mediu din Bucureşti. Anul 1958a marcat trecerea doctoratului la Universitatea din Bucureşti, la Facultatea de matematicăşi fizică cu teza, având tema: ,,Problema eficienţei funcţiilor de estimaţie pentru probabilităţilefundamentale, directe şi inverse, ale lanţurilor Markov omogene de ordin finit.“În toamna anului 1958 a fost numit lector la catedra de matematici aplicate a profesoruluiMihoc; la această catedră a fost avansat conferenţiar în 1961. După decesul lui ArnoKahane, în 1965, Dumitru Firescu arămas şeful catedrei de matematici de la Facultateade Chimie din cadrul Universităţii din Bucureşti. Din 1964 funcţionează şi la Centrul destatistică al Academiei Române, la secţia de aplicaţii ale statisticii matematice în economie,biologie, medicină şi agricultură.Publicaţiile lui Dumitru Firescu sunt din domeniul Teoriei probabilităţilor şi Statisticiimatematice. S-a ocupat în special de funcţii de estimaţie eficiente pentru probabilităţide trecere ale lanţurilor Markov sau, în colaborare cu Gh. Mihoc, de procese stochasticeîntâlnite în demografie ori de generalizarea proceselor stochastice.În primul memoriu publicat s-a ocupat de extinderea unor rezultate stabilite deGh. Mihoc în 1957 privind determinarea funcţiilor de estimaţie pentru probabilităţile de


260 Din viaţa societăţiitrecere ale unui lanţ Markov, discontinuu, simplu şi omogen; a considerat cazul când dintretoate probabilităţile de trecere, elemente ale unei matrici stochastice, numai unele suntnecunoscute şi a arătat, dând o serie de teoreme, că funcţiile de estimaţie sunt numai corecteşi gausiene; ele devin eficiente numai în cazul studiat de Mihoc. Pentru probabilităţilede trecere inverse ale unui lanţ Markov simplu, omogen, de ordin finit, Dumitru Firescua determinat funcţiile de estimaţie şi a studiat proprietatea de eficienţă şi normalitateasimptotică a acestor funcţii. Pentru cazul unui lanţ Markov simplu, care ia valori pe axareală, D. Firescu a determinat, în colaborare cu Gh. Mihoc, condiţiile necesare şi suficientede îndeplinit de densităţile de probabilitate, pentru ca funcţiile de estimaţie să fieeficiente.Considerând procesele stochastice de naştere şi stingere, generalizare a procesuluistochastic dat de fenomenul mortalităţii şi preocupat de generalizarea acestor procese prinintervenţia postacţiunii (ca în fenomenul invalidităţii), D. Firescu a studiat evoluţia probabilisticăa unui proces de naştere în care intervine o astfel de postacţiune; a calculatprobabilităţile care determină evoluţia ulterioară a procesului. Studiind familii de lanţuriMarkov discrete staţionare cu probabilităţi de trecere strict pozitive, a determinat condiţiilenecesare şi suficiente de îndeplinit de probabilităţi, ca funcţiile de estimaţie să fieeficiente.În urma colaborării cu doctorul în medicină P. Tăutu, a elaborat un model statistic alhematopoezei (procesul de formare a celulelor sanguine în măduva osoasă).A mai colaborat, în realizarea lucrărilor sale, cu G. Theiler şi D. Negoiu, careauapărut în: Analele Universităţii Bucureşti, Comunicările Academiei Române şi în revisteca: Studii şi cercetări matematice sau Revista de statistică.Bibliografie[1] G. Şt. Andonie, Istoria Matematicii în România, vol.3,EdituraŞtiinţifică, Bucureşti,1967.[2] S.Coatu, Mică enciclopedie matematică, Editura Tehnică, Bucureşti, 1980.[3] I. Purcaru şi O. Bâsca, Oameni, idei, fapte din Istoria Matematicii, Editura Economică,Bucureşti, 1996.DIN VIAŢA SOCIETĂŢIIŞcoala de vară delaBuşteniA XIII-a ediţie a Cursurilor Şcolii de vară pentru perfecţionarea profesorilor dinînvăţământul preuniversitar s-a desfăşurat, după tradiţie, la Buşteni, în perioada 27 iulie-6 august 2009. După cum se ştie, aceste cursuri organizate de S.S.M.R., au deja oîndelungată tradiţie, ele debutând la sfârşitul anilor ’50 ai secolului trecut (la Săcele) şicontinuând, cu o scurtăîntrerupere în anii ’90, pânăîn zilele noastre.Gazda cursurilor a fost, ca de obicei în ultimii ani, Centrul de Pregătire pentruPersonalul din Industrie, care ne-a asigurat – prin persoana domnului director general IrinelGhiţă şi a subordonaţilor săi – condiţii deosebite, atât în privinţa desfăşurării propriu-zise acursurilor, cât şi în privinţa cazării participanţilor; trebuie menţionat, de asemenea, totalarenovare şi modernizare a grupului alimentar, acesta fiind acum unul din cele mai elegantedin staţiune. Tuturor celor menţionaţi mai înainteleadresăm mulţumirile noastre pentrusprijinul acordat în organizare.Conform regulamentului stabilit de mai mulţi ani, programul zilnic inclusiv sâmbătă–acuprinstreiconferinţe, pentru a acoperi un număr de circa 40 de ore în intervalul 28 iulie-5august, în care s-au desfăşurat cursurile propriu-zise. Spre deosebire de anii precedenţi, la


Şcoala de vară delaBuşteni 261finele cursurilor nu s-a mai desfăşurat un colocviu, acesta fiind înlocuit printr-o sesiune decomunicări şi referate, prezentate atât de cursanţi, cât şi de alte persoane care s-au arătatinteresate. 1) La încheierea cursurilor, toţi participanţii au primit o diplomă caresăatesteabsolvirea lor.Faţă de anul precedent s-a înregistrat o scădere uşoară anumărului de participanţi(49 faţă de 54); trebuie să menţionăm aici faptul că la cursuri nu pot participa mai multde circa 60 de persoane datorită capacităţii sălii de conferinţe.La deschiderea cursurilor au rostit scurte alocuţiuni prof. univ. dr. Radu Gologan –preşedinteleS.S.M.R.–şi prof. Nicolae Angelescu – inspector general la I. Ş. J. Prahova,preşedintele filialei din Ploieşti a S. S. M. R. Trebuie să menţionăm, de asemenea, prezenţa–laînchiderea cursurilor – a domnului Emanoil Savin, primarul oraşului Buşteni, care ne-apromis, pe viitor, sprijinul domniei sale în organizarea acestora.Repartiţia zonală a participanţilor a fost destul de uniformă, menţinându-se, totuşi,pe primul loc judeţele din Moldova şi dintre acestea, judeţul Iaşi unde se remarcă, din nou,activitatea neobosită a profesorului Vasile Nechita – secretarul filialei locale.Absolvenţii cursurilor Şcolii de vară delaBuşteni, ediţia 2009, împreună cu profesorii:Eugen Păltănea, Dumitru Buşneag, Cătălin Gherghe, Ioan Tomescu şi Dan RaduCursurile au stârnit un viu interes printre participanţi, marcat pe de-o parte dediscuţiile dintre aceştia şi conferenţiari, iar pe de altă parte de sondajul efectuat la finele lor.De altfel, aceste cursuri au constituit totdeauna un teren fertil pentru schimbarea opiniilorîntre cursanţi şi între aceştia şi conferenţiaripetemeprivindînvăţământul matematicromânesc şi viitorul acestuia.Tematica cursurilor a fost atent selectată, ţinând seama de sugestiile făcute în aniianteriori în cadrul sondajelor de opinie. Lucrul a fost posibil şi prin cooptarea de noiconfernţiari, din generaţiile mai tinere, care s-au arătat interesaţi în prezentarea unorconferinţe axate pe probleme ştiinţifice şi metodice moderne. În măsura posibilităţilor,am căutat să păstrăm un echilibru între subiectele cu caracter de informare ştiinţifică şicele vizând metodica şi metodologia predării la clasă, primele având, evident, un caracter1) În legătură cu această sesiune se va vedea materialul următor inserat în prezentulnumăr al revistei. (N.A.)


262 Din viaţa societăţiipreponderent. Selectarea conferenţiarilor a fost făcută cudeosebităgrijă, fiind solicitaţi,cu precădere, acei universitari, care, în decursul timpului, s-au aplecat cu interes şi seriozitateasupra învăţământului preuniversitar, fiind preocupaţi de perpetua îmbunătăţireşi diversificare a acestuia; nu în ultimul rând, am ţinut seama şi de simpatiile cursanţilorexprimate în sondajele din anii anteriori. Conferenţiarii au izbutit să stabilească unmodde comunicare simplu şi eficient cu profesorii cursanţi, fără aabuzadeinformaţia ştiinţificăşi, ceea ce ni se pare esenţial, neadoptând o poziţie ex cathedra.Vom prezenta, mai jos, tematica conferinţelor susţinute.• prof. univ. dr. Ioan Tomescu (Universitatea din Bucureşti, membru corespondental Academiei Române), ,,Aspecte metodice privind predarea noţiunilor de combinatoricăînliceu“; ,,Poliedre convexe şi grafuri planare, fulerene şi lema lui Sperner“; ,,Blocuri design,triplete Steiner, grafuri bipartite şi implicaţii asupra problemelor de olimpiadă“.• prof. univ. dr. Constantin Popovici (Universitatea din Bucureşti), ,,Întregi pătratici“;,,Unicitatea descompunerii în factori primi ai întregilor pătratici“.• prof. univ. dr. Radu Gologan (Universitatea Politehnica din Bucureşti), ,,OlimpiadaInternaţională de Matematică – Bremen 2009“.• prof. univ. dr. Doru Ştefănescu (Universitatea din Bucureşti),,,Decenuneestefrică de polinoame“; ,,Cum cultivăm Istoria Matematicii?“.• prof. univ. dr. Dumitru Buşneag (Universitatea din Craiova), ,,Elemente de teoriacategoriilor în matematica preuniversitară“.• prof. univ. dr. Valeriu Guţu (Universitatea de Stat din Republica Moldova), ,,Dece tabela valorilor funcţiilor trigonometrice este aşa de mică?“; ,,Bilete cu noroc şi funcţiigeneratoare“.• conf. univ. dr. Dragoş Popescu (Universitatea din Bucureşti), ,,Teoria cuplajelorşi aplicaţii (I)“; ,,Teoria cuplajelor şi aplicaţii (II)“.• conf. univ. dr. Alexandru Gica (Universitatea din Bucureşti), ,,Probleme de ordinîn baraje şi la O.I.M.“; ,,Analiza matematică şi teoria numerelor. Probleme“.• conf. univ. dr. Cătălin Gherghe (Universitatea din Bucureşti), ,,Metode de teoriacodurilor în probleme de tip O.I.M.“; ,,Aria figurilor poligonale“;• conf. univ. dr. Eugen Păltănea (Universitatea Transilvania din Braşov), ,,Extinderiale unor criterii clasice de convergenţă“.• conf. univ. dr. Andrei Vernescu (Universitatea Valahia din Târgovişte), ,,Exempleşi contraexemple în analiza matematică liceală“; ,,Câteva rezultate şi formule completarereferitoare la integrala Riemann a funcţiilor continue“.Ca şi în anii precedenţi vom aminti prezenţa la cursuri a multora dintre ,,veterani“,acei profesori care constituie un adevărat nucleu al cursanţilor şi al căror lider necontestateste profesorul Vasile Nechita, participant la toate ediţiile de până acum. De asemenea, vommenţiona faptul că amacordatonouă diplomă de fidelitate (pentru participarea la şapteediţii consecutive a cursurilor) doamnei profesoare Maria Mihăeş de la Colegiul NaţionalDanubiana din Roman. O felicităm şi pe această cale, atât pe domnia sa, cât şi pe toţiceilalţi ,,veterani“ ai cursurilor.La încheierea acestor succinte însemnări trebuie să adresăm mulţumirile noastre sinceredomnului profesor Nicolae Angelescu – inspector general la I. Ş. J. Prahova şi preşedinteal filialei Ploieşti a S.S.M.R. şi doamnei profesoare Mirela Dobrea – directoare agrupului şcolar Ioan Kalinderu din Buşteni – pentru sprijinul neprecupeţit acordat în bunaorganizare şi desfăşurare a cursurilor . Acestora şi multor altora, anonimi, le exprimămgratitudinea noastră.Dan Radu


Şcoala de vară delaBuşteni 263Lista absolvenţilor cursurilor de perfecţionare pentruprofesorii de matematică, organizate de S.S.M.R.la Buşteni, în perioada 27 iulie-6 august 20081. Alexa Lenuţa Gr. Şc. Gh. Asachi – Galaţi2. Badea Aurelia Liliana Lic. Teoretic Dimitrie Bolintineanu – Bucureşti3. Bejan Cornelia Livia Universitatea Tehnică Gheorghe Asachi – Iaşi4. Burtea Anne-Marie Gr. Şc. – Oraş Ovidiu5. Cavachi Clarisa Ioana C. N. Bilingv Gh. Coşbuc – Bucureşti6. Câlţea Elena Colegiul Tehnic de Arhitectură şi Lucrări PubliceI. N. Socolescu – Bucureşti7. Chirea Elena Lic. Teoretic Nicolae Bălcescu – Medgidia8. Chiriac Gilena Gr. Şc. T. Vuia – Craiova9. Cojocea Tatiana Şc. cu clasele I-VIII nr. 108 Alexandru Obregia–Bucureşti10. Costăchescu Mărioara Lic. cu Program Sportiv – Roman11. Creţu Ciprian Col. Tehnic Gheorghe Asachi – Iaşi12. Dragomir Luminiţa C.N. C. Negri – Galaţi13. Frenţ Angela Col. Tehnic Feroviar – Braşov14. Gavriluţ Mihai C. N. Roman Vodă – Roman15. Georgescu Mioara Ş. nr.26–Craiova16. Iosub Maria C. N. Ştefan cel Mare – Tg. Neamţ17. Kifor Roxana Gr. Şc. Ind. Gheorghe Asachi – Bucureşti18. Luchian Dorel Lic Tehn. M. Costin – Iaşi19. Macovei Monica Felicia Colegiul Tehnic Petru Muşat – Suceava20. Maftei Maria Şc. de Arte şi Meserii – Dulceşti21. Marinescu Damian Şc. cu clasele I-VIII Tudor Vladimirescu – Târgovişte22. Mihai Marcela Gr. Şc. Ind. Gheorghe Asachi – Bucureşti23. Mihăeş Maria Colegiul Tehnic Danubiana – Roman24. Mihăilă Adriana Lic. Teoretic N. Iorga – Brăila25. Murar Aurelia Lic. cu Prog. Sportiv Banatul – Timişoara26. Müller Gina Lic. Ec. C. C. Kiriţescu – Bucureşti27. Nechita Vasile Colegiul Costache Negruzzi – Iaşi28. Necula Elena Gr. Şc. Forestier – Câmpina29. Negrea Ana Maria Gr. Şc. Traian Vuia – Târgu Mureş30. Negrea Nicodim Lic. Tehnic Traian – Deva31. Nica Paula Gr. Şc. Ind. Gheorghe Asachi – Bucureşti32. Niţu Ştefan Lic. Teoretic Al I. Cuza – Alexandria33. Nour Maria Şc. gimnazială nr. 41 Sf. Grigore Teologul - Galaţi34. Novac Iulian Şc.gen.A.Mureşan - Deva35. Oleniuc Claudia Gr. Şc. Virgil Madgearu – Iaşi36. Oleniuc Mariana Şc. cu clasele I-VIII Blăgeşti – Paşcani37. Pălici Aurelia Col. Naţional Octav Onicescu – Bucureşti38. Păuna Neculae Şc.gen.Coresi–Târgovişte39. Pâslaru Mihaela Lăcrămioara Şc. M. Eminescu – Roman40. Popa Filofteia Şc. cu clasele I-VIII nr. 12 – Târgovişte41. Popa Gabriel Col. Naţ. – Iaşi42. Popescu Maria Şcoala Centrală – Bucureşti43. Radu Margareta Şc. nr. 3 – Botoşani


264 Din viaţa societăţii44. Radu Marin Şc.gen.nr.3–Botoşani45. Roman Neculai Şc. cu cl. I-VIII Vasile Alecsandri–Mirceşti,46. Sbiera Elena Oltiţa Colegiul Tehnic Al. I. Cuza – Suceava47. Topan Daniela Adina Colegiul Tehnic de Transporturi Transilvania –Cluj-Napoca48. Tulcianu Ana-Maria Gr. Şc. D. Cantemir – Babadag49. Ţilică Daniela Gr. Şc. Ind. Gheorghe Asachi – Bucureşti50. Vega Mariana Şc. gen. nr. 13 Mircea cel Bătrân – Piteşti51. Voicu Adriana Lic. Ped. Matei Basarab – SloboziaPrima sesiune de comunicări şi referate metodico-ştiinţificeorganizată de S.S.M.R cu ocaziaŞcolii de vară delaBuşteniÎn urma unei hotărâri a conducerii S.S.M.R. şi conducerii Cursurilor de vară delaBuşteni, începând cu acest an, colocviul care marca finalul cursurilor a fost înlocuit cu osesiune de comunicări şi referate la care cursanţii se pot înscrie benevol. De asemenea, lasesiune pot participa şi alte persoane interesate care au trimis din timp comunicarea saureferatul cu care vor să participe.Afostalcătuit un comitet ştiinţific care a avut menirea de a selecta dintre lucrăriacelea care prezintă un interes deosebit şi care au fost susţinute în plen. 1) Anul acestacomitetul a fost compus din următorii:• Dumitru Buşneag (Universitatea din Craiova)• Cătălin Gherghe (Universitatea din Bucureşti)• Radu Gologan (Universitatea Politehnică din Bucureşti, Institutul de MatematicăSimion Stoilov al Academiei Române)• Eugen Păltănea(Universitatea Trabsilvania din Braşov)• Dan Radu (Universitatea Politehnica din Bucureşti)• Doru Ştefănescu (Universitatea din Bucureşti)• Ioan Tomescu (Universitatea din Bucureşti, membru corespondent al AcademieiRomâne).De asemenea, a fost format un comitet de organizare care s-a ocupat de bunadesfăşurare a lucrărilor. Din acesta au făcut parte:• Cătălin Gherghe (Universitatea din Bucureşti)• Radu Gologan (Universitatea Politehnică din Bucureşti, Institutul de MatematicăSimion Stoilov al Academiei Române)• Dan Radu (Universitatea Politehnica din Bucureşti)• Doru Ştefănescu (Universitatea din Bucureşti, prim-vicepreşedinte al S.S.M.R.)• Ioan Tomescu (Universitatea din Bucureşti, membru corespondent al AcademieiRomâne, preşedinte de onoare al S.S.M.R.).Lucrările sesiunii s-au desfăşurat în data de 5 august, după încheierea cursurilor.Vom mai menţiona că la sesiune a mai participat şi un invitat din străinătate în persoanadomnului Irineu Glajar de la Austin Community College din Austin, Texas, S.U.A.Subiectele comunicărilor şi referatelor au acoperit o arie largă de preocupări începândcu cele cu caracter ştiinţific sau metodic şi terminând cu cele care vizează teme din istoria1) Aceste lucrări au fost menţionate în lista de comunicări şi referate prezentată maijosprintr-un asterisc. (N.A.)


Şcoala de vară delaBuşteni 265matematicii, dar şi cele legate de problematica generală aînvăţământului preuniversitar.Diversitatea tematicii abordate reflectă lărgirea ariei de preocupări a profesorilor, tendinţaacestora de a se ancora mai ferm în realitate, preluând tradiţiile înaintaşilor. Vom remarca,în mod deosebit, seriozitatea şi competenţa cu care autorii au tratat tematica aleasă –desigur, în mare măsură clasică – foarte mulţi dintre ei vizând deschideri către alte domeniiale învăţământului sau ale vieţiidezicuzi,precumşi conexiuni metodice interesante şi cuun vădit caracter original.Vom insera, mai jos, lista completă alucrărilor înscrise în sesiune.(1) Maria Margareta Agapi –Şcoala George Călinescu, Oneşti, jud. Bacău –,,Zâmbetul ştiinţei“(2) Lenuţa Alexa – Grupul Şcolar Industrial Gheorghe Asachi, Galaţi, jud. Galaţi– ,,Asupra demonstraţiei teoremei lui Pitagora numai cu rigla negradată şi compasul“(3) Aurelia Badea – Liceul Teoretic Dimitrie Bolintineanu, Bucureşti – ,,Ion Barbu– poetul matematician“(4) Cornelia Bejan – Universitatea Tehnică Gheorghe Asachi, Iaşi, jud. Iaşi –,,Structuri algebrice definite geometric“(5) Ane-Marie Burtea – Grupul Şcolar, Ovidiu, jud. Constanţa – ,,Convexitate şimajorizare“(6) Lucia Buruiană –Şcoala cu clasele I-VIII Cezar Petrescu, Buşteni, jud. Prahova– ,,Numere prime, numere compuse“(7) Petre Buruiană – Grupul Şcolar Ion Kalinderu, Buşteni, jud. Prahova – ,,Inegalităţiîntre elementele unui triunghi“(8) Clarisa Ioana Cavachi – Colegiul Naţional Bilingv George Coşbuc, Bucureşti –,,Integrale ≪surori≫“(9) Elena Chirea – Colegiul Naţional Nicolae Bălcescu, Medgidia, jud. Constanţa –,,Traian Lalescu – un matematician peste ani“ (*)(10) Tatiana Cojocea –Şcoala nr. 108 Alexandru Obregia, Bucureşti – ,,Aspectespecifice ale rezolvării ecuaţiilor“(11) Mărioara Costăchescu – Liceul cu Program Sportiv, Roman, jud. Neamţ –,,Anul 2009 în lumea matematicii“ (*)(12) Ligia Darău – Colegiul Tehnic Dr. A. Bărbat, Victoria, jud. Braşov – ,,Experimentdidactic comparativ pe tema evaluării“(13) Luminiţa Dragomir – Colegiul Naţional Costache Negri, Galaţi, jud. Galaţi –,,Forme, metode şi instrumente de evaluare. Tehnici moderne la disciplina matematică“(14) Ştefan Nicolae Dumitrescu – Colegiul Naţional Ştefan Velovan, Craiova, jud.Dolj – ,,Ecuaţii diofantice de tipul x 2 − 4y 2 =1“(15) Mihai Gavriluţ – Colegiul Naţional Roman Vodă, Roman, jud. Neamţ –,,Generalizareaunor inegalităţi“(16) Mioara Georgescu –Şcoala nr. 26 Craiova, jud. Dolj – ,,Matematica aplicată“(17) Irineu Glajar – Austin Community College, Austin, Texas, S.U.A. – ,,Predareamatematicii la un colegiu de doi ani în S.U.A.“ (*)(18) Maria Iosub – Colegiul Naţional Ştefan cel Mare, Târgu Neamţ, jud. Neamţ –,,Generalizări ale unor probleme din revista Sinus“ (*)(19) Monica Macovei – Colegiul Tehnic Petru Muşat, Suceava, jud. Suceava –,,Omotetia în plan“ (*)(20) Damian Marinescu –Şcoala Tudor Vladimirescu, Târgovişte, jud. Dâmboviţa–,,Construcţia mediei armonice a două segmente“(21) Marcela Mihai – Gr. Şc. Ind. Gheorghe Asachi, Bucureşti – ,,Inegalităţi clasice.Aplicaţii“ (*)


266 Din viaţa societăţii(22) Adriana Mihăilă – Liceul Teoretic Nicolae Iorga, Brăila – ,,Aplicaţii ale inegalităţilorîn rezolvarea de ecuaţii şi sisteme“ (*)(23) Gina Müller – Colegiul Economic C. C. Kiriţescu, Bucureşti – ,,Aplicaţii aleteoremei lui Lagrange“(24) Aurelia Murar – Liceul cu Program Sportiv Banatul, Timişoara, jud. Timiş –,,Strategii didactice eficiente în lecţiile de recapitulare şi sistematizare“(25) Vasile Nechita – Colegiul Costache Negruzzi, Iaşi, jud. Iaşi – ,,Proprietăţi alefuncţiilor convexe. Demonstrarea unor inegalităţi cu ajutorul funcţiilor convexe“(26) Elena Necula – Grupul Şcolar Forestier, Câmpina, jud. Prahova – ,,Frontieradintre matematică şi inteligenţă“(27) Ana-Maria Negrea – Grupul Şcolar Traian Vuia, Târgu Mureş, jud. Mureş –,,Unele aplicaţii ale generalizării teoremei lui Lagrange“ (*)(28) Nicodim Negrea – Liceul Teoretic Traian, Deva, jud. Hunedoara – ,,Calcule desume“(29) Ştefan Niţu – Liceul Teoretic Al. I. Cuza, Alexandria, jud. Teleorman – ,,Vectori.Vectori a căror sumă este vectorul −→ 0 . Aplicaţii.“(30) Maria Nour –Şcoala Gimnazială nr. 41 Sf. Grigore Teologul, Galaţi, jud.Galaţi – ,,Promovarea metodelor activ-participative în însuşirea cunoştinţelor dematematică“(31) Iulian Novac –Şcoala Andrei Mureşanu, Deva, jud. Hunedoara – ,,Despre sistemede numeraţie“(32) Claudia Oleniuc – Grupul Şcolar Virgil Madgearu, Iaşi, jud. Iaşi – ,,Un numeromânesc pe harta lumii: Spiru Haret“(33) Mariana Oleniuc –Şcoala cu clasele I-VIII, Blăgeşti-Paşcani, jud. Iaşi – ,,Ordineşi haos“(34) Aurelia Pălici – Colegiul Naţional Octav Onicescu, Bucureşti – ,,Caracterizareasubgrupurilor aditive ale lui R şi aplicaţii în probleme de analiză matematică“ (*)(35) Nicolae Păuna –Şcoala Coresi, Târgovişte, jud. Dâmboviţa – ,,Explorarea problemelorde loc geometric cu instrumente virtuale create cu CABRI GEOMETRYII“ (*)(36) Mihaela Pîslaru –Şcoala Mihai Eminescu, Roman, jud. Neamţ – ,,Modernizareaconceptului de predare-învăţare prin informatizarea învăţământului (utilizarealaboratoarelor A.E.L. şi a softurilor la orele de matematică).“(37) Filofteia Popa –Şcoala nr. 12, Târgovişte, jud. Dâmboviţa – ,,Metode activparticipativefolosite în predarea matematicii“(38) Maria Popescu –Şcoala Centrală, Bucureşti – ,,Eleganţa raţionamentelor matematice“(*)(39) Neculai Roman –Şcoala Vasile Alecsandri, Mirceşti, jud. Iaşi – ,,În legătură cuproblemele C.O: 4932 şi C.O: 4953“ (*)(40) Elena Oltiţa Sbiera – Colegiul Tehnic Al. I. Cuza, Suceava, jud. Suceava –,,Inversiunea în plan“ (*)(41) Daniela Adina Topan – Colegiul Tehnic de Transporturi Transilvania, ClujNapoca, jud. Cluj – ,,Aspecte privind punctul intermediar în teorema de medie“(42) Daniela Ţilică – Grupul Şcolar Industrial Gheorghe Asachi, Bucureşti – ,,Şirurirecurente“(43) Mariana Vega –Şcoala nr. 13, Piteşti, jud. Argeş – ,,Istoria matematicii înlecţia curentă“(44) Adriana Voicu – Liceul Pedagogic Matei Basarab, Slobozia, jud. Ialomiţa –,,Conciziune şi eficienţăîn rezolvarea problemelor de matematică“


Recenzii 267Lucrările sesiunii s-au bucurat de un deosebit succes, multe dintre comunicărileprezentate stârnind vii discuţii şi aprecieri, comentarii şi completări. Având în vedereopinia pozitivă unanimă, vom perpetua această iniţiativă şi în anii următori.Dan RaduRECENZIITITU ANDREESCU, GABRIEL DOSPINESCU,Problems from the Book, XYZ Press, LLC, 2008Parafrazând un text celebru bazat pe problemele considerate dumnezeieşti de PaulErdös (Proof from the Book, autori M. Aijner şi G. M. Ziegler, Springer Verlag, 2003)autorii au adunat într-un text extrem de generos (peste 550 de pagini) 23 de lecţii dematematică problemistică, nu neapărat elementară, bazate pe afinităi¸le lor matematice.Fiecare astfel de lecţie conţine un preambul intitulat ,,Teorie şi exemple“ în care sunttrecute în revistă principiile legate de subiectul discutat şi câteva probleme cu soluţii completece exemplifică titlul paragrafului. A doua parte a paragrafelor ,,Proleme de antrenament“conţine între 15 şi 25 de probleme lăsate cititorului ca exerciţiu, evident caracteristicedomeniului discutat.Adeseori exemplele prezentate sunt total neelementare şi chiar legate de tehnici noide cercetare matematică: vezi, de exemplu, teorema Van der Corput din lecţia dedicatădistribuţiei uniforme (lecţia 15).Cu toate aceste excepţii, problemele provin în mare parte din cele aflate pe listeleolimpiadelor de matematică dindiverseţări şi ale Olimpiadelor Internaţionale. Mai mult,ambii autori fiind legaţi de aceste competiţii (Titu Andreescu este de peste 30 de ani implicatîn concursurile de matematică dinRomânia şi S.U.A. şi autor a peste 20 de monografii peacest subiect, iar Gabriel Dospinescu, un eminent problemist, fost câştigător al concursuluide admitere la celebra École Normale Superieure din Franţa şi fost olimpic internaţionalal României), aproape 50% din probleme îi au ca autori sau au ca autori matematicieniromâni.Cartea este, după părerea mea şi a multora dintre colegii ce au răsfoit-o sau austudiat-o, un excelent text pentru pregătirea concursurilor de tipul Olimpiadei Internaţionale,concursului Putnam din S.U.A. sau al olimpiadelor studenţeşti.Părerea personală esteîn acelaşi timp că, parcursul cărţii este inegal nu numai îndesfăşurarea lecţiilor, dar şi în cadrul fiecărei lecţii în parte: subiecte simple combinatecu unele nebanale, probleme minunate şi nebanale cu unele prea tehnice şi fără substratmatematic adevărat.Cu toate acestea, acest text nu poate lipsi din biblioteca oricărui matematician pasionatde ,,problem solving“, patina de excepţional problemist şi matematician a lui GabrielDospinescu fiind prezentă lafiecarepas.Radu Gologan


268 RecenziiADRIANA DRAGOMIR, LUCIAN DRAGOMIR,OVIDIU BĂDESCU, ION DAMIAN BÎRCHI, Exerciţii şi probleme dematematică pentruclasaaIX-a,aX-a,aXI-a,aXII-a(şi nu numai)Editura BÎRCHI, Timişoara, 2009Lucrarea reprezintă una dintre cele mai monumentale şi complete culegeri de problemedestinate învăţământului liceal, apărute în ultimii ani. Redactată în patru volume– fiecare dintre ele destinat unei clase de liceu – ea acoperă întreaga programă actualăaliceului, cu cele în jur dde 2000 de probleme pe care le conţine.Rod al unei îndelungate şi vaste experienţe a autorilor, culegerea constituie o selecţievaloroasă a unor probleme atât clasice, cât şi de ultimă oră, apărute în literatura de specialitate(culegeri, reviste etc.) de-a lungul timpului (ne referim la ultimii vreo 40 de ani).Ceea ce îi conferă un statut aparte printre alte demersuri similare este faptul căproblemele au un variat grad de dificultate de ea, putând să se folosească atât elevii cu posibilităţimai modeste cât şi cei ce pregătesc diverse concursuri de matematică sau olimpiade,deci cei care au un potenţial intelectual mai ridicat şi exigenţele pe măsură.Toate problemele sunt însoţite de răspunsuri, indicaţii şi–dacăestecazul–chiarsoluţii complete pentru cele mai dificile.Selecţia atentă şi riguroasă a problemelor oferă cititorului o paletă largă de subiectede reflecţie, pentru cei interesaţi constituind un stimul în perfecţionare şi autodepăşire. Înacest sens vom menţiona cele patru capitole finale, de la sfârşitul fiecărui volum, intitulatesugestiv ,,Probleme pentru olimpiade şcolare şi nu numai“, care constituie fiecare un florilegiual celor mai interesante probleme propuse în decursul timpului la diverse concursuride matematică şi olimpiade. În această ordinedeideivommenţiona faptul că pe acesteliste de probleme apar numele celor mai cunoscuţi propunători, mulţi dintre ei veterani aiacestui tip de activitate.În concluzie, recomandăm cu căldură utilizarea acestei culegeri de probleme, atât decătre elevi cât şi de către profesori, ea constituind un preţios adjuvant atât în munca dezi cu zi, cât şi în pregătirea concursurilor şi olimpiadelor. Pentru a uşura celor interesaţiprocurarea culegerii, vom da adresa de e-mail a editurii: rmt.mate@yahoo.com.Dan RaduERATĂI. În G.M.-A nr. 4/2008, la pag. 359, rândul 2 de jos, se va citi ,,Kalinderu“,în loc de ,,Kolinderu“.II. În G.M.-A nr. 1/2009 se vor face următoarele modificări:• la pag. 61, rândul 20 de sus, în loc de al doilea ,,x 1 “sevaciti,,x 3 “;• la pag. 63, rândul 11 de sus, în loc de ,,V. Chiriţă“ se va citi ,,V. Chiriac“;• la pag. 64, rândul 14 de jos, în loc de ,, 1 2 “sevaciti,,−1 2 “;• la pag. 64, rândul 4 de sus, în loc de ,,BC − QP “sevaciti,,BC||QP “;• la pag. 64, rândul 6 de jos, în loc de ,,b n − 1“ se va citi ,,b n−1 “;• la pag. 66, rândul 5 de sus, în loc de ,,4 cos α “ se va citi ,,4 cos 3 α“;• la pag. 66, rândul 9 de sus, în loc de ,,α =2“sevaciti,,a= 2“;• la pag. 98, rândul 3 de jos, în loc de ,,Mihaela Doinaca“ se va citi ,,MihaelaDoinaru“.Redacţia

Hooray! Your file is uploaded and ready to be published.

Saved successfully!

Ooh no, something went wrong!